CathSap B
CathSap B
Procedural Techniques
Question 1 of 130
#821
A 49-year-old woman presents with acute coronary syndrome, and is found to have severe narrowing of the
proximal right coronary artery and mild atherosclerotic disease in the left anterior descending and circumflex
arteries. She is diabetic, insulin-requiring, and hypertensive, and controlled on an angiotensin-converting enzyme
inhibitor and atenolol. She receives a 3.0 mm x 18 mm everolimus-eluting stent without incident. On angiography,
the lesion appears "hazy" despite post-stent balloon inflation. The operator is uncertain whether the stent is fully
expanded and in apposition to the vessel wall.
Which of the following methods best assesses stent expansion and apposition?
Submit Answer
That is incorrect
• Reasoning
• References
1
Stent expansion and apposition is an anatomic finding and is best determined by IVUS. Optical coherence
tomography can also be used for this purpose. QCA is inadequate for this purpose, as it cannot define the stent and
wall interface. FFR or CFR might demonstrate abnormalities if there is significant underexpansion leading to residual
flow limitation, but cannot discern the more typical, less severe obstruction caused by underexpansion or
malapposition.
Key Point
IVUS and OCT imaging are extremely useful in planning coronary stenting procedures, sizing of the stent,
determining adequacy of stent deployment, and defining potential complications after stenting. The reference
lumen diameter (by IVUS or OCT) is the conventional guide for stent sizing; it frequently leads to selection of a stent
size larger than what would have been expected based on angiography. IVUS media-to-media or midwall sizing is
more aggressive and requires caution and experience.
Related Text
Click the title below to read text related to this question in the Coronary Interventional Equipment and Techniques
chapter/ntravascular Ultrasound and Optical Coherence Tomography: Principles and Clinical Applications module.
Clinical Impact of Optical Coherence Tomography-Guided Perctuaneous Coronary Intervention: Stent Optimization
by Optical Coherence Tomography
Procedural Techniques
Question 2 of 130
#2979
An 84-year-old man with diabetes mellitus and hypertension is admitted with an inferior ST-segment elevation
myocardial infarction (STEMI). You perform coronary angiography via right radial access; the patient has received
aspirin, ticagrelor, and heparin. Entry into the ascending aorta is challenging due to tortuosity of the right subclavian
and brachiocephalic arteries, but you are able to engage the coronaries. The left anterior descending and left
circumflex arteries have nonobstructive disease. The right coronary artery (RCA) has a "shepherd's crook" proximal
portion and a calcified and tortuous mid segment. The culprit lesion is a 95% hazy-appearing stenosis with a filling
defect in the distal RCA.
You attempt percutaneous coronary intervention (PCI) with a 6F JR 4 guide catheter. Unfortunately, every time you
attempt to advance an angioplasty balloon into the proximal RCA, the guide catheter dislodges.
• Try to re-advance the balloon after asking the patient to take a deep breath.
• Rotational atherectomy.
2
Submit Answer
That is incorrect
• Reasoning
• References
This patient is having a STEMI, and likely has thrombus in the culprit lesion. Although rotational atherectomy is an
effective strategy to facilitate PCI in calcified lesions, it is relatively contraindicated in the presence of a large
thrombus burden and would not solve the issue of guide support. Techniques that can optimize guide catheter
support include using the contralateral aorta for support. Femoral access may help improve support, but the patient
has already been anticoagulated, and thus, additional efforts of proceeding with the already established transradial
approach first with a Left Amplatz guide is the next best step. Left radial access is unlikely to provide additional
support in this case and will involve a further access site. While a deep breath may help with balloon advancement,
treating the culprit lesion will likely require a greater degree of support; thus, it is likely best to address the guide
issue first.
Key Point
Failure of adequate guiding catheter support may be resolved by improving passive support (larger caliber guiding
catheters) or active support (deep intubation or using the contralateral aortic root), longer femoral sheaths, and
guide catheter extensions (mother-child systems).
Related Text
Click the titles below to read text related to this question in the Coronary Interventional Equipment and Techniques
chapter/Coronary Guiding Catheters, Guidewires, and Snares module.
Procedural Techniques
Question 3 of 130
#2851
A 76-year-old woman with fatigue comes to you for a second opinion regarding a diagnostic coronary angiogram
that revealed a total occlusion of the mid right coronary artery (RCA) with excellent collaterals from the left anterior
descending artery (LAD) septals. Her primary cardiologist told her that no intervention was needed because of the
robust collateral supply to the distal RCA.
Which of the following statements regarding collateral circulation in chronic total occlusion (CTOs) is most accurate?
• Fractional flow reserve (FFR) will not normalize after revascularization of a CTO.
3
• Recommending CTO percutaneous coronary intervention is contraindicated in a patient who does not have
chest pain.
• Noninvasive stress testing is not valuable in patients with good collateral flow.
Submit Answer
That is incorrect
• Reasoning
• References
Collateral vessels help mitigate the extent of myocardial ischemia and symptoms. However, they typically do not
completely eliminate the ischemic burden. In addition, patients with CTOs often present with fatigue or reduced
exercise tolerance (anginal equivalents) rather than classic chest pain. Even in the presence of good collaterals,
noninvasive stress testing will often demonstrate ischemia. The presence of significant ischemia is an indication for
revascularization. Following revascularization, FFR will generally normalize. Although exercise has been shown to
improve collateralization in patients with peripheral arterial disease, the same has not been shown in the coronary
circulation.
Key Point
Collateral vessels do not typically reduce ischemic burden in the CTO territory.
Related Text
Click the title below to read text related to this question in the Coronary Interventional Equipment and Techniques
chapter/Chronic Total Occlusions module.
Rationale, Indications, and Clinical Benefits of Chronic Total Occlusion Revascularization: Ischemia-Driven
Revascularization of Chronic Total Occlusion
Procedural Techniques
Question 4 of 130
#2847
A 58-year-old man presenting with accelerating angina underwent percutaneous coronary intervention of a calcified
90% stenosis within a tortuous mid left anterior descending artery (LAD). Orbital atherectomy was performed after
which a drug-eluting stent was deployed in the mid LAD. Within minutes, the patient started complaining of severe
chest pain, and the electrocardiogram monitor now shows ST-segment elevation. At this point, repeat coronary
angiography revealed a filling defect in the proximal LAD at the proximal edge of the stent. The nurse reports that
4
the last activated clotting time was just called in as 170 seconds.
Which of the following next steps will best help the operator understand the mechanism of this patient's sudden
deterioration?
• Start infusing adenosine and set up for fractional flow reserve (FFR).
• Perform optical coherence tomography (OCT) because it has better resolution than intravascular ultrasound
(IVUS).
• Perform intravascular ultrasound (IVUS) because it has better penetration than optical coherence
tomography (OCT).
• Perform both intravascular ultrasound (IVUS) and optical coherence tomography (OCT).
Submit Answer
That is incorrect
• Reasoning
• References
This patient is most likely suffering from sudden intracoronary thombus, probably related to subtherapeutic
anticoagulation. The best way to establish this mechanism is with OCT, which has better resolution than IVUS. IVUS
does have better penetration than OCT, but this is a luminal phenomenon and penetration is not advantageous in
this situation. Following OCT, the operator can then decide to aspirate, or balloon, or give intracoronary platelet
inhibitor. FFR is superfluous in this case, as there is already evidence of diminished coronary blood flow.
Procedural Techniques
Question 5 of 130
#888
You perform percutaneous coronary intervention (PCI) on a 55-year-old diabetic man without known peripheral
vascular disease. You insert a vascular closure device at the end of the case.
• His risk of an infection is high, and somewhat higher than with manual compression.
• If he has mycotic pseudoaneurysm, the risk that it will require surgery is low.
• His risk of an infection is very low, but is higher than with manual compression.
That is incorrect
• Reasoning
• References
The risk of an infected access site is low (0.25% overall), but higher than with manual compression, where it is close
to zero (0.06%). Infections occur predominately in diabetic patients (80%), and mycotic pseudoaneurysms have been
reported in >40% of patients. These invariably need to be operated on; thus, once these occur, treating with
antibiotics alone is a mistake. The median time to onset of symptomatic infection is 8 days, with a range of 2 days to
almost a month.
Key Point
Specific ACD complications, including embolization of device, device-mediated leg ischemia, and access site
infection, are uncommon but potentially life-threatening.
Related Text
Click the title below to read text related to this question in the Post-Procedural Care chapter/Arteriotomy Closure
Devices and Complications module.
Procedural Techniques
Question 6 of 130
#2961
You are seeing a 60-year-old active man who presented with angina when he exercises. While his exercise tolerance
is high, he continues to suffer from angina. A coronary angiogram was performed, which revealed a chronic total
occlusion (CTO) of the right coronary artery (RCA) that was partially collateralized.
An initial attempt at percutaneous coronary intervention (PCI) was not successful via an anterograde approach, and
he was referred to you for a second attempt. You review his angiograms. They show an abrupt blunt occlusion of the
RCA at a site of heavy calcification with a long occlusion (>25 mm).
• Calcification.
• Patient age.
• Second attempt.
• Long occlusion.
6
Submit Answer
That is incorrect
• Reasoning
• References
Several risk scores have been used to predict both immediate procedural and clinical success. These include the J-
CTO score (which uses previous failure at PCI, blunt morphology, bending, calcification, and lesion length >20 mm as
variables) and the PROGRESS CTO score (incorporating age >65, lesion length >23 mm, and utilization of a retrograde
approach). This patient has a high J-CTO score predicting a >40% risk of procedural failure. His age (<65 years) makes
him at lower risk.
A recent analysis suggested that all of these scores are only modestly predictive (area under the curve, 0.64-0.69).
Key Point
Several anatomical scores, such as the J-CTO (Multicenter CTO Registry in Japan) score and the PROGRESS CTO
(Prospective Global Registry for the Study of Chronic Total Occlusion Intervention) score, have been proposed to
predict technical and procedural success.
Related Text
Click the title below to read text related to this question in the Coronary Interventional Equipment and Techniques
chapter/Chronic Total Occlusions module.
Contemporary Paradigms for Chronic Total Occlusion Percutaneous Coronary Intervention: Predictors of Technical
and Procedural Success
Procedural Techniques
Question 7 of 130
#2852
A 76-year-old woman with a right coronary artery occlusion presents with exertional angina and undergoes a stress
test showing a large area of inferior ischemia with an ejection fraction of 36%.
In discussing the risks and benefits of chronic total occlusion (CTO) percutaneous coronary intervention with her,
which of the following statements is most accurate?
7
• Her risk of perforation requiring pericardiocentesis is 10%.
Submit Answer
That is incorrect
• Reasoning
• References
CTO is associated with impaired quality of life, poorer tolerance for acute myocardial infarction, reduced regional
ventricular performance, higher likelihood of ventricular arrhythmia, and potentially reduced longevity.
Key Point
CTO is associated with impaired quality of life (QoL), poorer tolerance for acute myocardial infarction, reduced
regional ventricular performance, higher likelihood of ventricular arrhythmia, and potentially reduced longevity.
Related Text
Click the title below to read text related to this question in the Coronary Interventional Equipment and Techniques
chapter/Chronic Total Occlusions module.
Rationale, Indications, and Clinical Benefits of Chronic Total Occlusion Revascularization: Ischemia-Driven
Revascularization of Chronic Total Occlusion/Effect on Mortality
Rationale, Indications, and Clinical Benefits of Chronic Total Occlusion Revascularization: Effect on Left Ventricular
Dysfunction/Effect on Quality of Life
Procedural Techniques
Question 8 of 130
#1392
A 78-year-old diabetic man with hypertension and hyperlipidemia presents with unstable angina and troponin value
suggestive of myocardial injury. Cardiac catheterization is performed and reveals discrete lesions in the mid left
anterior descending (90%), mid circumflex (80%), and proximal right coronary arteries (80%). You elect to perform
multivessel percutaneous coronary intervention (PCI).
Which of the following is most consistent with the benefit of drug-eluting stents (DES) in this patient?
8
• Enhanced quality of life.
Submit Answer
That is correct!
• Reasoning
• References
Compared with bare-metal stents, DES have consistently demonstrated a decreased need for repeat
revascularization of both the target lesion and target vessel, but have no impact on mortality or recurrent MI. Three-
year follow-up of the HORIZONS-AMI trial comparing Taxus DES with bare-metal stents demonstrated that the risk of
MI, stent thrombosis, and death was not different, while Taxus stent usage resulted in significant improvement in
risk of target lesion revascularization.
In the EXAMINATION trial, the use of everolimus-eluting stents (EES) versus bare-metal stents was associated with
no difference in the risk of death, MI, or revascularization at 1 year, but the use of EES was associated with a
decreased risk of target lesion and target vessel revascularization, as well as stent thrombosis at 1 year. These
findings remained robust through 2 years. The recently published randomized NORSTENT study demonstrated that
in patients with stable angina or unstable angina/non-ST-segment elevation MI, use of current-generation
zotarolimus-eluting stents or EES was associated with a lower risk of recurrent revascularization, no difference in MI,
but a significant improvement in revascularization. In addition, DES use was associated with a nominally significant
decreased risk of stent thrombosis (p = 0.0498).
Key Point
DES significantly reduce in-stent restenosis and the need for target lesion and target vessel revascularization
compared with bare-metal stents (BMS) in simple and complex coronary lesions.
Related Text
Coronary Interventional Equipment and Techniques chapter/Permanent Polymer Drug-Eluting Stents module.
Clinical Outcomes With First-Generation DES: Efficacy in Patient and Lesion Subsets/Late Outcomes
Procedural Techniques
Question 9 of 130
#841
A 68-year-old man with prior inferior myocardial infarction 5 years earlier presents with debilitating angina on
maximal medical therapy. Exercise nuclear stress testing is performed, and the test is stopped after 3 minutes due to
chest pain and downsloping anterior ST depression. Nuclear images demonstrate a reversible apical defect and
inferior scar with peri-infarct ischemia. Left ventricular function is normal.
9
On catheterization, he is found to have a mid left anterior descending (LAD) lesion of approximately 65% and a
totally occluded mid right coronary artery (RCA) of 2.5 mm diameter. The LAD provides collaterals to the RCA.
Based on available data, which of the following would be the most appropriate intervention?
• Proceed with percutaneous coronary intervention (PCI) of the LAD with a drug-eluting stent (DES) and then
proceed to PCI of the RCA with a DES.
• Measure fractional flow reserve (FFR) in the LAD; if it is >0.8, proceed to percutaneous coronary intervention
(PCI) of the RCA with a drug-eluting stent (DES).
• Measure fractional flow reserve (FFR) in the LAD; if it is >0.8, proceed to opening of the RCA with a bare-
metal stent.
• Proceed with percutaneous coronary intervention (PCI) of the RCA with a drug-eluting stent (DES).
Submit Answer
That is incorrect
• Reasoning
• References
The restenosis rate for stents placed for chronic total occlusion is greater than that for stenotic lesions without total
occlusion. DES have lower target vessel revascularization rates after PCI for chronic total occlusion. The LAD is
intermediate and should not be treated unless ischemia can be documented. With collaterals to the RCA, an FFR >0.8
is clearly not significant, and will only get better once collaterals are relieved. If FFR of the LAD is significant (<0.8),
then it is reasonable to perform PCI of the LAD with staged revascularization of the RCA as clinically indicated.
Key Point
DES significantly reduce in-stent restenosis and the need for target lesion and target vessel revascularization
compared with bare-metal stents (BMS) in simple and complex coronary lesions.
Related Text
Click the titles below to read text related to this question in the Coronary Interventional Equipment and Techniques
chapter/Permanent Polymer Drug-Eluting Stents module.
Clinical Outcomes With First-Generation DES: Efficacy in Patient and Lesion Subsets/Late Outcomes
Procedural Techniques
Question 10 of 130
#726
10
A 70-year-old man is admitted to the hospital with increasing episodes of chest pain that developed over the past 72
hours. His initial electrocardiogram in the emergency department demonstrated 2 mm of anterior ST-segment
depression that resolved after two sublingual nitroglycerin tablets. His initial troponin was normal. Four months ago,
a bare-metal stent (BMS) was placed in his proximal left anterior descending artery to treat a severe stenosis
associated with unstable angina. Coronary angiography is performed to assess for in-stent restenosis (ISR).
Which of the following best characterizes the process of ISR after a BMS?
• ISR is an ongoing process, which peaks about 1 year after the procedure.
• Intimal hyperplasia is more pronounced after stenting than after balloon angioplasty.
Submit Answer
That is incorrect
• Reasoning
• References
Histologically, ISR is quite distinct from restenosis after balloon angioplasty. Restenosis after balloon angioplasty is
thought to involve elastic recoil of the vessel, negative remodeling or contraction, thrombus at the site of injury,
smooth muscle cell proliferation and migration, and excessive extracellular matrix production. These last two
processes contribute to neointima proliferation or hyperplasia. Alternatively, intravascular ultrasound studies show
that stenting virtually eliminates elastic recoil and negative remodeling, and that ISR is largely a result of neointima
hyperplasia. The neointima is composed principally of proliferating smooth muscle cell and extracellular matrix.
The risk of ISR increases with diabetes, renal dysfunction, longer stents, and smaller diameter stents.
Procedural Techniques
Question 11 of 130
#719
A 72-year-old man is undergoing coronary angiography from the femoral approach after presenting with an acute
coronary syndrome. His left coronary angiogram shows a 90% proximal stenosis of the left anterior descending
artery and an 80% stenosis of the proximal circumflex artery.
You are having difficulty finding the origin of the right coronary artery (RCA) with a 4-curve right Judkins catheter.
After trying several other catheters, the angiogram shown is obtained (Figure 1).
11
Which of the following other catheter shape is most likely to be successful in this setting?
Submit Answer
That is incorrect
• Reasoning
• References
This patient has a more anterior and superior origin of the RCA. Even with a large amount of torque applied, it is
unlikely that a left Judkins catheter would work. Moreover, from its normal position in the left cusp,
counterclockwise torque would move the catheter tip in the wrong direction. A right bypass graft catheter is similar
to a right Judkins catheter, and is unlikely to work. This anatomic situation requires a catheter with a large curve at
the end; thus, an Amplatzcatheter is the most likely to work.
Key Point
There are specific anatomical differences between the right radial, left radial, and femoral approaches that affect the
choice of guiding catheter, and familiarity with a wide range of guiding catheter shapes is required to match the best
12
tool to the anatomy.
Related Text
Click the title below to read text related to this question in the Coronary Interventional Equipment and Techniques
chapter/Coronary Guiding Catheters, Guidewires, and Snares module.
Procedural Techniques
Question 12 of 130
#826
A 75-year-old woman who underwent coronary artery bypass grafting 7 years ago presents with increasing dyspnea
on exertion, limiting her favorite activities. Noninvasive testing showed a scar with a small area of viability in the
inferior wall, and ischemia in the posterior and lateral walls. Her left ventricular ejection fraction was 40%. Coronary
angiography shows three-vessel coronary artery disease (left anterior descending [LAD] occluded in the mid
segment, left circumflex has a long, chronic occlusion in the proximal segment, and the right coronary artery is
occluded in the mid segment). The vein graft to the posterior descending artery is occluded, the vein graft to a large
marginal artery has an 85% stenosis in its proximal portion, and the left internal mammary artery graft to the LAD
graft is patent. Because of her limiting symptoms and objective evidence of ischemia, you decide to treat the
diseased marginal vein graft.
• Attempt to open the occluded vein graft to the posterior descending artery.
Submit Answer
That is incorrect
• Reasoning
• References
Percutaneous treatment of saphenous vein grafts remains challenging for the interventional cardiologist. Lesions in
vein grafts may be friable, and often have associated thrombus that may contribute to the development of slow-flow
(no-reflow phenomenon), distal embolization, and periprocedural myocardial infarction (MI). When feasible, the use
of an embolic protection device is recommended (Class I indication). In the SAFER (Saphenous Vein Graft Angioplasty
13
Free of Emboli Randomized) trial, use of a protection device reduced a composite endpoint of complications from
16.5% to 9.6% in the group using distal protection.
Saphenous vein graft stenting is associated with a higher rate of non–Q-wave MI compared with stenting of native
coronary arteries. Newer stents are certainly easier to use than older stents, but have not lowered the complications
of vein graft stenting. Although some early reports were favorable, neither PTFE-covered stents nor various types of
extraction catheters have been shown to improve outcome in the treatment of vein grafts.
The use of DES appears to reduce further intimal hyperplasia and restenosis. Restenosis and target-vessel
revascularization rates are lower with DES compared with BMS, although mortality and stent thrombosis rates are
similar. In a 2011 meta-analysis, there was a decrease in mortality, MI, and major adverse cardiac events with DES as
compared with BMS in randomized clinical trials (RCTs) and observational trials. When looking only at RCTs, no
difference in mortality was observed.
Key Point
Atherectomy devices, lasers, and covered stents have failed to demonstrate a clinical advantage for patients
undergoing SVG PCI, and are infrequently used.
Related Text
Click the title below to read text related to this question in the Coronary Interventional Equipment and Techniques
chapter/Saphenous Vein Graft and Arterial Graft Intervention module.
Strategies for Saphenous Vein Graft Percutaneous Coronary Intervention: Covered Stents / Atherectomy and
Aspiration
Procedural Techniques
Question 13 of 130
#2864
A 57-year-old man presents to the emergency department via emergency medical service with 2 hours of crushing
substernal chest pressure. His initial electrocardiogram demonstrated ST elevation in leads II, III, and AVF. He
receives aspirin (325 mg) and unfractionated heparin (5,000 units) and reaches the cardiac catheterization
laboratory for emergent coronary angiography using right radial access.
Diagnostic angiography demonstrates a visible thrombus and TIMI 0 flow in the right coronary artery. A 6 Fr JR4
guiding catheter engages the vessel and a non-hydrophilic workhorse wire is used to cross the lesion and appears to
be in the posterior descending artery. Initial predilation with a 2.5 x 15 mm compliant balloon re-establishes TIMI 1-2
flow, but a large mobile partially occlusive thrombus continues to be present.
14
• Aspiration thrombectomy is an acceptable bailout therapy.
Submit Answer
That is incorrect
• Reasoning
• References
Routine aspiration thrombectomy before primary percutaneous coronary intervention is now not recommended
(Class III: No Benefit, Level of Evidence A). There are insufficient data to assess the potential benefit of a strategy of
selective or bailout aspiration thrombectomy (Class IIb, Level of Evidence C-LD).
Filter wires and covered stents in native coronary arteries have not been shown to be effective. An intravenous
platelet inhibitor (i.e., Cangrelor) has not been studied in this patient subset. There is no evidence for switching to
bivalirudin.
Key Point
In STEMI, if after balloon dilation large thrombus persists with reduced flow, thrombectomy may be used as a bailout
therapy.
Related Text
Click the title below to read text related to this question in the Coronary Interventional Equipment and Techniques
chapter/Thrombectomy module.
Manual Aspiration Thrombectomy Devices: Evidence for Efficacy and Outcome of Aspiration Thrombectomy Devices
Procedural Techniques
Question 14 of 130
#2963
You are asked by one of your interventional colleagues to review an angiogram. The patient underwent unsuccessful
recanalization via the antegrade wire escalation strategy of a chronically occluded right coronary artery (RCA) by
your partner several weeks ago. The angiogram revealed significant calcification at the proximal cap of the RCA
chronic total occlusion (CTO), which was ambiguous and has a small side branch. There are several moderate caliber
septal collaterals from the left anterior descending to the distal posterior descending artery, and the distal cap is
tapered and not calcified.
Which of the following can you tell your colleague about the success and complication rates for a retrograde
approach?
• A retrograde approach will increase the procedural success rate, but will increase the rate of complications
including periprocedural myocardial infarction (MI).
• The failure of the antegrade approach predicts the failure of the retrograde approach.
15
• The antegrade approach has been demonstrated to be superior to the retrograde approach in determining
long-term patency for CTO lesions.
• The retrograde approach is not feasible given the calcification of the proximal CTO cap.
• Since there are no epicardial collaterals, the retrograde approach cannot be attempted.
Submit Answer
That is correct!
• Reasoning
• References
In experienced CTO operators, the retrograde approach is highly successful and safe. However, it is associated with a
greater increase in complications, including periprocedural MI. There are no long-term term patency data in head-to-
head trials comparing either approaches. Epicardial collaterals are not the preferred route in the retrograde
approach. Failure of either approaches does not predict outcomes.
Key Point
Strategies available to successfully cross CTOs include antegrade wire escalation, antegrade dissection/re-entry,
retrograde wire escalation, and retrograde dissection/re-entry (controlled antegrade and retrograde tracking [CART]
and reverse CART).
Related Text
Click the title below to read text related to this question in theCoronary Interventional Equipment and Techniques
chapter/Chronic Total Occlusions module.
Contemporary Paradigms for Chronic Total Occlusion Percutaneous Coronary Intervention: Anatomy Dictates
Strategy
Procedural Techniques
Question 15 of 130
#862
A 63-year-old man with chest pain and multiple coronary risk factors is scheduled for cardiac catheterization
following an abnormal stress echocardiogram. His creatinine is 2.1 with an estimated glomerular filtration rate of 39
ml/min/1.73 m2.
Which of the following may help reduce the risk of contrast nephropathy?
16
• Use of acetylcysteine.
Submit Answer
That is incorrect
• Reasoning
• References
Hydration and limiting the amount of contrast given are currently the only procedures for which there is evidence of
some benefit. Current evidence suggests that there is no clear benefit to the use of acetylcysteine for the prevention
of contrast nephropathy. It received a Class III indication in the 2011 guideline for percutaneous coronary
intervention. Metformin has been associated with metabolic acidosis in very rare cases in patients with chronic
kidney disease. It has not been implicated in contrast nephropathy. There are no data that stopping angiotensin-
converting enzyme inhibitors prior to the procedure affects the incidence of contrast nephropathy. Early studies on
the use of furosemide and mannitol failed to show any benefit in reducing the risk of contrast nephropathy.
Procedural Techniques
Question 16 of 130
#2841
A 72-year-old woman status/post failed coronary artery bypass grafting with two occluded grafts, but a patent left
internal mammary artery to left anterior descending artery comes to you for a second opinion regarding attempting
recanalization of an right coronary artery chronic total occlusion (CTO). Your nurse practitioner has prescreened the
patient and has indicated that she denies angina (chest pain).
• You should reassure the patient and tell her that she should return to you when she develops angina.
• CTO percutaneous coronary intervention (PCI) I has lower procedural success in patients over 70 years of
age.
• The patient should be advised to wean off her beta-blocker, and if she develops chest pain subsequently, she
would be a candidate for CTO percutaneous coronary intervention (PCI).
• You should specifically question this patient for angina equivalent symptoms such as dyspnea and fatigue in
addition to angina.
• In the absence of angina, there is no need to get an authorization from the insurance carrier for an imaging
test to assess for myocardial viability.
Submit Answer
17
That is incorrect
• Reasoning
• References
Patients with CTO often are miscategorized as asymptomatic; often the symptoms are dyspnea and fatigue rather
than classic angina pectoris. Among selected US programs, retrograde CTO PCI is often performed in patients with
prior coronary bypass graft surgery, and is associated with favorably high success and low complication rates.
Key Point
Patients with CTO(s) are often incorrectly characterized as asymptomatic; often, patients with CTOs present with
dyspnea and fatigue rather than classical angina pectoris.
Related Text
Click the title below to read text related to this question in the Coronary Interventional Equipment and Techniques
chapter/Chronic Total Occlusions module.
Rationale, Indications, and Clinical Benefits of Chronic Total Occlusion Revascularization: Effect on Left Ventricular
Dysfunction/Effect on Quality of Life
Procedural Techniques
Question 17 of 130
#752
A 75-year-old man who underwent coronary artery bypass grafting 11 years ago presents with unstable angina. His
operation consisted of separate saphenous vein grafts (SVGs) to the left anterior descending artery (LAD), first
obtuse marginal artery, and the posterior descending artery. He is treated with heparin, aspirin 325 mg daily, and
intravenous nitroglycerin, but continues to have episodes of angina with minimal or no activity. Coronary
angiography results are shown in the image (Figure 1).
Which of the following statements best describes the available options for treatment of this SVG lesion?
18
• The use of a covered stent has shown clear superiority over bare-metal stents for saphenous vein lesions,
and should be the treatment of choice.
• Whatever device is used, it is also necessary to use a glycoprotein (GP) IIb/IIIa inhibitor for vein graft
intervention.
• Directional atherectomy would be the optimal treatment based on the CAVEAT II trial results.
Submit Answer
That is incorrect
• Reasoning
• References
The treatment of saphenous venous lesions remains controversial. Directional atherectomy was compared with
balloon angioplasty for the treatment of vein graft stenoses in the CAVEAT II (Coronary Angioplasty Versus Excisional
Atherectomy) trial. In this trial, directional atherectomy was associated with more complications, higher costs, and a
similar restenosis rate at 6 months. Although angiography is not a sensitive method to identify thrombus, this is a
very focal vein graft stenosis, and thrombectomy devices are not necessary.
19
Embolization of large fragments of atherosclerotic debris is always a concern in vein grafts, and the majority of data
suggest that GP IIb/IIIa inhibitors are of limited help. However, some data from the EPIC trial (Evaluation of IIb/IIIa
platelet receptor antagonist 7E3 in Preventing Ischemic Complications trial) suggest that abciximab reduces
complications in the setting of vein-graft interventions. Polytetrafluoroethylene (PTFE)-coated stents have not been
shown to have an advantage, and neither has routine atherectomy. Distal microvascular protection using a balloon
occlusion and aspiration system reduces events compared with percutaneous coronary intervention without an
embolic protection device (SAFER [Saphenous Vein Graft Angioplasty Free of Emboli Randomized] trial), and similar
results have been obtained by use of filter protection devices (FIRE [FilterWire EX Randomized Evaluation] trial).
Key Point
The SAFER (Saphenous Vein Graft Angioplasty Free of Emboli Randomized) trial demonstrated that use of embolic
protection devices (EPDs) was superior to conventional percutaneous coronary intervention (PCI) in saphenous vein
graft intervention.
Related Text
Click the title below to read text related to this question in the Coronary Interventional Equipment and Techniques
chapter/Embolic Protection Devices module.
Review of Clinical Data Supporting Embolic Protection Device Utilization: Proximal Occlusion Devices/Distal Embolic
Protection Devices of the Saphenous Vein Graft
Procedural Techniques
Question 18 of 130
#702
A 65-year-old man with end-stage renal disease experiences angina during every dialysis session. He is referred for
angiography, which shows a severe proximal left anterior descending stenosis approximately 10 mm long. You
decide to examine the stenosis with intravascular ultrasound (IVUS) before performing a coronary intervention. The
IVUS catheter cannot be passed across the stenosis. The proximal reference vessel is shown in Figure 1, and the most
distal image obtainable is shown in Figure 2.
20
Based on the IVUS information, which of the following is the most appropriate next step?
21
• Dilate the stenosis with a 4.5 mm balloon, hopefully avoiding the need for a stent.
• Perform several additional angiograms in different projections to better assess the lesion.
• Directly implant a 4 mm stent mounted on a low-profile delivery system over an extra-stiff guidewire.
Submit Answer
That is incorrect
• Reasoning
• References
The proximal reference image (Figure 1) shows typical moderate remodeling, with a lumen diameter of 3.9 mm and
an adventitial (external elastic membrane) diameter of 4.8 mm. The region of the lesion (Figure 2) shows extensive
calcification. Failure to cross the stenosis with current IVUS devices nearly always implies a very fibrocalcific lesion,
and is considered by many to be an indication for rotational or orbital atherectomy. Although the use of oversized
balloons to perform provisional stenting has been effective in remodeled vessels, the finding of severe calcification
in the region of the stenosis would be a contraindication to this approach.
Although predilation may allow successful IVUS interrogation of the lesion, it would also make rotational or orbital
atherectomy less attractive and would be unlikely to affect the overall outcome of the procedure. Direct stenting in a
calcified vessel would be contraindicated because the stent may be left severely underexpanded. Additional
angiography would not alter the conclusions because IVUS detects vascular calcification with a higher sensitivity than
angiography. Multiple studies have shown poor stent expansion in the presence of calcified lesions, and pre-stent
rotational atherectomy for these lesions has been shown to improve both acute and chronic outcomes. The IVUS
finding of >270 degrees of calcium for >3-5 mm or failure to pass the catheter should be indications for rotational
atherectomy before stenting.
Key Point
IVUS and OCT imaging are extremely useful in planning coronary stenting procedures, sizing of the stent,
determining adequacy of stent deployment, and defining potential complications after stenting. The reference
lumen diameter (by IVUS or OCT) is the conventional guide for stent sizing; it frequently leads to selection of a stent
size larger than what would have been expected based on angiography. IVUS media-to-media or midwall sizing is
more aggressive and requires caution and experience.
Related Text
Click the title below to read text related to this question in the Coronary Interventional Equipment and Techniques
chapter/ntravascular Ultrasound and Optical Coherence Tomography: Principles and Clinical Applications module.
Clinical Impact of Optical Coherence Tomography-Guided Perctuaneous Coronary Intervention: Stent Optimization
by Optical Coherence Tomography
22
Procedural Techniques
Question 19 of 130
#780
A 62-year-old man with a calcified stenosis of the second marginal artery undergoes coronary angioplasty. Inflation
of the angioplasty balloon to 16 atm is required to fully expand the balloon catheter. Shortly after the waist on the
balloon disappears, the pressure recorded by the inflation device suddenly drops and the balloon deflates. Coronary
angiography shows persistence of a moderate stenosis at the lesion site, with contrast staining confined to the vessel
wall and slow-flow into the distal marginal. Chest pain and ST-segment elevation are noted and do not improve after
intracoronary nitroglycerin.
Which of the following is the most appropriate next step in management of this situation?
• An echocardiogram.
• Rotational atherectomy.
• Placement of a stent.
Submit Answer
That is incorrect
• Reasoning
• References
The behavior of the pressure with rapid unexpected deflation suggests balloon rupture likely caused by the
calcification. Frequently, this causes additional damage to the vessel, but in this case, it does not appear to be
severe. Although the balloon catheter was fully inflated, the remaining stenosis is still severe and slow-flow is
present, indicating the need for repeat balloon inflation or a stent. Coronary spasm seems unlikely because
nitroglycerin had already been given. Because the balloon was fully inflated, there is no indication for rotational
atherectomy. Had there been a persistent waist in the balloon, placing a stent as the next step in the procedure
would have been incorrect because complete stent expansion would be uncertain. There is no suggestion of
tamponade or contrast extravasation; thus, an echocardiogram is unnecessary. Calcium channel blockers are not
contraindicated, but are unlikely to provide definitive therapy. Stenting is clearly the best option offered in the
setting of vessel injury and closure.
Key Point
Compared with balloon angioplasty, the use of coronary stents has nearly eliminated abrupt vessel closure in PCI and
significantly reduced the incidence of restenosis.
Related Text
Click the title below to read text related to this question in the Coronary Interventional Equipment and Techniques
chapter/Stents—Bare Metal, Self-Expanding, and Covered—and Coronary Stenting module.
23
Introduction
Procedural Techniques
Question 20 of 130
#889
A 71-year-old man presents with threatened ST-segment elevation myocardial infarction (STEMI) involving the
anterior leads, with resolution of anterior ST elevation upon arrival to the emergency department. His troponin is
elevated, but he adamantly refuses emergent cardiac catheterization, and is ultimately treated with heparin, aspirin,
and ticagrelor, and is discharged without stress testing because he refuses this as well.
He returns with recurrent chest complaints 2 weeks later with his children, who have convinced him to undergo
angiography. He is taken to the catheterization laboratory, where a 50% left anterior descending (LAD) artery lesion
with TIMI 3 flow is seen.
• Perform fractional flow reserve (FFR) on the LAD lesion, and defer percutaneous coronary intervention (PCI)
if FFR >0.80.
Submit Answer
That is incorrect
• Reasoning
• References
A normal FFR is indicative of reversal of myocardial perfusion defects in patients with prior MI. De Bruyne and
colleagues examined FFR in 57 patients who had sustained an MI ≥6 days prior to investigation. Sensitivity and
specificity of an FFR of 0.75 to detect abnormal scintigraphic imaging were 82% and 87%, respectively; the
concordance between FFR and scintigraphy was 85% (p < 0.001). Excluding false-positive and negative studies, the
corresponding values increased to 87%, 100%, and 94%, respectively (p < 0.001). Patients with positive scintigraphic
imaging before PCI had lower FFRs than patients with negative imaging (0.52 ± 0.18 vs. 0.67 ± 0.16; p < 0.008) and
had significantly higher ejection fraction (63 ± 10% vs. 52 ± 10%; p < 0.0009). An FFR >0.75 distinguished patients
after MI with negative scintigraphic imaging. Similar findings relating FFR of infarct-related arteries in patients early
after MI to reversible perfusion defects by single-photon emission computed tomography scanning and myocardial
contrast echocardiography have been reported by Samady and colleagues. IVUS should not be used for intermediate
24
lesion assessment for revascularization outside of the left main coronary artery. PCI of a 50% stenosis should not be
performed without physiologic assessment. Stress testing would be reasonable following the initial presentation, but
if performed after catheterization in this scenario, it would potentially necessitate a second procedure if ischemia
were demonstrated. Therefore, FFR is a more appropriate strategy.
Key Point
The National Cardiovascular Data Registry accepts FFR evaluation of lesion significance as adequate documentation
of ischemia prior to performance of elective percutaneous coronary intervention.
Related Text
Click the title below to read text related to this question in the Coronary Interventional Equipment and Techniques
chapter/Application of Intracoronary Physiology: Use of Pressure and Flow Measurements module.
Procedural Techniques
Question 21 of 130
#772
You are performing coronary angiography on a 68-year-old woman who presents with accelerating angina despite
medical therapy with beta-blockers and a long-acting nitrate, and a positive functional study showing inferior
ischemia. Five years ago, this patient had undergone bypass surgery for multivessel coronary artery disease for
which a saphenous vein graft (SVG) was placed to the distal right coronary artery (RCA). At angiography, you note
that the graft to the RCA is completely occluded, while the mid-RCA has a diffuse 90% stenosis. Grafts are patent to
the left anterior descending and second obtuse marginal (OM2).
Submit Answer
That is incorrect
• Reasoning
• References
It is appropriate to consider percutaneous coronary intervention (PCI) in the prior coronary artery bypass graft
patient who has failed medical therapy and/or who has an abnormal functional test. It is a Class III guidelines
indication (should not be done) to attempt PCI on chronic total vein graft occlusions. The culprit artery accounting
25
for the inferior ischemia is likely the RCA, as the graft to the OM2 is patent. Attempting native RCA PCI is reasonable
and appropriate to help with this patient’s symptoms. Given her accelerating angina symptoms, PCI is likely to result
in the most immediate and more complete relief of her symptoms.
Ranolazine was demonstrated to be ineffective in a reduction of events and symptoms in patients with incomplete
revascularization in the RIVER-PCI (Ranolazine for Incomplete Vessel Revascularization Post-Percutaneous Coronary
Intervention) trial.
Key Point
SVG lesions vary in risk of distal embolization. Risk is minimal for in-stent restenotic lesions and anastomotic lesions,
increases with age of the graft and bulkiness of the lesion, and is highest with chronic totally occluded SVGs.
Related Text
Click the title below to read text related to this question in the Coronary Interventional Equipment and Techniques
chapter/Saphenous Vein Graft and Arterial Graft Intervention module.
Strategies for Saphenous Vein Graft Percutaneous Coronary Intervention: Distal Protection
Procedural Techniques
Question 22 of 130
#693
A 59-year-old man presents with somewhat atypical symptoms for myocardial ischemia 1 year after percutaneous
coronary intervention (PCI) and stent placement in the mid left anterior descending (LAD) artery. An exercise test
with myocardial perfusion study showed an equivocal reversible defect of the anterior wall. Figure 1 is an angiogram
of the LAD. The arrows indicate the stented segment.
Which of the following statements indicates the best course of management for the patient?
26
• Stent placement is indicated because there are sequential stenoses.
• A lumen area of 3.2 mm2 measured by intravascular ultrasound (IVUS) at the distal lesion site would indicate
that PCI should be considered.
• A fractional flow reserve of 0.72 in the LAD measured distal to the stented segment indicates that PCI should
be considered.
• A fractional flow reserve of 0.85 measured distal to the stented segment may be inaccurate because of the
large diagonal branch between the two stenoses.
Submit Answer
That is incorrect
• Reasoning
• References
The stenoses in the LAD appear to be of intermediate severity. In a patient with somewhat atypical symptoms and
equivocal noninvasive findings, further testing to determine the hemodynamic significance of the stenoses is
appropriate before revascularization. This can be performed using a pressure wire to compute the fractional flow
reserve (FFR). The use of IVUS cross-sectional areas should not be used for the assessment of lesion severity for
lesions outside of the left main coronary artery.
27
Sequential stenoses have a greater impact on coronary flow than a single stenosis, but this alone is not a valid reason
to perform PCI in this setting. In this clinical circumstance, simply deciding that stent placement is not indicated
based on the angiogram may be incorrect. There is a large diagonal branch between the two intermediate stenoses,
but this should not affect the FFR if the pressure transducer is in the distal portion of the vessel.
Key Point
While noninvasive testing and angiography remain the cornerstone modalities for evaluating patients with coronary
artery disease, there is a strong rationale for performing fractional flow reserve (FFR) in a growing population of
patients in the catheterization laboratory.
Related Text
Click the title below to read text related to this question in the Coronary Interventional Equipment and Techniques
chapter/Application of Intracoronary Physiology: Use of Pressure and Flow Measurements module.
Introduction/Background
Procedural Techniques
Question 23 of 130
#2632
A 60-year-old diabetic smoker is 10 years post-successful three-vessel coronary artery bypass grafting. He is now
presenting with acute coronary syndrome, a positive troponin, and diagnostic catheterization reveals that his left
internal mammary artery to left anterior descending artery (LAD) is patent, his saphenous vein graft (SVG) to the
small left circumflex artery is occluded, and his SVG to the distal right coronary artery has a 90% discrete mid graft
stenosis.
The operator deploys a distal embolic protection device and proceeds with predilation with a 4.0 noncompliant
balloon angioplasty. This leaves a 30% residual stenosis, TIMI 3 distal flow, and no evidence of distal coronary
embolization.
Which of the following is the most accurate lesson learned with regard to percutaneous coronary intervention (PCI)
applied to a SVG?
• Based on the SAVED trial, a strategy using stents in SVGs was associated with a lower MACE rate over the
follow up period but also a lower procedural success rate than plain old balloon angioplasty (POBA).
• Based on the SAVED trial, a strategy using plain old balloon angioplasty (POBA) instead of bare-metal stent
(BMS) was associated with less major adverse cardiac events (MACE) in SVG interventions.
• Deploying a bare-metal stent (BMS) in order to reduce the residual 30% stenosis to 0% will result in less
restenosis within the SVG.
• A strategy using bare-metal stent (BMS) in SVGs was associated with a higher procedural success rate than
plain old balloon angioplasty (POBA).
• Routine application of a direct stenting strategy, instead of plain old balloon angioplasty (POBA), or balloon
predilation followed by stenting, will avoid no-reflow.
28
Submit Answer
That is incorrect
• Reasoning
• References
The SAVED (Saphenous Vein de Novo) trial reported that compared with balloon angioplasty, BMS were associated
with higher procedural success (92% vs. 69%, p < 0.001), and a lower MACE through 240 days (26% vs. 38%, p =
0.04). However, a reduction in angiographic restenosis (36% vs. 47%, p = 0.11) was NOT significant. Since the SAVED
report, the overwhelming majority of SVG intervention has been performed with stents, and subsequent randomized
trials have compared BMS with covered stents or drug-eluting stents.
Key Point
The SAVED trial demonstrated an increased freedom from MACE with stenting compared with balloon angioplasty in
SVGs, but showed no difference in the rate of angiographic restenosis.
Related Text
Click the title below to read text related to this question in the Coronary Interventional Equipment and Techniques
chapter/Stents—Bare Metal, Self-Expanding, and Covered—and Coronary Stenting module.
Pivotal Clinical Trials for Specific Lesion Subsets: Saphenous Vein Grafts
Procedural Techniques
Question 24 of 130
#803
A 54-year-old man with clinical angina, a positive nuclear stress test with a moderate to large area of anteroapical
ischemia of moderate degree, and guideline-directed medical therapy still has limiting angina. He is referred to you
for catheterization and consideration of revascularization. He gives a history of long-standing allergy to shellfish.
As a result, for the precatheterization orders, you write which of the following?
• Skin test him for contrast allergy and, if positive, pretreat with steroid prophylaxis.
29
Submit Answer
That is incorrect
• Reasoning
• References
It is widely believed that there is cross-reactivity between shellfish allergy and contrast allergy because they both
contain iodine; however, iodine is not the mediator of the allergic reaction in either situation. Contrast allergy is an
anaphylactoid reaction and, as such, cannot be skin tested nor desensitized. True contrast allergies do require
pretreatment, but this patient’s seafood allergy does not make him more likely to have a contrast allergy. Therefore,
the correct answer is no pretreatment, as shellfish allergy is not linked to contrast allergy.
Procedural Techniques
Question 25 of 130
#2862
A 65-year-old woman with chest pain and a positive stress test presents for coronary angiography. Radial access is
attempted, but because of a radial loop and severe pain while trying to traverse it, the operator crosses over to a
femoral approach. Percutaneous coronary intervention (PCI) of a high-grade lesion in the mid left circumflex artery is
performed without difficulties and a suture-based closure device is successfully deployed. The patient was hoping for
a radial procedure because she wanted to go home that same day.
• Vascular closure devices are preferred with same-day discharge due to the reduced risk of vascular
complications.
• The cost savings with transradial same-day PCI has not been observed with transfemoral same-day PCI.
• Femoral access with a vascular closure device does not preclude same-day discharge.
Submit Answer
That is incorrect
• Reasoning
30
• References
The primary benefits of same-day discharge following PCI are related to improved patient satisfaction, decreased
length of stay, and decreased costs. While same-day discharge following transradial PCI has been readily accepted,
there is also sufficient evidence that same-day discharge is appropriate following uncomplicated femoral PCI. In
randomized controlled trials, vascular closure devices have not been shown to reduce bleeding or vascular
complications, but they do promote early ambulation, thereby facilitating same-day discharge. Use of GP IIb/IIIa
inhibitors has been shown to be safe with same-day discharge when only a bolus is given (no infusion). Routine
troponin assessment is not necessary afterwards for uncomplicated PCI.
Key Point
Use of femoral access with ACDs for percutaneous coronary intervention (PCI) may be an alternative to radial for
same-day PCI cases.
Related Text
Click the title below to read text related to this question in the Post-Procedural Care chapter/Arteriotomy Closure
Devices and Complications module.
Procedural Techniques
Question 26 of 130
#937
A 60-year-old woman with a history of hypertension, chronic renal insufficiency, and diabetes presents with several
days of increasing substernal chest pain. Admission electrocardiogram reveals T-wave inversion in leads I, aVL, and
V4-V6. Laboratory results are significant for a positive troponin T of 1.04. She is started on aspirin, clopidogrel, and
intravenous heparin and presents to the cardiac catheterization laboratory for angiography.
The left main artery contains only mild disease; the mid left anterior descending (LAD) artery has a focal 80% lesion
involving the takeoff of a large D1. The left circumflex and right coronary arteries have mild nonflow-limiting disease.
Intervention with a single stent technique is performed. After stenting of the LAD, the large D1 is “jailed.” Rescue
percutaneous transluminal coronary angioplasty (PTCA) of the D1 is performed with a 2.5 mm compliant balloon.
• Intravascular ultrasound.
Submit Answer
That is correct!
31
• Reasoning
• References
According to the Nordic-Baltic Bifurcation Study III, side branch stenosis in the no FKBD group was higher than in the
FKBD group (15.4% compared with 7.9%, p = 0.039). While reasonable to optimize PCI outcomes, the other
strategies have not been associated with improved clinical outcomes.
Key Point
High-pressure kissing balloon inflation, particularly with noncompliant balloons, is an important part of stent
expansion and carinal coverage.
Related Text
Click the title below to read text related to this question in the Coronary Interventional Equipment and Techniques
chapter/Ostial and Bifurcation Lesions module.
Procedural Techniques
Question 27 of 130
#2971
A 67-year-old man is referred to you by your invasive noninterventional colleague for percutaneous coronary
intervention to the left anterior descending artery (LAD) in the setting of a positive stress test and unstable angina.
Initial diagnostic angiography demonstrated a significant mid LAD lesion adjacent to a septal perforator (Figure 1).
You start the procedure with your workhorse wire, but the guidewire persistently advances into the septal branch
and you are unable to cross the lesion despite multiple attempts.
Which of the following options is the most appropriate initial next step?
32
• Use a low tip stiffness hydrophobic wire with a shallow bend 1 mm from the tip.
• Use a hydrophilic guidewire with low tip stiffness, and a microcatheter for support.
• Advance the wire into the septal perforator and perform balloon angioplasty of the LAD into the septal
branch.
• Use a tapered, high tip stiffness guidewire with a shallow bend 1 mm from the tip.
• Use a high tip stiffness (>9 g) hydrophilic wire, and a microcatheter for support.
Submit Answer
That is incorrect
• Reasoning
• References
The significant stenosis of the mid LAD is associated with a side branch, which makes crossing the lesion more
challenging. Initial attempts should be made with the lowest gram-force tip before escalation to heavier wires. A
hydrophilic and/or polymer-jacket 0.014-inch guidewire, with a low gram-force wire may be tried initially. Careful
shaping of the tip is critical to improve procedural success. Microcatheters provide additional support, better wire
torque control, and facilitate wire exchanges.
Key Point
Methods to overcome failure of guidewire lesion crossing include use of hydrophilic and tapered tip guidewires,
specialized guidewire support catheters, and more advanced techniques (e.g., blocking balloon, hairpin wire
33
technique).
Related Text
Click the titles below to read text related to this question in the Coronary Interventional Equipment and Techniques
chapter/Coronary Guiding Catheters, Guidewires, and Snares module.
Inability to Cross the Lesion With a Wire: Severe Rigid Stenosis/Abrupt Angulation/Negotiating Microchannels
Procedural Techniques
Question 28 of 130
#707
A 48-year-old man presents with acute myocardial infarction (MI). While preparing for the catheterization
laboratory, you notice an alternating bundle branch block on the telemetry.
Which of the following should be done before the percutaneous coronary intervention (PCI) procedure is initiated?
Submit Answer
That is incorrect
• Reasoning
• References
Alternating bundle branch block in the setting of acute MI is an indication for temporary pacing. The conduction
system is likely affected by myocardial injury, and can progress to high-grade atrioventricular block or ventricular
tachycardia.
Key Point
Patient preparation for invasive procedures requires knowledge of the particular patient’s medical condition (i.e.,
history, physical, laboratory data, noninvasive testing) and how it relates to and/or dictates the procedures that
need to be performed.
Related Text
Click the title below to read text related to this question in the Fundamentals of Cardiac Catheterization
chapter/Patient Selection, Preparation, Risks, and Informed Consent module.
34
Procedural Techniques
Question 29 of 130
#869
A 60-year-old diabetic man with past medical history of hypertension, hypercholesterolemia, tobacco use, and
coronary artery bypass grafting presents with crescendo angina. The patient is on aspirin, a statin, a beta-blocker,
and an angiotensin-converting enzyme inhibitor.
Urgent cardiac catheterization is performed, and the saphenous vein graft (SVG) to the right coronary artery shows a
hazy, eccentric 90% stenosis with TIMI grade 1 flow, with preserved left ventricular function.
Which of the following is the best adjunct therapy for SVG percutaneous coronary intervention (PCI) in this patient?
• Covered stent.
Submit Answer
That is incorrect
• Reasoning
• References
Embolic protection devices in SVG PCI have been shown to prevent distal embolization and to reduce major adverse
cardiac events. Intra-aortic balloon pump, glycoprotein IIb/IIIa inhibitors, and covered stents have not been shown to
be beneficial in SVG PCI.
Key Point
The SAFER (Saphenous Vein Graft Angioplasty Free of Emboli Randomized) trial demonstrated that use of embolic
protection devices (EPDs) was superior to conventional percutaneous coronary intervention (PCI) in saphenous vein
graft intervention.
Related Text
Click the title below to read text related to this question in the Coronary Interventional Equipment and Techniques
chapter/Embolic Protection Devices module.
Review of Clinical Data Supporting Embolic Protection Device Utilization: Proximal Occlusion Devices/Distal Embolic
Protection Devices of the Saphenous Vein Graft
35
Procedural Techniques
Question 30 of 130
#754
A 56-year-old man in the cardiac catheterization laboratory for primary percutaneous coronary intervention of a
thrombotic occlusion of the proximal left anterior descending artery remains in refractory shock. You decide to place
a mechanical circulatory support device using transseptal access to the left atrium. Echocardiographic guidance is
unavailable.
Which of the following landmarks is most useful for performing transseptal catheterization?
• The spine.
Submit Answer
That is incorrect
• Reasoning
• References
Among the options, the best answer is the aortic valve, which is most closely related to the atrial septum. The spine
is helpful, but less reliable because many patients with heart disease have a rotated or enlarged heart. Depending on
the view (anteroposterior, left or right anterior oblique), the other landmarks will be valuable in determining the
proper place to puncture the interatrial septum.
Key Point
Transseptal access is being used more frequently for an increasing number of structural heart disease procedures.
Related Text
Click the title below to read text related to this question in the Fundamentals of Cardiac Catheterization
chapter/Vascular Access: Arterial, Venous, and Transseptal Access module.
Procedural Techniques
Question 31 of 130
36
#954
A 64-year-old man undergoes successful percutaneous coronary intervention to the right coronary artery with
bivalirudin, aspirin, and clopidogrel. An Angio-Seal device is placed in the common femoral artery access site without
complication. He is discharged home.
He returns to the office within a week, complaining of a low-grade fever, right leg pain, and erythema at the access
site.
• Access from contralateral femoral artery and balloon angioplasty of the affected side.
• Access from contralateral femoral artery and placement of covered stent on the affected side.
Submit Answer
That is incorrect
• Reasoning
• References
The patient has symptoms and findings consistent with access-site infection. All of the commercially available closure
devices may potentially cause infection, although the incidence is exceedingly low. The best treatment strategy is
two sets of blood cultures followed by antibiotics. All patients with a vascular closure device infection should be
considered for surgical debridement.
Watchful waiting is inappropriate due to the potential fatal nature of this complication. The history is not consistent
with pseudoaneurysm (treated with ultrasound-guided compression), femoral artery stenosis (treated with balloon
angioplasty), or arteriovenous fistula (treated with covered stent or surgery).
Key Point
Specific ACD complications, including embolization of device, device-mediated leg ischemia, and access site
infection, are uncommon but potentially life-threatening.
Related Text
Click the title below to read text related to this question in the Post-Procedural Care chapter/Arteriotomy Closure
Devices and Complications module.
Procedural Techniques
#945
A 67-year-old man is referred for a second opinion regarding chronic total occlusion (CTO) revascularization of his
right coronary artery (RCA). He has no significant disease in the left system. His ejection fraction is 60%, and he has
exertional class III angina despite beta-blockers and long-acting nitrates. A nuclear stress test shows a large inferior
wall defect. Angiographically, the RCA CTO is in the mid segment, the lesion length is 35 mm, and the reference
vessel diameter is 3.0 mm. He was told by his cardiologist that percutaneous coronary intervention (PCI) would be
inappropriate based on recently published criteria, and that even if he had a successful procedure, his long-term
outcome would be poor because of high “stent failure” rate.
• Disagree with his primary cardiologist and recommend proceeding with CTO PCI, quoting a clinically driven
target vessel revascularization of approximately 10% at 1 year.
• Agree with his primary cardiologist that CTO PCI in this setting would be inappropriate and that the long-
term results of stenting would be poor due to unacceptably high restenosis rates.
• Recommend CTO PCI, but agree with his cardiologist that the restenosis and stent thrombosis rate would be
very high, likely requiring a repeat procedure within a year.
• Advise him against CTO PCI, since there are a paucity of data to suggest that he would benefit from it.
Submit Answer
That is correct!
• Reasoning
• References
This is an ideal patient for CTO PCI. He is symptomatic despite therapy on two medications and has objective
evidence of significant ischemic burden. Data from multiple registries and a meta-analysis suggest that he would
derive symptomatic and potentially long-term survival benefit with successful CTO PCI. Given his anatomy, the 1-
year target vessel revascularization, myocardial infarction, target vessel failure, and stent thrombosis are expected
to be 9.8%, 1.0%, 10.9%, and 1.0%, respectively.
Key Point
Although randomized trials are lacking, accumulating data from well-conducted registries suggest
that successful percutaneous revascularization of coronary CTO is associated with improved survival, left ventricular
function, and QoL and reduced need for coronary bypass surgery (compared with failed CTO percutaneous coronary
intervention (PCI) attempts, where the target vessel remains occluded).
Related Text
Click the title below to read text related to this question in the Coronary Interventional Equipment and Techniques
chapter/Chronic Total Occlusions module.
Rationale, Indications, and Clinical Benefits of Chronic Total Occlusion Revascularization: Ischemia-Driven
Revascularization of Chronic Total Occlusion/Effect on Mortality
38
Rationale, Indications, and Clinical Benefits of Chronic Total Occlusion Revascularization: Effect on Left Ventricular
Dysfunction/Effect on Quality of Life
Procedural Techniques
Question 33 of 130
#1425
Compared with first-generation drug-eluting stents (DES), second-generation DES are structurally designed to be
more deliverable. Other differences include both safety and efficacy outcomes.
In regards to the Academic Research Consortium definite stent thrombosis, COMPARE (A Trial of Everolimus-Eluting
Stents and Paclitaxel-Eluting Stents for Coronary Revascularization in Daily Practice) demonstrated that a second-
generation everolimus DES compared with a first-generation paclitaxel DES has what?
Submit Answer
That is incorrect
• Reasoning
• References
The COMPARE trial was a single-center, single-blinded (patient), randomized control trial of 1800 patients
undergoing routine percutaneous coronary intervention (PCI). Consecutive patients between 18 to 85 years of age
undergoing PCI were eligible to participate without limitations as to number of lesions or vessels, location of lesions,
or their length. Patients were excluded if they had contraindications to dual antiplatelet therapy in the 2 months
after procedure or planned major surgery within 30 days. The primary endpoint was a composite of all death,
myocardial infarction, and target vessel revascularization (major adverse cardiac events [MACE]). The 30-day and 2-
year data showed reduced MACE and stent thrombosis in the everolimus-treated group.
Key Point
Second-generation DES have improved safety and similar or superior efficacy compared to first-generation DES.
Related Text
Click the title below to read text related to this question in the Coronary Interventional Equipment and Techniques
chapter/Permanent Polymer Drug-Eluting Stents module.
39
Second-Generation Versus First-Generation DES
Procedural Techniques
Question 34 of 130
#796
You are asked to see an 89-year-old man who has a history of hypertension, hyperlipidemia, and diabetes, who had
coronary artery bypass surgery 20 years ago with saphenous vein grafts to the mid left anterior descending artery
(LAD), posterior descending artery, and first obtuse marginal.
He was admitted to the hospital with a non–ST-segment elevation myocardial infarction (NSTEMI) 2 days ago, and
one of your noninterventional partners performed a diagnostic catheterization. She is concerned about a complex
lesion in the mid portion of the vein graft to the LAD that is 20 mm in length. The graft is approximately 4.0 mm in
diameter. She has asked you to perform a percutaneous coronary intervention on the lesion.
• Direct stenting.
Submit Answer
That is incorrect
• Reasoning
• References
This lesion is in a very old vein graft that is likely to embolize material with manipulation. Current guidelines
recommend, as a Class I indication, the use of distal embolic protection in saphenous vein grafts when technically
feasible. This recommendation is without regard to the age of the graft or the clinical presentation (acute or chronic
angina). Direct stenting, debulking technologies, and rheolytic thrombectomy have not been shown to reduce the
incidence of distal embolization, and thus, are inferior strategies to using an embolic protection device.
Key Point
The SAFER (Saphenous Vein Graft Angioplasty Free of Emboli Randomized) trial demonstrated that use of embolic
protection devices (EPDs) was superior to conventional percutaneous coronary intervention (PCI) in saphenous vein
graft intervention.
Related Text
Click the title below to read text related to this question in the Coronary Interventional Equipment and Techniques
40
chapter/Embolic Protection Devices module.
Review of Clinical Data Supporting Embolic Protection Device Utilization: Proximal Occlusion Devices/Distal Embolic
Protection Devices of the Saphenous Vein Graft
Procedural Techniques
Question 35 of 130
#2964
A 67-year-old man is referred for coronary angiography due to exertional angina unresponsive to medical therapy
and an abnormal nuclear study demonstrating a moderate, reversible anterolateral perfusion defect.
Diagnostic angiography is performed using a 6 Fr system from the right radial artery. There is an 80-90% lesion in the
mid segment of the left anterior descending (LAD) extending across a diagonal branch that appears 1.5-2.0 mm. The
LAD lesion appears to involve the ostium of the diagonal branch as well, yielding a more moderate-appearing lesion
at the ostium of the diagonal only.
Which of the following statements best represents the current approach to bifurcation lesions?
• Use of the “crush” technique offers superior outcomes as compared to a provisional approach.
• Get femoral access in anticipation of the need for a second stent for the side branch.
• A provisional side branch stenting strategy is the preferred approach in this clinical scenario.
• Dedicated bifurcation stents perform better than a provisional side branch stenting approach.
Submit Answer
That is incorrect
• Reasoning
• References
Many bifurcation lesions are amenable to PCI. There are a wealth of clinical data supporting a strategy of main vessel
stenting and provisional side branch stenting. These data have demonstrated short- and long-term outcomes. Most
bifurcation lesions and techniques can be performed using 6 or 7 Fr guiding catheters. No conclusive data support a
specific two-stent strategy over a provisional approach.
Key Point
Provisional side branch stenting should be the initial approach in most bifurcation lesions. Many residual stenoses at
the ostium of the SB are clinically silent.
Related Text
Click the title below to read text related to this question in the Coronary Interventional Equipment and Techniques
41
chapter/Ostial and Bifurcation Lesions module.
Procedural Techniques
Question 36 of 130
#2965
A patient of yours who underwent coronary artery bypass grafting (CABG) 2 weeks ago (left internal mammary
artery—left anterior descending, saphenous vein graft [SVG]—DIAG and SVG—right coronary artery [RCA]) presents
at 2:00 a.m. with 5 mm inferior ST elevation. In the catheterization laboratory, you confirm the previously occluded
native RCA and an occluded SVG-RCA in its midsegment. The other two grafts are patent.
Which of the following is the most appropriate approach to a patient with early SVG failure?
• Early SVG failure is most commonly due to underlying harvested vein atherosclerosistic disease.
• Due to a high thrombotic burden and recent surgery, rheolytic thrombectomy is the preferred modality to
remove thrombus in this clinical scenario.
• Neointimal hyperplasia develops in SVGs in the first 2 weeks after surgery and predisposes to early graft
failure.
• Stenting is contraindicated in SVGs within the first month after surgery due to high risk of vessel rupture.
• SVG occlusion in the first month after surgery is due to thrombosis and surgical technical considerations.
Either percutaneous coronary intervention (PCI) of the SVG or PCI of the native RCA is indicated.
Submit Answer
That is incorrect
• Reasoning
• References
Occlusion in the first month after surgery is due to thrombosis and surgical technical problems, occlusion between 1
month and 1 year is due to intimal proliferation, and after 1 year, it is due to the accelerated atherosclerosis in vein
grafts. PCI of the anastomotic sites (but not the body of the graft) of occluded SVGs in the first month after CABG
may be dangerous due to the risk of suture rupture. Rheolytic thrombectomy is not necessarily a preferred strategy
to treat SVG occlusion.
Key Point
Occlusion in the first month after surgery is due to thrombosis and surgical problems, between 1 month and 1 year is
due to intimal proliferation, and after 1 year is due to the accelerated atherosclerosis in vein grafts. Percutaneous
coronary intervention (PCI) of occluded saphenous vein graft (SVG) in the first month after coronary artery bypass
graft (CABG) may be dangerous due to the risk of suture rupture.
42
Related Text
Click the title below to read text related to this question in the Coronary Interventional Equipment and Techniques
chapter/Saphenous Vein Graft and Arterial Graft Intervention module.
Procedural Techniques
Question 37 of 130
#2840
You are asked to do an ad-hoc percutaneous coronary intervention on a 55-year-old male with a low high-density
lipoprotein who has an 80% smooth concentric stenosis in a dominant left circumflex (LCX). The lesion is 1-2 mm
distal to a large obtuse marginal (OM) branch, with no disease at its origin. You are running late for an important
cardiology division meeting. You tell the nurse to set up quickly for a simple direct stent, with a drug-eluting stent to
the main LCX. After deployment of the stent, a new lesion appears in the OM side branch.
Which of the following is the most accurate statement regarding this type of case?
• Given the proximity of the drug-eluting stent to the side branch, the drug will also elute into the side branch
and improve the flow beyond the stenosis.
• Operators should be liberal about wiring large side branches, and be prepared to access the branch with a
balloon if needed.
• It is not necessary to administer nitrates to see if the new side-branch lesion may be transient spasm.
• New side-branch lesions are common, are usually transient, and go away over time.
Submit Answer
That is incorrect
• Reasoning
• References
Always consider an ostial lesion as a possible bifurcation lesion, except for the aorto-ostial location. Be aware of
possible complications involving the nondiseased branch. Be liberal about wiring the other branch, and be prepared
to access the branch with a balloon if needed. Do not dismiss that many lesions on the other branch may be spasms,
resolving with nitrates and time. The FFR measurement in jailed side-branch lesions is both safe and feasible.
Quantitative coronary angiography is unreliable in the assessment of the functional severity of jailed side-branch
lesions, and measurement of FFR suggests that most of these lesions do not have functional significance.
Key Point
Always consider an ostial lesion as a possible bifurcation lesion except for the aorto-ostial location. Be aware about
possible complications involving the nondiseased branch. Be liberal about wiring the other branch and be prepared
43
to access the branch with a balloon if needed. Do not dismiss that many lesions on the other branch may be spasm,
resolving with nitrates and time.
Related Text
Click the title below to read text related to this question in the Coronary Interventional Equipment and Techniques
chapter/Ostial and Bifurcation Lesions.
Procedural Techniques
Question 38 of 130
#2975
While waiting in line at a fast food restaurant, a 63-year-old man with a history of degenerative joint disease suffers
a cardiac arrest. Fortunately, the woman behind him is basic life support (BLS) certified, and immediately initiates
cardiopulmonary resuscitation. An automated external defibrillator is connected, and shock is advised. The patient is
successfully resuscitated, and taken to the local emergency department.
The following week, the patient's daughter calls you stating that he was to undergo elective knee surgery in 2 weeks,
and the orthopedic surgeon's office called her father and told him to stop aspirin and ticagrelor 2 days ago.
• His risk for stent thrombosis is high off all antiplatelet agents, and he should resume both agents
immediately, deferring surgery.
• Admit the patient to the hospital now for bridging therapy with intravenous glycoprotein IIb/IIIa inhibitors
and undergo surgery in the next few days given that he has a window to proceed.
• The risk of stent thrombosis is not serious, but he should restart aspirin now, remain off ticagrelor, and
undergo the surgery in 2 weeks.
• Given that he has tolerated being off antiplatelet agents, he can undergo the surgery as planned.
• Advise that the patient is very high risk, and any future surgery should be avoided.
Submit Answer
That is correct!
• Reasoning
44
• Related Text / Key Point
• References
Based on the 2016 Focused Update on Duration of Dual Antiplatelet Therapy (DAPT), patients with acute coronary
syndromes who undergo PCI with DES should be treated with DAPT for 1 year to reduce the risk of stent thrombosis.
Patients with stable ischemic heart disease who undergo PCI with DES should be treated with DAPT for a minimum
duration of 6 months. This patient was previously scheduled for elective surgery. Given the greater risks of stent
thrombosis associated with premature cessation of DAPT, the recommended duration of DAPT should not be
interrupted.
Premature discontinuation of DAPT is a strong risk factor for stent thrombosis. Other risk factors include diabetes
mellitus, acute coronary syndromes, incomplete stent expansion, undersizing of the stent, bifurcation stenting,
increased stent length, and smaller stent diameter.
Key Point
Premature discontinuation of dual antiplatelet therapy (DAPT) is a strong risk factor for DES thrombosis.
Related Text
Click the title below to read text related to this question in the Coronary Interventional Equipment and Techniques
chapter/Permanent Polymer Drug-Eluting Stents module.
Procedural Techniques
Question 39 of 130
#2616
You are trying to explain how you assess preprocedural risk of percutaneous coronary intervention (PCI) to your
catheterization laboratory staff.
Which of the following statements is most accurate regarding procedural risk during elective PCI?
• Procedural risk during PCI is consistently low when done in an elective fashion.
• The procedural risk is strongly associated with the baseline clinical risk profile of the patient.
• The risk of PCI in the setting of a non-ST-segment elevation myocardial infarction is similar to that of elective
PCI.
• Procedural risk is no longer influenced by left ventricular dysfunction given the availability of percutaneous
hemodynamic support.
Submit Answer
That is incorrect
45
• Reasoning
• References
Procedural risk is typically greater with increasing clinical risk of the patient. Specific factors that contribute to
procedural risk include the acuity of the presentation and other risk modifiers such as ejection fraction. While
hemodynamic support can allow PCI to be done more safely, ejection fraction remains a strong predictor of risk. The
risk of elective procedures may not be uniformly low, particularly if other risk factors are present. Procedural risk can
be calculated using risk predictors, but the actual risk of a procedure is also modifiable by adaptations in procedural
technique.
Key Point
In general, the higher the clinical risk profile of the patient, the higher the risk of the procedure (diagnostic and
therapeutic).
Related Text
Click the title below to read text related to this question in the Fundamentals of Cardiac Catheterization
chapter/Patient Selection, Preparation, Risks, and Informed Consent module.
Procedural Risks
Procedural Techniques
Question 40 of 130
#2618
A 60-year-old woman (height 5 feet, weight 210 lbs) presents for cardiac catheterization for chronic stable angina.
You are assessing her access sites. She has a history of right femoral-popliteal bypass surgery for critical limb
ischemia, and additionally has a history of scleroderma with severe Raynaud's symptomatology of her hands. Radial
pulses are slightly bilaterally diminished, and femoral pulses and distal lower extremity pulses are normal.
Which of the following correctly reflects considerations for access site in this patient?
• Right brachial artery access is the best access site for this patient.
• The femoral crease on the left is a reliable predictor of common femoral access on the left side.
• Her risk of access site complications is equivalent with right and left femoral artery access.
• Obesity is associated with an increased risk of retroperitoneal hemorrhage with femoral artery access.
Submit Answer
That is incorrect
• Reasoning
46
• Related Text / Key Point
• References
Raynaud's disease is a relative contraindication to radial artery access. Low body mass index, not obesity, is
associated with increased risk of retroperitoneal hemorrhage. While access of the right femoral artery could be
performed, it could also compromise her lower extremity bypass, and the risk of vascular complications is higher on
the right femoral than left, given her history of prior vascular surgery on the right. The brachial approach is
associated with a greater incidence of complications (thrombotic).
Key Point
Regardless of the vascular access site chosen, there should be a careful evaluation of the patient prior to the
procedure to consider factors that may make access from a particular site more favorable, or may exclude access
from a particular site.
Related Text
Click the title below to read text related to this question in the Fundamentals of Cardiac Catheterization
chapter/Vascular Access: Arterial, Venous, and Transseptal Access module.
Procedural Techniques
Question 41 of 130
#710
A 69-year-old man is referred to you for coronary angiography because of limiting angina with a reversible
inferolateral perfusion defect at a workload of 6.5 MET. He has no past history of significant illnesses, but had not
sought medical care for several years. He presents fasting for angiography and his only medical therapy is 81 mg
aspirin daily. Laboratory work early this morning showed a hemoglobin of 9.0 mg/dl with a mean corpuscular volume
of 79. His creatinine was 1.0 and electrolytes were normal.
Which of the following is the most appropriate next step in patient care?
• Recheck the complete blood cell count; it was probably a diluted sample.
• Proceed with coronary angiography, and if disease is discovered that is amendable to percutaneous coronary
intervention (PCI), proceed with bare-metal stent placement.
• Proceed with coronary angiography, and if disease is discovered that is amendable to percutaneous coronary
intervention (PCI), proceed with drug-eluting stent placement.
Submit Answer
That is incorrect
• Reasoning
47
• Related Text / Key Point
• References
This patient has newly recognized microcytic anemia, and although the risk of diagnostic angiography is likely not
greatly increased due to this, subsequent decision making if PCI is contemplated can be problematic due to the risk
of bleeding. If this case was emergent (i.e., an ST-segment elevation myocardial infarction), proceeding with
intervention with a bare-metal stent would be appropriate, acknowledging that the best treatment for restenosis
prevention is a drug-eluting stent. However, in this case, the patient has stable angina with a reasonable workload,
and although inconvenient to the patient and staff, the optimal management strategy would be to evaluate the
etiology of the anemia first. Rechecking the complete blood cell count is unlikely to alter decision making. Since this
patient is not on medical therapy, starting a beta-blocker at this point and evaluating the anemia would be
appropriate.
Key Point
Patient preparation for invasive procedures requires knowledge of the particular patient’s medical condition (i.e.,
history, physical, laboratory data, noninvasive testing) and how it relates to and/or dictates the procedures that
need to be performed.
Related Text
Click the title below to read text related to this question in the Fundamentals of Cardiac Catheterization
chapter/Patient Selection, Preparation, Risks, and Informed Consent module.
Procedural Techniques
Question 42 of 130
#782
A 65-year-old widow with medical history significant for diabetes and hypertension is admitted to your coronary care
unit after having a large anterior wall myocardial infarction (MI) that had been treated with primary stenting 48
hours earlier. She has been chest pain free since then, but has not been out of bed. She is taking all the medicines
you prescribed, including an antidepressant. She is not a smoker, and was very active prior to her MI. Her vital signs,
including her pulse oximeter, are normal. She has clear lungs and normal heart sounds. You order phase I cardiac
rehabilitation to be started, and the patient is evaluated and begins the program. On the second day, the cardiac
rehabilitation nurse informs you that your patient is doing well and she is ready to go home. After seeing the patient
on rounds, you agree to discharge her home.
48
Submit Answer
That is correct!
• Reasoning
• References
After discharge from the hospital, the patient should start phase II of cardiac rehabilitation, which includes a low-
level exercise program 3-5 times a week at a specialized rehabilitation facility with cardiac monitoring during
exercise, and not with a personal trainer. Bedrest is contraindicated if the patient does not have other medical
reasons to be in bed. Walking one mile might be achievable by the patient, but this can start after phase II of cardiac
rehabilitation is completed.
Key Point
As with all PCI patients, post-CABG patients who undergo PCI should have risk factors optimized and should be
referred to cardiac rehabilitation programs.
Related Text
Click the title below to read text related to this question in the Coronary Interventional Equipment and Techniques
chapter/Saphenous Vein Graft and Arterial Graft Intervention module.
Procedural Techniques
Question 43 of 130
#842
A 72-year-old woman with no significant cardiac history who presented with exertional chest pain and shortness of
breath undergoes a stress test during which she develops an early hypotensive response with diffuse ST-segment
depression. She has a markedly abnormal myocardial perfusion imaging study showing transient left ventricular (LV)
dilation with mild LV dysfunction, and she is referred for coronary angiography, which is performed using 4 Fr
catheters. The left main artery is calcified with 40% diffuse angiographic stenosis, and the left anterior descending
and left circumflex arteries both have 30-40% proximal stenoses. The right coronary artery (RCA) is co-dominant, and
has a mid-vessel 90% stenosis.
Based on these angiographic findings, which of the following is the next best step in management of this patient?
• Refer the patient for percutaneous coronary intervention of the right coronary artery.
• Perform intravenous ultrasound or fractional flow reserve measurement of the right coronary artery.
Submit Answer
49
That is correct!
• Reasoning
• References
In the setting of a high-risk stress test (hypotensive response, transient ischemic dilatation) and intermediate left
main coronary artery (LMCA) disease, further interrogation is warranted. The caliber catheters can underestimate or
miss ostial left main disease. Fractional flow reserve and intravenous ultrasound are both acceptable modalities for
assessment of LMCA disease. If negative, percutaneous coronary intervention of the RCA may be considered.
Surgical revascularization is recommended if this patient has significant LMCA disease.
Key Point
Studies comparing preintervention IVUS or OCT with physiology have produced a wide range of minimum lumen
area (MLA) cutoff values. In general, these studies have a high negative predictive value but a low positive predictive
value, indicating that IVUS or OCT may be useful to defer intervention but should not be used as definitive proof that
a lesion is flow limiting. For the left main coronary artery (LMCA), there is better agreement between IVUS and
physiology; and the best cutoff MLA that determines hemodynamic significance is probably <6.0 mm2. For technical
reasons, OCT is limited in assessing LMCA severity.
Related Text
Click the title below to read text related to this question in the Coronary Interventional Equipment and Techniques
chapter/ntravascular Ultrasound and Optical Coherence Tomography: Principles and Clinical Applications module.
Procedural Techniques
Question 44 of 130
#695
A 73-year-old man is referred for coronary angiography for lifestyle-limiting angina on a satisfactory medical
program. A physician’s assistant meets him and his wife in the prep area, and offers a brief description of the
procedure, then gives the patient an opportunity to ask questions. He has none. He signed the comprehensive
consent form, which describes the procedure, risks, goals, and alternatives in great detail, at the request of a nurse.
The patient is found to have single-vessel disease, and during the course of percutaneous coronary intervention (PCI)
suffers an embolic stroke with a dense right upper-extremity paresis. The patient and wife sue the physician, citing a
lack of informed consent.
50
• The consent was not valid because the physician neither obtained the consent nor confirmed that full
disclosure occurred.
• The patient had an opportunity to ask questions, and should have queried the assistant more about the risks
of the procedure.
• The patient signed the consent form; therefore, the physician has no liability.
• The consent was valid because it listed all the potential complications of the procedure.
Submit Answer
That is correct!
• Reasoning
• References
The physician cannot delegate a full discussion regarding the procedure, risks, benefits, and alternatives to another
individual. A signature on a consent form alone is insufficient to prove that informed consent was obtained; cursory
rather than thorough review of the consent forms by patients is well described. Effective communication between
the physician and patient/family/guardian is key in establishing reasonable goals and discussing risks.
Key Point
Informed consent includes the patient’s willingness to undergo the procedure, as well as acknowledgment that the
performing physician has discussed and confirmed the goals, risks, benefits, and alternatives.
Related Text
Click the title below to read text related to this question in the Fundamentals of Cardiac Catheterization
chapter/Patient Selection, Preparation, Risks, and Informed Consent module.
Informed Consent
Procedural Techniques
Question 45 of 130
#868
A 46-year-old woman with dyslipidemia, obesity, and hypertension is referred to you for a second opinion, given that
she has ongoing chest pain despite a negative workup. She describes a very active lifestyle working as a neonatal
critical care unit nurse, two children, and an exercise program until 2 years ago when she developed progressive
chest pain and shortness of breath on exertion and during episodes of mental stress. She has had two previous
negative exercise stress perfusion imaging tests, although she did develop some chest pain on the treadmill with 0.5-
1.0 mm ST depression in lead V5. A 1-year-old coronary angiogram showed minimal luminal irregularities throughout
the coronary tree with no angiographic evidence of obstructive lesions.
Because her symptoms have significantly worsened, you decide to proceed with coronary angiography for further
51
evaluation.
If her coronary anatomy on the angiogram shows minimal luminal irregularities, which of the following intravascular
physiologic tests would you consider?
Submit Answer
That is incorrect
• Reasoning
• References
Given that the patient has angina symptoms in the absence of obstructive coronary disease, the patient may have
microvascular dysfunction. This patient has several cardiac risk factors and persistent angina symptoms despite
negative stress tests and angiograms. Given her persistent and progressive symptoms, it would be reasonable to
proceed with a functional catheterization.
If there is no angiographic disease, there is no indication to perform epicardial lesion assessment, such as FFR, HSR,
or IVUS. It would be reasonable to perform microvascular function (CFR) testing.
Key Point
Patients with nonobstructive epicardial disease and persistent anginal syndromes should be considered for
microvascular function and endothelial function testing.
Related Text
Click the title below to read text related to this question in the Coronary Interventional Equipment and Techniques
chapter/Application of Intracoronary Physiology: Use of Pressure and Flow Measurements module.
Procedural Techniques
Question 46 of 130
#886
A 55-year-old smoker is noted to have a systolic ejection murmur over the left upper sternal border when he visits
his doctor because of progressive shortness of breath and fatigue.
On echocardiography, a gradient is noted across the pulmonary valve measuring 25 mm Hg. There is minimal right
52
heart hypertrophy, and right heart systolic function is preserved.
• Pulmonary valvuloplasty.
• Dobutamine echocardiography.
Submit Answer
That is correct!
• Reasoning
• References
This patient’s gradient falls below the threshold that would normally be associated with pulmonary stenosis severe
enough to require intervention. His symptoms are the nonspecific ones that are associated with pulmonary stenosis,
but are by no means diagnostic. With a straightforward echocardiogram not suggesting right ventricular dysfunction
or hypertrophy, the gradient is unlikely to be enhanced significantly by dobutamine infusion.
The question is designed to address the threshold for intervention (typically at least a 30 mm Hg gradient) and to
highlight the importance of excluding other more likely etiologies in these patients; in this case, pulmonary function
testing should be performed.
Key Point
The cognitive aspect of invasive/interventional practice is as important as the technical expertise. The
catheterization laboratory operator should be able to provide a “mini-consultation” as needed for the referring
physician and patient with regard to how the clinical question can best be answered, the scope of the procedure,
interpretation of the findings, and treatment recommendations in the context of the specific patient’s clinical
situation.
Related Text
Click the title below to read text related to this question in the Fundamentals of Cardiac Catheterization
chapter/Patient Selection, Preparation, Risks, and Informed Consent module.
Operator Preparation
Procedural Techniques
Question 47 of 130
#697
53
A 71-year-old woman is found to have a 70% proximal left anterior descending stenosis and a 90% mid-vessel
stenosis in a large dominant right coronary artery (RCA) after having been found to have an abnormal stress test
showing posterobasal hypokinesis at peak stress after exercising for 7 minutes on a Bruce protocol.
Your partner refers the patient for elective percutaneous revascularization later that day. One hour later, you find
out that the patient is diabetic and reevaluate the revascularization choice.
• The patient has a significantly higher mortality within 5 years after bypass surgery than if she was not
diabetic.
• The patient should undergo immediate drug-eluting stent placement to both of the lesions because this
therapy provides equivalent mortality benefit as compared with coronary artery bypass grafting (CABG), and
superior benefit to an optimal medical therapy regimen without revascularization.
• You advise that the patient should still be treated percutaneously, but not with drug-eluting stents because
her bleeding risk from taking long-term clopidogrel therapy is too high.
• The patient has a similar mortality benefit from percutaneous revascularization regardless of her diabetic
status.
Submit Answer
That is correct!
• Reasoning
• References
The FREEDOM trial was a randomized comparison of drug-eluting stenting versus CABG surgery in diabetic patients
with multivessel coronary artery disease. The study showed a significant reduction in major adverse cardiac events
(including mortality) in the patients treated with CABG.
There is no reason for this patient to suspect that there would be an increased risk of bleeding with dual antiplatelet
therapy for a year. Furthermore, the likelihood of in-stent restenosis in diabetics is significantly higher with bare-
metal stents compared with drug-eluting stents. There is at present no mortality benefit demonstrated for
percutaneous coronary intervention over optimal medical therapy for stable ischemic heart disease.
Key Point
The cognitive aspect of invasive/interventional practice is as important as the technical expertise. The
catheterization laboratory operator should be able to provide a “mini-consultation” as needed for the referring
physician and patient with regard to how the clinical question can best be answered, the scope of the procedure,
interpretation of the findings, and treatment recommendations in the context of the specific patient’s clinical
situation.
Related Text
Click the title below to read text related to this question in the Fundamentals of Cardiac Catheterization
chapter/Patient Selection, Preparation, Risks, and Informed Consent module.
54
Operator Preparation
Procedural Techniques
Question 48 of 130
#2969
A selective guide wire and device portfolio are often favored for a streamlined and step-wise approach to
recanalizing coronary artery chronic total occlusion (CTO) lesions.
Which of the following guide wire and device selection options would be most important for you to consider?
• A nontapered guide wire is recommended for antegrade crossing of a CTO, while a tapered guide wire is
preferred for retrograde crossing and navigating collaterals.
• In knuckle wire antegrade dissection technique, a looped stiff, tapered penetration wire is generally chosen.
• A high–gram-force 0.014-inch guide wire, with a tapered 0.009-inch nonjacketed tip should be used for
antegrade microchannel or soft tissue probing and also for knuckle techniques.
• A guide wire escalation strategy should only be considered as the initial strategy of choice in a >40 mm CTO
with reasonably definable proximal cap and distal target.
• The initial wire to probe the CTO is typically a low–gram-force, hydrophilic/polymer-jacket wire, unless the
proximal cap is clearly blunt, calcified, or ambiguous.
Submit Answer
That is incorrect
• Reasoning
• References
A guide wire escalation strategy should only be considered as the initial strategy of choice in a <20 mm CTO with
reasonably definable proximal cap and distal target. A hydrophilic and/or polymer-jacket 0.014-inch guide wire, low–
gram-force, with tapered 0.009-inch tip, is for antegrade microchannel or soft tissue probing and also for knuckle
techniques. The initial wire to probe the CTO is typically a low–gram-force, polymer-jacket wire, unless the proximal
cap is clearly blunt, calcified, or ambiguous. A looped, polymer-jacket wire (knuckle wire) is generally advanced
without rotation in the direction of the distal CTO segment during knuckle wire antegrade dissection. Finally, a
tapered guide wire is recommended for antegrade crossing of a CTO, while a nontapered guide wire is preferred for
retrograde crossing and navigating collaterals.
Key Point
Detailed knowledge of the CTO toolbox, including wires from each task-based class, is critical to success in CTO PCI.
Related Text
55
Click the title below to read text related to this question in the Coronary Interventional Equipment and Techniques
chapter/Chronic Total Occlusions module.
Contemporary Paradigms for Chronic Total Occlusion Percutaneous Coronary Intervention: Chronic Total Occlusion
Toolbox
Procedural Techniques
Question 49 of 130
#873
A 75-year-old woman with diabetes comes to the emergency room with severe chest pressure and diaphoresis. An
ST-segment elevation myocardial infarction code is called, as she has ST elevation in the anterior leads. Angiography
demonstrates a proximal occlusion of the left anterior descending artery. After re-establishing TIMI 3 flow with
predilation using a 2.0 x 15 mm balloon, you are trying to deliver a stent, but due to the tortuosity and calcification in
the proximal vessel, the stent cannot be advanced across the stenosis.
Which of the following would be the best additional piece of equipment to try?
Submit Answer
That is incorrect
• Reasoning
• References
A guide catheter extension permits very deep intubation of the target vessel, thus providing backup support to
facilitate stent delivery across heavily calcified lesions in tortuous vessels. The Venture catheter is a steerable,
flexible catheter that allows directing guidewires in severely angulated coronary arteries. The Corsair catheter is a
microcatheter with a hydrophilic polymer coating. The Tornus catheter is a stiff support catheter specifically for
chronic total occlusion recanalization.
Key Point
Failure of adequate guiding catheter support may be resolved by improving passive support (larger caliber guiding
catheters) or active support (deep intubation or using the contralateral aortic root), longer femoral sheaths, and
guide catheter extensions (mother-child systems).
Related Text
Click the title below to read text related to this question in the Coronary Interventional Equipment and Techniques
56
chapter/Coronary Guiding Catheters, Guidewires, and Snares module.
Procedural Techniques
Question 50 of 130
#2804
Stenting of the coronary lesion shown in Figure 1 carries increased risk for which of the following?
• Aortocoronary dissection.
• Stent underexpansion.
• Perforation.
Submit Answer
57
That is incorrect
• Reasoning
• References
The left anterior descending artery was imaged with near infrared spectroscopy that demonstrates a near
circumferential lipid core plaque. Stenting of such plaques is more likely to result in distal embolization and
periprocedural myocardial infarction, likely due to lipid fragment embolization and/or thrombus formation. Lipid
core plaques are likely to expand well with balloon inflation and stent deployment. The lesion does not involve the
aorto-coronary ostium; hence, aorto-coronary dissection is unlikely.
Key Point
IVUS and OCT imaging are extremely useful in planning coronary stenting procedures, sizing of the stent,
determining adequacy of stent deployment, and defining potential complications after stenting. The reference
lumen diameter (by IVUS or OCT) is the conventional guide for stent sizing; it frequently leads to selection of a stent
size larger than what would have been expected based on angiography. IVUS media-to-media or midwall sizing is
more aggressive and requires caution and experience.
Related Text
Click the title below to read text related to this question in the Coronary Interventional Equipment and Techniques
chapter/ntravascular Ultrasound and Optical Coherence Tomography: Principles and Clinical Applications module.
Clinical Impact of Optical Coherence Tomography-Guided Perctuaneous Coronary Intervention: Stent Optimization
by Optical Coherence Tomography
Procedural Techniques
Question 51 of 130
#700
A 72-year-old morbidly obese woman is referred for coronary angiography and possible percutaneous coronary
intervention (PCI) because of the recent-onset limiting exertional chest pain consistent with myocardial ischemia.
Her internist heard a systolic ejection murmur and ordered a transthoracic echocardiogram (TTE), which was
performed 1 day before the procedure. It showed normal left ventricular (LV) function and restricted mobility of the
aortic valve leaflets, with a mean aortic valve gradient of 20 mm Hg, although the report notes that the aortic valve
Doppler signals did not appear complete.
Your exam shows a grade 3/6 systolic ejection murmur that radiates to the carotids with no diastolic murmur. Her
carotid upstrokes are slightly diminished. Her electrocardiogram shows increased voltage, but does not meet criteria
for LV hypertrophy.
Which of the following is the most appropriate next step in patient care?
• Cancel the procedure and repeat the TTE to confirm the findings.
• Proceed with coronary angiography and PCI if lesion(s) is amenable, but place Swan-Ganz catheter before
intervention to monitor filling pressures.
Submit Answer
That is incorrect
• Reasoning
• References
Her physical exam suggests the possibility of significant aortic stenosis. The echo reports discordant findings
regarding the aortic valve. Although 2D images suggest restricted mobility, the Doppler suggests mild stenosis, but
the signal may have been incomplete. Significant aortic stenosis could contribute to her symptoms, and thus,
confirming the presence or absence of significant stenosis is critical to this patient’s management. Repeating another
TTE may only result in the same data, but a transesophageal echocardiogram (TEE) might be helpful. However, the
patient is already set for an invasive procedure, and if the aortic stenosis is severe, she would require coronary
angiography in the future. Performing a diagnostic catheterization study to confirm the severity of aortic stenosis is
appropriate when noninvasive information is conflicting.
Key Point
Patient selection for invasive diagnostic/therapeutic catheterization procedures requires the operator to have a clear
understanding of the clinical question to be addressed for a particular patient and how to apply the expertise of the
catheterization laboratory to determine a diagnosis and treatment strategy.
Related Text
Click the title below to read text related to this question in the Fundamentals of Cardiac Catheterization
chapter/Patient Selection, Preparation, Risks, and Informed Consent module.
Procedural Techniques
Question 52 of 130
#766
A 55-year-old woman has a 6-month history of anginal-type chest discomfort. A stress myocardial perfusion study is
abnormal, showing anterior ischemia at a heart rate of 105 bpm. Six years ago, she had a St. Jude aortic valve
replacement for aortic stenosis. Coronary arteriography is requested for evaluation. You decide to use femoral
access.
Which of the following is the most effective management plan for her anticoagulation before cardiac
catheterization?
59
• Give high-dose vitamin K in the 24 hours before the angiogram, and check to make sure the international
normalized ratio is <1.5 before the procedure.
• Discontinue warfarin 3 days before the outpatient catheterization, and check her international normalized
ratio on arrival.
• Admit the patient to the hospital to discontinue warfarin and start intravenous unfractionated heparin.
• Admit the patient to the hospital to discontinue warfarin and start intravenous eptifibatide.
Submit Answer
That is incorrect
• Reasoning
• References
The mechanical prosthesis is well healed, with little likelihood of thromboembolism from a short interruption of
warfarin. This is especially true for a mechanical prosthetic valve in the aortic position compared with the mitral
position. Therefore, admitting the patient to the hospital for a period of heparin anticoagulation is not required and
is more costly (unless the patient has high risk of thrombosis). Fresh frozen plasma would reverse the effect of
warfarin, but is an unnecessary use of a valuable resource and carries a small risk of disease transmission. High-dose
vitamin K therapy to reverse the effect of warfarin is not recommended (unless an emergency situation exists)
because it may create a hypercoagulable state.
There are no data to suggest that short-term use of an intravenous antiplatelet agent would prevent
thromboembolic complications from mechanical valve prostheses. To discontinue warfarin 3 days before the
outpatient catheterization and check her international normalized ratio on arrival provides the appropriate degree of
patient safety with the minimum amount of resource utilization, and is the preferred choice. Outpatient
subcutaneous heparin or low molecular weight heparin during the period of subtherapeutic international normalized
ratio may be considered in patients with high risk of thrombosis (Class II, Level of Evidence B).
Key Point
Patient preparation for invasive procedures requires knowledge of the particular patient’s medical condition (i.e.,
history, physical, laboratory data, noninvasive testing) and how it relates to and/or dictates the procedures that
need to be performed.
Related Text
Click the title below to read text related to this question in the Fundamentals of Cardiac Catheterization
chapter/Patient Selection, Preparation, Risks, and Informed Consent module.
Procedural Techniques
Question 53 of 130
60
#2839
A 70-year-old man presents with dyspnea on exertion and a stress test positive for ischemia. However, using a Bruce
protocol, he exercised for 10 minutes. Diagnostic catheterization reveals a 70% proximal left anterior descending
artery lesion. You request to set up for fractional flow reserve, but the circulating nurse tells you the machine is
broken, so you switch to intravascular ultrasound (IVUS).
• IVUS imaging is inferior to quantitative coronary angiography for measuring plaque burden.
• IVUS imaging allows quantification of plaque volume by tracing the external elastic membrane and lumen
areas.
• IVUS imaging allows the identification of arterial remodeling in the presence of circumferential calcification.
• IVUS imaging allows the identification of positive, but not negative, remodeling.
Submit Answer
That is incorrect
• Reasoning
• References
IVUS imaging allows the identification of the presence and types of arterial remodeling. Complete quantification of a
nonstented lesion is possible by directly tracing the external elastic membrane (EEM) and lumen areas of the
proximal reference, lesion, and distal reference. Derived measures (minimum and maximum EEM and lumen
diameters, plaque and media area and thickness, and plaque burden) can also be calculated. Lesion length can be
assessed using fixed-velocity motorized pullbacks.
Procedural Techniques
Question 54 of 130
#828
A 43-year-old woman with no prior history of atherosclerosis comes to a cardiology appointment due to a premature
family history of coronary heart disease (CHD). She is 61 inches, 180 lbs, and has a waist circumference of 36 inches.
Her blood pressure is 128/86 mm Hg, and the physical exam is otherwise normal. Her laboratory values include a
fasting glucose of 103 mg/dl, total cholesterol 185 mg/dl, triglycerides 160 mg/dl, and high-density lipoprotein
cholesterol (HDL-C) 52 mg/dl.
61
• She requires a statin since she is a CHD risk equivalent.
Submit Answer
That is incorrect
• Reasoning
• References
Metabolic syndrome is not considered a CHD risk equivalent. Blood pressure drug treatment would be required for
>140/90 mm Hg. Her non-HDL-C is only 133 mg/dl, which is not close to requiring drug treatment with only one
National Cholesterol Education Program (NCEP) risk factor. She has all metabolic syndrome risk factors, except for
HDL.
Key Point
Patient selection for invasive diagnostic/therapeutic catheterization procedures requires the operator to have a clear
understanding of the clinical question to be addressed for a particular patient and how to apply the expertise of the
catheterization laboratory to determine a diagnosis and treatment strategy.
Related Text
Click the title below to read text related to this question in the Fundamentals of Cardiac Catheterization
chapter/Patient Selection, Preparation, Risks, and Informed Consent module.
Procedural Techniques
Question 55 of 130
#2972
A 67-year-old woman with known coronary disease and prior percutaneous coronary intervention (PCI) of the right
coronary artery 2 years prior presents with chest pain and inferior ST-segment elevation. She is hemodynamically
stable and referred for immediate catheterization and primary PCI. She received aspirin, ticagrelor, and 4,000 units
of heparin in the emergency room, and she arrives in the catheterization laboratory 30 minutes after presentation
with mild chest discomfort. Her pulse is 75 bpm and blood pressure is 115/80 mm Hg.
Angiography is performed via right radial access. The left system is normal, and the right system is shown in Figure 1.
Anticoagulation is initiated with bivalirudin and the lesion is successfully crossed with a 0.014-inch standard
guidewire.
• There is insufficient evidence to recommend for or against the selective use of aspiration thrombectomy.
• Adjunct glycoprotein IIb/IIIa inhibitors (GPIs) are associated with reduced mortality compared to bivalirudin
monotherapy.
Submit Answer
That is incorrect
• Reasoning
• References
This patient has very late stent thrombosis with what appears to be a significant thrombus burden. She has been
managed appropriately to this point, and the next steps of management should focus on successful revascularization
with minimal downstream embolization. There are multiple potential strategies including (but not limited to) balloon
dilation and aspiration/rheolytic thrombectomy.
While thrombectomy has previously been thought to be of benefit, the routine use of thrombectomy recently has
been called into question due to a lack of overall benefit and increased risk of stroke. Routine thrombectomy is now
considered a Class III: No Benefit recommendation. There is insufficient evidence to recommend for or against
63
selective use of thrombectomy, particularly in cases of large thrombus burden.
Rheolytic thrombectomy has not been shown to be a superior strategy to anything, although it may be intuitively
attractive in certain circumstances. Adjunct GPIs may be of benefit, but mortality reduction has not been
demonstrated. Balloon dilation is a reasonable strategy, but it has not been shown to reduce mortality risk. Direct
stenting is feasible when the distal vessel is visualized, but that is not the case in the current scenario.
Key Point
Clinical utility of mechanical thrombectomy, including for example by rheolytic or physical means, remains uncertain
and therefore should not be used routinely.
Related Text
Click the title below to read text related to this question in the Coronary Interventional Equipment and Techniques
chapter/Thrombectomy module.
Procedural Techniques
Question 56 of 130
#2967
A 60-year-old man had undergone percutaneous revascularization with a single bare-metal stent (BMS) to the right
coronary artery 2 years ago. He was on aspirin and clopidogrel for 1 year, but recently stopped taking aspirin since
he was feeling healthy and doing regular exercise.
He is again admitted with recurrent chest pain and diagnosed with a non-ST-segment elevation myocardial infarction
with mildly elevated troponins. Coronary angiography showed diffuse in-stent restenosis (ISR) of the BMS.
Which of the following is the best treatment option during percutaneous coronary intervention (PCI)?
• Vascular brachytherapy.
Submit Answer
That is incorrect
• Reasoning
• References
64
Although DEB inflation and DES are major options for ISR with BMS or DES, compared with DEB, DES implantation is
associated with better long-term clinical efficacy. Given the diffuse ISR, cutting balloon angioplasty is not the best
response. The long-term efficacy of BVS has not been proven in this scenario. Vascular brachytherapy has been
demonstrated to be less effective than DES for ISR.
Key Point
The two most effective strategies for the treatment of BMS- or DES-ISR are DES implantation and drug-eluting
balloon (DEB) inflation. Compared with DEB, DES implantation is associated with better long-term clinical efficacy,
especially with use of the new-generation metallic everolimus-eluting stent.
Related Text
Click the title below to read text related to this question in the Coronary Interventional Equipment and Techniques
chapter/In-Stent Restenosis module.
Treatment of Restenosis
Procedural Techniques
Question 57 of 130
#876
A 63-year-old man is undergoing primary percutaneous coronary intervention (PCI) for anterior ST-segment
elevation myocardial infarction (STEMI).
Which of the following is most appropriate regarding periprocedural anticoagulation in the setting of STEMI?
• Use of bivalirudin is associated with a lower risk of stent thrombosis compared with unfractionated heparin
(UFH).
• Use of bivalirudin is associated with thrombocytopenia compared with unfractionated heparin (UFH).
• Low molecular weight heparin plus glycoprotein (GP) IIb/IIIa inhibitor is associated with less bleeding.
• Use of bivalirudin is associated with a mortality advantage compared with unfractionated heparin (UFH) and
glycoprotein (GP) IIb/IIIa inhibitors.
Submit Answer
That is incorrect
• Reasoning
• References
In the HORIZONS-AMI trial, bivalirudin compared with UFH and GP IIb/IIIa was associated with a mortality reduction.
Fondaparinux alone is contraindicated because of the association with catheter thrombosis. Low molecular weight
65
heparin was associated with a trend toward less ischemia in the ATOLL study, but has not been associated with any
difference in bleeding. Thrombocytopenia is not a complication of bivalirudin therapy.
Key Point
Radial artery access confers several advantages including lower rates of postprocedural bleeding and earlier mobility
for patients, but is limited by the size of the sheath that can be inserted into the radial artery; and arterial spasm
may be problematic in some patients.
Related Text
Click the title below to read text related to this question in the Fundamentals of Cardiac Catheterization
chapter/Vascular Access: Arterial, Venous, and Transseptal Access module.
Procedural Techniques
Question 58 of 130
#770
A 55-year-old woman is referred for cardiac catheterization because of refractory chest pain and a positive nuclear
stress test. She is 5 feet 2 inches tall and 280 lbs, with a large abdominal pannus. Attempts at right and left radial
access are unsuccessful. You opt to perform coronary angiography from the right femoral artery using Judkins
catheters.
Which of the following landmarks is most associated with puncture of the common femoral artery?
Submit Answer
That is correct!
• Reasoning
• References
Localiziing the site of the skin nick to the inferior border of the femoral head using fluoroscopy is more reliable than
other surface landmarks. The point midway between the anterior superior spine and the pubic ramus is typically the
site of the strongest pulsation of the common femoral pulse. When the tip of the insertion needle is located
66
fluoroscopically in the medial lower quadrant of the femoral head, this location accesses the common femoral artery
80% of the time. Compared with these bony landmarks, the location of the inguinal crease varies with body habitus.
In obese patients, the inguinal crease tends to sag below the inguinal ligament, increasing the possibility of accessing
the superficial or deep femoral artery.
Key Point
Careful identification of anatomic landmarks in the groin is necessary to avoid complications such as retroperitoneal
hematoma, femoral artery pseudoaneurysm, and arteriovenous fistula.
Related Text
Click the title below to read text related to this question in the Fundamentals of Cardiac Catheterization
chapter/Vascular Access: Arterial, Venous, and Transseptal Access module.
Procedural Techniques
Question 59 of 130
#736
A 68-year-old man who had bypass surgery 12 years ago presents with severe angina. Coronary angiography shows
occluded native vessels, 75% bulky stenoses at three sites in the vein graft to the left anterior descending (LAD), a
70% stenosis of the vein graft to the circumflex, and a 50-60% stenosis of the vein graft to the right coronary artery
(RCA). The inferior wall is hypokinetic, and his ejection fraction is 40%.
Which of the following is the best therapeutic option for this patient?
• Stenting of the saphenous vein graft (SVG) to the LAD and the SVG to the circumflex.
• Stenting of the saphenous vein graft (SVG) to the LAD, the SVG to the circumflex, and the SVG to the RCA.
• Redo coronary artery bypass graft (CABG) with left internal mammary artery grafting to the LAD.
Submit Answer
That is incorrect
• Reasoning
• References
The probability of long-term success after stenting the SVG to the LAD would be modest and, considering the disease
in the other grafts, CABG is the best therapeutic option. Once vein grafts develop significant disease, long-term
patency is greatly reduced. Long-term follow-up of post-CABG patients showed that long-term outcome was closely
related to the patency of the LAD or its graft.
67
An alternative treatment strategy is percutaneous coronary intervention of the native coronary arteries in selected
patients.
Key Point
SVG lesions vary in risk of distal embolization. Risk is minimal for in-stent restenotic lesions and anastomotic lesions,
increases with age of the graft and bulkiness of the lesion, and is highest with chronic totally occluded SVGs.
Related Text
Click the title below to read text related to this question in the Coronary Interventional Equipment and Techniques
chapter/Saphenous Vein Graft and Arterial Graft Intervention module.
Strategies for Saphenous Vein Graft Percutaneous Coronary Intervention: Distal Protection
Procedural Techniques
Question 60 of 130
#1391
A 73-year-old man without a prior cardiac event, but with a history of recurrent gastrointestinal (GI) bleeding
secondary to arteriovenous malformations presents to the emergency room after experiencing several hours of
ischemic chest pain. His electrocardiogram (ECG) obtained while having chest pain demonstrated 1-2 mm ST-
segment depression in leads 2, 3, and aVF. He becomes pain free after receiving two sublingual nitroglycerin tablets
and his ECG returns to normal.
His admission laboratories reveal an elevated troponin I at 0.5 ng/ml (normal < 0.02) and a hemoglobin of 10.1 g/dl
(normal 14.0-18.0). He has no further chest pain and coronary angiography is performed the next morning. This
revealed a severe stenosis of the right coronary artery (Figure 1) and normal left coronary arteries. Concerned about
the possibility of recurrent GI bleeding, the operator plans to perform balloon angioplasty alone without stenting.
Which of the following statements is most correct regarding the operator's planned strategy of balloon angioplasty
alone for treatment of this coronary lesion?
68
• In-hospital outcomes are unaffected by the choice of revascularization technique.
• There is approximately a 30-40% chance of the patient developing restenosis within 1 year.
• In this patient subset, the expected 1-year rate of repeat revascularization for balloon angioplasty alone is
the same as for bare-metal stenting.
• The strategy of balloon angioplasty alone is associated with the same likelihood of abrupt vessel closure as a
strategy of planned stenting.
• There is no difference in outcomes after hospital discharge compared to a strategy of bare-metal stenting.
Submit Answer
That is incorrect
• Reasoning
• References
Prior to the implementation of coronary stenting, balloon angioplasty was limited by abrupt vessel closure and high
restenosis rates. Abrupt vessel closure was primary due to dissection and occurred in between 5-10% of cases
selected for balloon angioplasty. Without stents, many of these events resulted in significant myocardial infarctions
and many required emergency bypass surgery as a bail out. Coronary stents have almost eliminated the need for
emergency surgery from dissection and abrupt vessel closure, and thus, the in-hospital outcomes have dramatically
improved with coronary stents. Restenosis rates with balloon angioplasty were very high, with a rate of repeat
revascularization after successful balloon angioplasty of at least 30% and often much higher in certain lesion subsets.
69
Bare-metal stents reduced the rate of restenosis significantly as compared to balloon angioplasty, and drug-eluting
stents have further reduced the rate of restenosis to <10% in the current era.
Key Point
Compared with balloon angioplasty, the use of coronary stents has nearly eliminated abrupt vessel closure in PCI and
significantly reduced the incidence of restenosis.
Related Text
Click the title below to read text related to this question in the Coronary Interventional Equipment and Techniques
chapter/Stents—Bare Metal, Self-Expanding, and Covered—and Coronary Stenting module.
Introduction
Procedural Techniques
Question 61 of 130
#848
Following advancement of the frequency-domain optical coherence tomography (FD-OCT) catheter across a high-
grade proximal left anterior descending stenosis, the patient is noted to have severe angina and ST-segment
elevations. Test injection demonstrates that the catheter is obstructive without any anterograde flow noted past the
stenosis.
Which of the following is the correct next step for this patient?
• Withdraw the OCT catheter and ensure that perfusion has been restored.
Submit Answer
That is incorrect
• Reasoning
• References
As with IVUS, advancement of the OCT catheter can lead to transient vessel occlusion when positioned across a high-
grade stenosis. Given that OCT requires clearance of the blood field to adequately visualize the arterial wall, power
injection of contrast will not likely clear the vessel for imaging and risk dissection. While administration of coronary
vasodilators can relieve vasospasm, which may be contributing to vessel occlusion, immediate withdrawal of the
imaging catheter can most rapidly restore anterograde vessel perfusion.
Key Point
Intravascular ultrasound (IVUS) and optical coherence tomography (OCT) users should be familiar with the following
70
documents: Standards for the Acquisition, Measurement, and Reporting of Intravascular Ultrasound Studies: A
Report of the ACC Task Force on Clinical Expert Consensus Documents; Consensus Standards for Acquisition,
Measurement, and Reporting of Intravascular Optical Coherence Tomography Studies: A Report From the
International Working Group for Intravascular Optical Coherence Tomography Standardization and Validation;
and Expert Review Document Part 2: Methodology, Terminology and Clinical Applications of Optical Coherence
Tomography for the Assessment of Interventional Procedures.
Related Text
Click the title below to read text related to this question in the Coronary Interventional Equipment and Techniques
chapter/ntravascular Ultrasound and Optical Coherence Tomography: Principles and Clinical Applications module.
Introduction
Procedural Techniques
Question 62 of 130
#1390
A 72-year-old man with type II diabetes mellitus and chronic kidney disease presented with continued exertional
chest pain despite optimal antianginal therapy. Stress nuclear perfusion imaging demonstrated a moderate-severe
anterior defect consistent with a large area of reversible ischemia. Diagnostic angiography revealed a heavily
calcified 90% stenosis in the mid–left anterior descending coronary artery lesion adjacent to a diagonal branch. Your
colleague attempts to perform percutaneous coronary intervention but, despite using a noncompliant balloon, is
unable to predilate the lesion. Given there is no evidence of dissection, you suggest using atherectomy to debulk the
lesion.
Which of the following options will maximize the chances of procedural success and minimize the risk of slow
flow/no reflow?
Submit Answer
That is incorrect
• Reasoning
• References
71
• Use of a single burr with burr:artery ratio of 0.5 to 0.6.
Combined with meticulous technique, optimal antiplatelet therapy, vasodilators, flush solution, and provisional use
of atropine, temporary, pacing, vasopressors, and mechanical support may prevent slow flow/no reflow, which in
contemporary series is reported in 0.0% to 2.6% of cases.
Procedural Techniques
Question 63 of 130
#1384
A colleague recently started doing transradial procedures and has had problems with spasm. She asks your advice on
ways of minimizing spasm.
Which of the following has not been shown to help reduce spasm in transradial procedures?
• Spasmolytic cocktail.
• Longer sheaths.
• Sedation.
• Hydrophilic sheaths.
Submit Answer
That is incorrect
• Reasoning
• References
Hydrophilic sheath coating, but not sheath length, reduces the incidence of radial artery spasm during transradial
coronary procedures. In addition, smaller diameter sheaths, spasmolytic cocktail (generally a combination of calcium
channel blocker and nitrate), and sedation have all been shown to reduce spasm.
72
Procedural Techniques
Question 64 of 130
#731
You are asked to perform percutaneous coronary intervention (PCI) on the artery shown in the angiogram (Figure 1),
which was obtained with a 5 French catheter. The patient is a 66-year-old man who has had increasing episodes of
exertional angina despite therapy with metoprolol, amlodipine, isosorbide, atorvastatin, clopidogrel, and aspirin.
Past medical history includes a significant lower gastrointestinal (GI) bleed related to colonic polyps.
Which of the following statements favors the use of a drug-eluting stent (DES) compared with a bare-metal stent
(BMS)?
• This could require bifurcation stenting, and the results of DES are superior to BMS if this is necessary.
Submit Answer
73
That is incorrect
• Reasoning
• References
The angiogram shows a severe stenosis in the proximal circumflex artery. The lesion is short, and the vessel caliber is
large. These, combined with the fact that the patient does not have diabetes or an acute coronary syndrome, would
suggest a low rate of restenosis if treated with a single BMS. The patient has a past history of GI bleeding, which
raises concern about long-term dual antiplatelet therapy; thus, he would favor the use of a BMS. With careful
placement, this lesion can likely be treated with just one stent and no compromise of either distal branch. Several
studies now suggest that the preferred strategy would be to stent the main branch and perform balloon angioplasty
on the side branch only if necessary. However, if bifurcation stenting is required, the results with DES are better than
with BMS.
Key Point
DES significantly reduce in-stent restenosis and the need for target lesion and target vessel revascularization
compared with bare-metal stents (BMS) in simple and complex coronary lesions.
Related Text
Click the title below to read text related to this question in the Coronary Interventional Equipment and Techniques
chapter/Permanent Polymer Drug-Eluting Stents module.
Clinical Outcomes With First-Generation DES: Efficacy in Patient and Lesion Subsets/Late Outcomes
Procedural Techniques
Question 65 of 130
#743
You are asked to perform pulmonary artery catheterization in a patient with suspected pulmonary artery
hypertension and extensive burn injury around the neck and collarbones. The patient has known right and left
femoral venous occlusions from prior cardiac catheterizations as a child. You have never done this before, but recall
hearing about a preferred route to the right heart from the arm in patients undergoing transradial left-heart
catheterization.
Which of the following is key in planning the route to be used for cardiac catheterization?
74
Submit Answer
That is correct!
• Reasoning
• References
The basilic vein along the medial aspect of the arm becomes the axillary vein directly and is preferred for right-heart
access over the cephalic vein, which has an acute bend as it enters the axillary vein that makes catheter
manipulation more difficult. Direct RV puncture would carry high risk. A transhepatic approach is feasible, but carries
a higher risk than the arm vein for access.
Key Point
Regardless of the vascular access site chosen, there should be a careful evaluation of the patient prior to the
procedure to consider factors that may make access from a particular site more favorable, or may exclude access
from a particular site.
Related Text
Click the title below to read text related to this question in the Fundamentals of Cardiac Catheterization
chapter/Vascular Access: Arterial, Venous, and Transseptal Access module.
Procedural Techniques
Question 66 of 130
#775
A 72-year-old woman was admitted with unstable angina and transient ST-segment depression in the anterolateral
leads. She is referred for coronary angiography and two of the views obtained are shown (Figures 1, 2).
Which of the following additional views could also be useful to assess diagonal branch involvement when planning
percutaneous coronary intervention (PCI)?
75
• Right anterior oblique (RAO) caudal.
76
• Anteroposterior.
Submit Answer
That is incorrect
• Reasoning
• References
The images show a mid LAD stenosis. Figure 1 is an RAO view with some caudal angulation. The stenosis is seen, but
there are overlapping diagonal vessels. Figure 2 is an RAO cranial view, which often is an excellent view for
demonstrating mid LAD stenosis, but there may be overlapping marginal arteries, depending on the anatomy and
exact angulation.
Visualizing the diagonal branch ostia is important when planning PCI of the LAD to avoid compromising these
branches, and in this case, more LAO angulation in the cranial views will likely delineate these branches.
Additionally, in this case, the diagonal branch ostia are not well-visualized. A LAO caudal view is likely to have
important foreshortening of the mid LAD, and a pure anteroposterior view is not optimal for the mid LAD.
Key Point
An operator must be knowledgeable regarding the nomenclature of each coronary artery segment, and the
angiographic view used to best visualize each segment.
Related Text
Click the title below to read text related to this question in the Fundamentals of Cardiac Catheterization
chapter/Optimal Angiography module.
Procedural Techniques
Question 67 of 130
#2976
You perform a heart catheterization on a 48-year-old man with increasing, but relatively stable angina. He is found to
have a relatively diffuse 90% lesion in the mid left anterior descending artery with diffuse nonobstructive coronary
artery disease in other territories. Before proceeding with percutaneous coronary intervention (PCI), he wants to
discuss stent choice.
77
Which of the following best reflects current data concerning the use of these bioresorbable vascular scaffolds (BVS)?
• Use of a BVS is associated with a lower risk of very late stent thrombosis.
• Meta-analyses demonstrate no difference in ischemic events between drug-eluting stents (DES) and BVS.
Submit Answer
That is incorrect
• Reasoning
• References
While BVS has theoretical appeal, recent clinical data are less reassuring. Meta-analyses have suggested that the rate
of myocardial infarction (MI) and stent thrombosis after deployment of BVS is higher than with conventional drug-
eluting stent use, and recently presented 3-year data from the ABSORB II trial demonstrate a higher risk of vessel-
related MI and scaffold-related thrombosis in patients treated with BVS. In addition, BVS use did not improve
vasomotor reactivity, a theoretical attractive property of BVS use. Clinical trials of BVS have focused on relatively
simple lesions.
Key Point
Adequate lesion preparation and postdilation may improve the safety and efficacy of percutaneous coronary
intervention when bioresorbable scaffolds are implanted.
Related Text
Coronary Interventional Equipment and Techniques chapter/Bioresorbable Polymer and Non-polymeric Drug-Eluting
Stents and Bioresorbable Scaffolding module.
Procedural Techniques
Question 68 of 130
#769
An 87-year-old woman who resides in an assisted living complex is brought to the emergency department (ED)
complaining of acute shortness of breath. An electrocardiogram performed in the ED shows 4 mm ST elevations in
V2-V6.
Her family arrives shortly thereafter. Her son is a noninvasive cardiologist who works at a neighboring hospital and is
very concerned that his mother receives optimal care. You are an accomplished interventionalist, and your hospital’s
laboratory is proficient in primary percutaneous coronary intervention for acute myocardial infarction. The family,
78
including her son, is pushing for an emergency catheterization, but the patient is concerned and not sure she wants
the procedure.
Which of the following is the most appropriate next step in her management?
• Take the patient immediately to the catheterization laboratory and attempt emergency angioplasty because
this is a lifesaving procedure.
• Discuss the options again with the patient, explaining that her prognosis is very poor with conservative
medical therapy. Answer any questions she poses. If she opts for catheterization, proceed with
catheterization. If she declines, admit her to the coronary care unit.
• Tell the patient she is going to die if you do not perform a catheterization emergently, and that if she does
not agree to this, she should find another doctor to care for her.
• Discuss the grave prognosis with the family, and have them convince their mother that she should go
emergently to the catheterization laboratory.
• Have the patient’s son sign a consent form for his mother because she is acutely ill, and then take her to the
catheterization laboratory emergently.
Submit Answer
That is incorrect
• Reasoning
• References
Regardless of the clinical situation or family pressures, a competent patient has the right to determine his or her
care. Circumventing this right by going to the family, by coercion, or by threatened abandonment is not ethical.
Key Point
Informed consent includes the patient’s willingness to undergo the procedure, as well as acknowledgment that the
performing physician has discussed and confirmed the goals, risks, benefits, and alternatives.
Related Text
Click the title below to read text related to this question in the Fundamentals of Cardiac Catheterization
chapter/Patient Selection, Preparation, Risks, and Informed Consent module.
Informed Consent
Procedural Techniques
Question 69 of 130
#1383
79
Recanalization of chronic total occlusions (CTO) has been associated with reductions in angina, decreased ischemic
burden, and improved left ventricular function. However, CTO percutaneous coronary intervention (PCI) is not
routinely performed.
What are the estimates of the current prevalence of CTO in diagnostic angiography and PCI success rates in
unselected United States practice?
Submit Answer
That is incorrect
• Reasoning
• References
Coronary chronic total occlusions (CTOs) are commonly encountered complex lesions identified in 15% to 30% of all
patients referred for coronary angiography. In a meta-analysis including 23 observational studies of patients referred
for angiography because of persistent angina and/or presence of ischemia in functional study the success rate for
percutaneous CTO recanalization was 71.2%.
Procedural Techniques
Question 70 of 130
#2629
A 42-year-old woman from the Caribbean with history of rheumatic heart disease presents with multiple bouts of
recurring chest pain. She has had long-standing atrial fibrillation (AF) with well-controlled ventricular response, but is
still menstruating. She does not have a history of diabetes or hypertension. A computed tomography coronary
angiogram is performed showing a noncalcified severe (90%) proximal left anterior descending (LAD) filling defect,
with otherwise normal coronary arteries and a coronary calcium score of 0. Coronary angiography confirms a hazy
90% LAD lesion with otherwise normal coronaries and the patient is being prepared for an ad hoc percutaneous
coronary intervention.
Which of the following would provide the best insight into the etiology of her LAD pathology?
80
• Optical coherence tomography.
• Intravascular ultrasound.
Submit Answer
That is incorrect
• Reasoning
• References
This patient is very unlikely to be suffering from atherosclerotic heart disease and plaque. Therefore in the absence
of calcified plaque, intravascular ultrasound will be less sensitive at detecting the etiology of the LAD lesion than
optical coherence tomography (OCT). Near-infrared spectroscopy is unlikely to add much in this case without
additional imaging. The lesion is already 90% with unstable coronary syndrome so there is no incremental value to
be gained from fractional flow reserve or instantaneous wave-free ratio (particularly regarding etiology). This patient
with AF very likely sustained a thromboembolism to her LAD, and OCT would be the current gold standard to prove
it.
Procedural Techniques
Question 71 of 130
#2617
A referring cardiologist sends you a patient, who presents for diagnostic coronary angiography with possible
percutaneous coronary intervention following a nuclear stress test that demonstrated mild inferior ischemia.
When you speak with the patient, he states that he has an unlimited exercise tolerance and underwent the stress
test as a "precaution." During the test, he states that he had no symptoms and feels like he was able to exercise for
"at least 10 minutes."
Which of the following is correct regarding the informed consent for the procedure?
• A signed consent form by the patient is sufficient evidence of informed consent in order to proceed with the
procedure.
• Explanation of the procedural risks, including death, myocardial infarction, bleeding, and infection, is
sufficient to proceed with the procedure.
• If conscious sedation is already administered, a discussion of the risks, benefits, and alternatives of the
procedure with the patient's wife and family is sufficient informed consent .
81
• Prior to the administration of conscious sedation, it is necessary to discuss the indications for angiography
with the patient including risks and alternatives.
• If the patient signs the hospital's consent form, the patient's charted discussion with the referring physician
is sufficient to confirm his understanding of the risks, benefits, and alternatives of the procedure.
Submit Answer
That is incorrect
• Reasoning
• References
Particularly in this case, where the indication is not clear, the onus is on you (the performing physician) to confirm
the indication for the procedure and to explain the risks, benefits, and especially the alternatives to proceeding with
coronary angiography (prior to informed consent). Given that the patient is intact, he himself should consent for the
procedure, rather than his family.
Key Point
Informed consent includes the patient’s willingness to undergo the procedure, as well as acknowledgment that the
performing physician has discussed and confirmed the goals, risks, benefits, and alternatives.
Related Text
Click the title below to read text related to this question in the Fundamentals of Cardiac Catheterization
chapter/Patient Selection, Preparation, Risks, and Informed Consent module.
Informed Consent
Procedural Techniques
Question 72 of 130
#2968
A 60-year-old man presents with chronic stable angina and has an angiogram demonstrating a 20 mm long 90%
stenosis in his proximal left anterior descending artery. He is apprehensive about undergoing percutaneous coronary
intervention with a stent because his father had a stent procedure in 2000 that resulted in "renarrowing" and
required several repeat procedures including vascular brachytherapy.
Which of the following is the most appropriate statement regarding this patient's risk of in-stent restenosis?
• This patient's restenosis rate should be <7% at 1 year with contemporary drug-eluting stents (DES).
• This patient should undergo bare-metal stenting because in this lesion location, there is no restenosis
benefit with drug-eluting stents (DES).
• This patient's restenosis rate is similar to his father's because a predilection to restenosis is inheritable.
82
• A first-generation drug-eluting stents (DES)used to treat this lesion would have similar anti-restenotic
efficacy to a bare-metal stent (BMS) at 1 year.
• Given the patient's concern with in-stent restenosis, plaque modification with a cutting or scoring balloon
should be attempted as a first-line strategy.
Submit Answer
That is correct!
• Reasoning
• References
The rate of restenosis has been dramatically reduced with the use of both first-generation and more contemporary
generation DES compared with BMS. While there are specific lesion subsets in which the absolute risk reductions in
restenosis may be less with DES compared with BMS, for a lesion in this location, DES should be the dominant
strategy.
Key Point
The incidence of ISR showed a dramatic reduction when comparing BMS with first- and new-generation DES.
Related Text
Click the title below to read text related to this question in the Coronary Interventional Equipment and Techniques
chapter/In-Stent Restenosis module.
Incidence of Restenosis
Procedural Techniques
Question 73 of 130
#928
An 85-year-old man is undergoing cardiac catheterization for a non–ST-segment elevation myocardial infarction. His
right common femoral artery has a strong pulse, but initial fluoroscopy of the area shows moderate calcification.
Using the modified Seldinger technique, a 5 Fr sheath is placed. The iliofemoral arteries are extremely tortuous and
this makes it difficult to advance a wire into the abdominal aorta without the use of a JR4 catheter as backup
support. The JR4 is advanced over a wire to the aortic root, but it is very difficult to torque and engage the right
coronary artery.
Which of the following would be the next best step in managing this patient?
That is incorrect
• Reasoning
• References
Longer sheaths from the femoral artery often will allow easier manipulation of catheters due to a reduction of
friction on bends and straightening of the iliac artery. While accessing the radial artery could allow completion of the
case, it would involve a second access site.
Key Point
Regardless of the vascular access site chosen, there should be a careful evaluation of the patient prior to the
procedure to consider factors that may make access from a particular site more favorable, or may exclude access
from a particular site.
Related Text
Click the title below to read text related to this question in the Fundamentals of Cardiac Catheterization
chapter/Vascular Access: Arterial, Venous, and Transseptal Access module.
Procedural Techniques
Question 74 of 130
#2845
A 65-year-old man, prior smoker, and hypertensive, is referred to you for percutaneous coronary intervention (PCI)
after a diagnostic catheterization reveals a 90% stenosis in the mid left anterior descending artery (3.0 mm
diameter), 1 mm beyond the take-off of a large (2.75 mm diameter) diagonal branch. The branch appears to have a
70% ostial stenosis as well. The operator opts for a two-stent strategy.
• Considering the generous caliber of both the main branch and the side branch, a final kissing inflation after
elective double stenting is not necessary.
• In bifurcation PCI, using a bare-metal stent (BMS) yields similar outcomes to a drug-eluting stent (DES).
• Treatment of this bifurcation lesion with two stents (one in main branch and one in side branch) is an
acceptable approach in this high-risk setting.
• Provisional stenting with one stent, and converting to two stents (one in main branch and one in side
branch) in case of a poor angiographic result, is not recommended.
84
Submit Answer
That is incorrect
• Reasoning
• References
In patients with low-risk bifurcation lesions (minimal or moderate ostial side-branch disease [50% diameter stenosis]
of focal length [5-6 mm]), provisional stenting yields similar clinical outcome to elective double stenting, with a lower
incidence of periprocedural biomarker elevation. Conversely, in patients with high-risk bifurcations, elective double
stenting is reasonable to use, with similar procedural success and major adverse cardiac event rates compared with
provisional stenting. Culotte, Crush, and T-stent techniques have been studied in randomized controlled trials. Use of
DES yields better outcomes than BMS, and sirolimus-eluting stents yield better outcomes than paclitaxel-eluting
stents. Clinical evidence supports the use of final kissing balloon inflation after elective double stenting.
Key Point
Treatment of a bifurcation lesion with two stents (main and side branches) as intention to treat is an acceptable
approach in selected lesions.
Related Text
Click the title below to read text related to this question in the Coronary Interventional Equipment and Techniques
chapter/Ostial and Bifurcation Lesions module.
Procedural Techniques
Question 75 of 130
#896
A 67-year-old woman with a history of dyslipidemia, type 2 diabetes, and essential hypertension presented to the
hospital with symptoms consistent with unstable angina. The troponins were negative. There was transient ST-
segment depression that resolved with medical stabilization. Following this, she underwent diagnostic coronary
angiography. Angiography demonstrated a moderate-to-large right coronary artery (RCA). There were two
sequential lesions in the RCA: A 90% mid-RCA and an 80% bifurcation lesion involving the right posterior descending
artery and posterior lateral branch. The left main was angiographically normal. There was a moderately calcified,
tortuous lesion in her left anterior descending artery, which included a stenosis of 85-95%. This lesion involved the
diagonal branch. There were no apparent lesions in the diagonal branch. The left circumflex (LCX) was a moderate
caliber vessel with a stenosis in the proximal LCX of 85%, and a second stenosis of 90% in a large first obtuse
marginal branch.
• Medical therapy.
Submit Answer
That is incorrect
• Reasoning
• References
CABG is the best answer. While a number of studies have shown equipoise between death, myocardial infarction,
and stroke in patients with diabetes undergoing PCI or CABG, this patient’s SYNTAX score is relatively high. The
disease burden is substantial. There is a high likelihood for incomplete revascularization. There is no prior history of
cerebrovascular accident, and the patient is relatively young. All of these items favor CABG over multivessel PCI in
this scenario. CABG would undoubtedly be associated with a higher likelihood of event-free survival if
revascularization were included in the endpoint.
Key Point
Calculation of the anatomic SYNTAX (Synergy between Percutaneous Coronary Intervention With Taxus and Cardiac
Surgery) score, Society of Thoracic Surgeons (STS) score, European System for Cardiac Operative Risk Evaluation
(EuroSCORE), and SYNTAX score II is reasonable to aid the heart team decision-making process.
Related Text
Click the title below to read text related to this question in the Coronary Interventional Equipment and Techniques
chapter/Multivessel Coronary Artery Disease module.
Procedural Techniques
Question 76 of 130
#2959
A 65-year-old man with a history of diabetes, chronic kidney disease (glomerular filtration rate [GFR] 20 ml/min), and
known coronary artery disease (prior stent in the right coronary artery [RCA]) presents to the clinic with a 2-day
history of worsening exertional chest pain and dyspnea. His chest pain resolves with rest and sublingual
nitroglycerin. He reports that his chest pain is similar to his presentation prior to his RCA stent. Vitals are stable. His
medications include aspirin, metoprolol, amlodipine, and atorvastatin.
His coronary angiogram shows a focal 40-50% lesion in a large proximal left main, 40% in-stent stenosis of the RCA
stent, and 40% stenosis in the left circumflex artery.
86
• Medical management.
• Surgical consult.
Submit Answer
That is correct!
• Reasoning
• References
The correct answer is to perform IVUS of the left main lesion. Based on the 2011 percutaneous coronary intervention
guideline, it is a Class IIa recommendation to perform IVUS of an angiographically indeterminate left main lesion, and
Class IIb recommendation for guidance of left main stent placement.
Medical management, surgical consult, and no further intervention are incorrect answers. The patient is on two
antianginal medications and presents with unstable angina. His indeterminate left main lesion needs further
assessment to determine if surgical or percutaneous intervention is required. The LITRO study demonstrated that at
2-year follow-up, patients with left main minimum luminal area = 6 mm2 had good prognosis with only medical
management. OCT is more sensitive than IVUS for detection of ruptured plaque. However, it is technically
challenging to perform OCT in the ostial/proximal left main. Compared with OCT, IVUS is better suited for
ostial/proximal left main assessment. Given his chronic kidney disease stage IV (GFR 15-29), intracoronary contrast
OCT would not be the preferred method of assessing his left main lesion. OCT can be performed with intracoronary
saline.
Fractional flow reserve would be another possible strategy to assess the functional significance of the left main
lesion.
Key Point
Studies comparing preintervention IVUS or OCT with physiology have produced a wide range of minimum lumen
area (MLA) cutoff values. In general, these studies have a high negative predictive value but a low positive predictive
value, indicating that IVUS or OCT may be useful to defer intervention but should not be used as definitive proof that
a lesion is flow limiting. For the left main coronary artery (LMCA), there is better agreement between IVUS and
physiology; and the best cutoff MLA that determines hemodynamic significance is probably <6.0 mm2. For technical
reasons, OCT is limited in assessing LMCA severity.
Related Text
Click the title below to read text related to this question in the Coronary Interventional Equipment and Techniques
chapter/ntravascular Ultrasound and Optical Coherence Tomography: Principles and Clinical Applications module.
87
Procedural Techniques
Question 77 of 130
#863
A 61-year-old man has an intermediate left anterior descending lesion, and fractional flow reserve (FFR) is measured.
The FFR is 0.92 and is derived from the tracing in Figure 1.
Submit Answer
That is incorrect
• Reasoning
• References
The tracing shows damping of the guide catheter, which blocks flow down vessel. Both the guide pressure and wire
pressure are nearly the same. The FFR thus calculates 0.92, but is not correct. This is demonstrated in the full tracing
(Figure 2). FFR does not reflect microvascular disease because derivation excludes resistance. Pressure drift means
identical signals with separation, producing a false gradient and FFR.
To correct this problem, recognize damped arterial waveform, then pull the guide catheter back. This problem was
induced during wire pullback across the lesion. The guide catheter was inadvertently advanced and the FFR changed
88
from abnormal to normal, as shown in Figure 2.
Procedural Techniques
Question 78 of 130
#861
A 67-year-old patient undergoes successful percutaneous coronary intervention. A femoral angiogram performed at
the end of the case is shown (Figure 1). You elect to use a vascular closure device instead of manual compression.
Which of the following is most likely following use of a vascular closure device compared with manual compression?
89
• A higher risk of vascular obstruction.
Submit Answer
That is correct!
• Reasoning
• References
This patient has a relatively good location for femoral access, slightly below the centerline of the femoral head. The
sheath entry is well below the inferior epigastric artery, so the risk of retroperitoneal hemorrhage is not increased.
Because puncture is over the femoral head and well above the femoral bifurcation, there is a good “anvil” for
compression and some of the risks of puncturing into the bifurcation vessels, such as increased risk of
pseudoaneurysm, do not apply. However, note that the common femoral artery is only slightly larger than the
sheath and substantially tapered from the external iliac above it. By quantitative angiography, the common femoral
is <3 mm in diameter. In this setting, most vascular closure devices are not indicated because of concern of vascular
obstruction.
Key Point
Specific ACD complications, including embolization of device, device-mediated leg ischemia, and access site
90
infection, are uncommon but potentially life-threatening.
Related Text
Click the title below to read text related to this question in the Post-Procedural Care chapter/Arteriotomy Closure
Devices and Complications module.
Procedural Techniques
Question 79 of 130
#1393
A 42-year-old man with a history of tobacco use presents to the emergency department (ED) with substernal chest
pain of 2 hours' duration. An electrocardiogram (ECG) reveals peaked Ts with associated ST elevation in leads V2-V6.
The patient is 6 feet tall and weighs 250 lbs. He received 5000 U intravenous heparin and 325 mg aspirin in the ED
and is taken to the cardiac catheterization lab for primary percutaneous coronary intervention (PCI). Hs coronary
angiogram reveals a 99% lesion past the origin of a small high diagonal of a wrap-around left anterior descending
(LAD) coronary artery with staining. He is treated with primary PCI with a 2.75 x 23 mm second-generation drug-
eluting stent with acceptable results, except for transient slow flow. During the case he receives bivalirudin and is
loaded with ticagrelor orally at the time of arrival to the cardiac catheterization lab. His subsequent course is
unremarkable, and he is discharged home 2 days later.
Three days following his discharge, he experiences recurrent chest pain, less intense than before. The evening before
he ran out of ticagrelor which he could not afford, and a prescription for clopidogrel was filled in. He calls emergency
medical services promptly, and his field ECG reveals upright Ts in the anterior leads with minimal ST elevation. He is
taking again emergently to the cardiac catheterization lab and undergoes coronary angiogram and subsequent
intravascular ultrasound (IVUS).
• Thrombotic occlusion of the LAD stent with with stent underexpansion by IVUS.
• Patent LAD stent with residual nonocclusive plaque/thrombus prolapse through the stent struts by IVUS.
• Thrombotic occlusion of the LAD stent with a proximal edge dissection by IVUS.
• Thrombotic occlusion of the LAD with a well-apposed stent with no edge dissections by IVUS.
Submit Answer
That is correct!
• Reasoning
• References
91
Recurrent peaked Ts are indicative of acute ischemia—in this time frame one would expect the Ts to be inverted—an
important diagnostic element for stent thrombosis. Therefore the most likely angiographic finding will be an
occluded LAD. Therefore neither of the patent left anterior descending stent options are likely. In this large man, a
2.75 stent in the LAD is likely undersized; the presence of slow flow also hints at the fact that perhaps less aggressive
post dilation was performed. The most likely IVUS finding is that of an underexpanded stent. Given the stent size,
edge dissection is less likely, therefore thrombotic occlusion of the LAD stent with a proximal edge dissection is
unlikely. The short period of ticagerlor discontinuation is a misleading element, in the absence of a mechanical issue
with the stent missing a dose of ticagrelor is unlikely to cause stent thrombosis. Therefore thrombotic occlusion of
the LAD with a well-apposed stent with no edge dissections is not correct.
Procedural Techniques
Question 80 of 130
#1417
You are called to the cardiac catheterization laboratory to review an angiogram on your clinic patient. The 58-year-
old retired mailman has had several years of shortness of breath that usually develops after walking his dog for
about 10 blocks. He continues to smoke and has been compliant with his medications that include low-dose aspirin,
candesartan, sustained-release metoprolol, long-acting nitrate, and rosuvastatin. During his last clinic visit his blood
pressure was 137/82 mm Hg and low-density lipoprotein cholesterol was 87 mg/dl. He reports increased use of
sublingual nitroglycerin for relief of his exercise-induced shortness of breath.
The right coronary artery is shown in Figure 1. The left coronary system demonstrates only minimal luminal
irregularities.
92
• Perform coronary flow reserve measurement.
• Add clopidogrel and ranolazine and target a low-density lipoprotein-cholesterol level of <70 mg/dl.
Submit Answer
That is incorrect
• Reasoning
• References
The angiogram demonstrates an intermediate proximal right coronary artery lesion. Although the stress test is
abnormal in this area, the angiographic appearance of this lesion is approximately 50% diameter stenosis. It is
important to determine the true hemodynamic significance of this intermediate lesion.
Fractional flow reserve (FFR) identifies physiologically significant coronary stenoses by objectively measuring flow
reduction caused by an atheroma rather than depending on traditional, more subjective angiographic visual
inspection (Figure 2). The FAME 2 (Fractional Flow Reserve Versus Angiography for Multivessel Evaluation 2) trial
randomized 888 patients with stable multivessel coronary artery disease undergoing percutaneous coronary
intervention (PCI) to either FFR-guided PCI plus optimal medical therapy or to optimal medical therapy alone. Only
93
patients with physiologically significant coronary lesions, defined as FFR = 0.80 were randomized. FFR-guided PCI
yielded an 8% absolute risk reduction of the primary endpoint, driven primarily by a reduction in urgent
revascularizations.
Placement of a drug-eluting stent in a hemodynamically nonsignificant lesion would expose the patient to risks of
stunting with no ischemic outcome benefit, and coronary flow reserve measurement in this clinical scenario has not
been shown to improve outcomes and facilitate decision making. The patient is currently receiving optimal medical
therapy and, despite it, experiences anginal equivalent, and while optical coherence tomography provides
exceptional intravascular images, it is unlikely it would add additional information in this lesion.
Procedural Techniques
Question 81 of 130
#718
You are asked to perform cardiac catheterization on a 22-year-old man known to have a large ventricular septal
defect that never was closed. He is cyanotic and has developed Eisenmenger physiology. He is admitted for
hydration before the procedure.
94
Which of the following is the highest priority in his management?
Submit Answer
That is correct!
• Reasoning
• References
In a cyanotic patient with pulmonary hypertension, a substantial proportion of the blood bypasses the lungs. When
hydration is used, careful protection for air bubbles in all IV access is mandatory. Due to the right-to-left shunt
physiology, there is a substantial risk of air embolism during hydration if air filters are not used. The same
physiological principles explain why cyanotic patients are more susceptible to developing brain abscess.
Key Point
Patient selection for invasive diagnostic/therapeutic catheterization procedures requires the operator to have a clear
understanding of the clinical question to be addressed for a particular patient and how to apply the expertise of the
catheterization laboratory to determine a diagnosis and treatment strategy.
Related Text
Click the title below to read text related to this question in the Fundamentals of Cardiac Catheterization
chapter/Patient Selection, Preparation, Risks, and Informed Consent module.
Procedural Techniques
Question 82 of 130
#915
You are discussing the risks of coronary intervention with your fellow.
In the current stenting era, which of the following is the percentage of cases that is complicated by acute abrupt
vessel closure?
• <1%.
• 5%.
95
• 15%.
• 10%.
Submit Answer
That is correct!
• Reasoning
• References
The majority (greater than two-thirds) of abrupt closures occur while the patient is still in the catheterization
laboratory. The other cases occur most often in the first 6 hours after angioplasty and are associated with
discontinuation of anticoagulation, platelet transfusion, and hypotension. Stents have the ability to scaffold
disruptions in the arterial wall, thereby simultaneously treating the acute dissection and preventing its extension.
Key Point
Compared with balloon angioplasty, the use of coronary stents has nearly eliminated abrupt vessel closure in PCI and
significantly reduced the incidence of restenosis.
Related Text
Click the title below to read text related to this question in the Coronary Interventional Equipment and Techniques
chapter/Stents—Bare Metal, Self-Expanding, and Covered—and Coronary Stenting module.
Introduction
Procedural Techniques
Question 83 of 130
#2962
You are starting a case of a right coronary artery chronic total occlusion (CTO). The lesion is short (12 mm) with
favorable antegrade anatomy as well as potential septal collaterals for a retrograde approach. The initial antegrade
wire is placed in the subintimal space.
• Perform an angioplasty with a small balloon at high pressures to extend the lumen.
• Immediately switch to a retrograde approach using a septal perforator from the left anterior descending
artery.
• Abort the procedure, as the wire has exited the true lumen.
• Advance the wire as far distally as possible into the subintimal space to allow for plenty of potential access
space back into the true lumen.
96
• Perform an antegrade re-entry into the true lumen using a wire-based technique (e.g., sideport re-entry
balloon catheter).
Submit Answer
That is incorrect
• Reasoning
• References
Percutaneous recanalization of coronary CTO historically has been limited by the inability to consistently achieve
wire position in the distal true lumen across the CTO segment. The three major CTO crossing techniques currently
being used are: 1) antegrade wire escalation, 2) antegrade dissection/re-entry, and 3) retrograde. Following the
hybrid algorithm for CTO crossing techniques enhances successful revascularization.
Key Point
Percutaneous recanalization of coronary CTO historically has been limited by the inability to achieve a wire position
in the true lumen distal to a CTO.
Related Text
Click the title below to read text related to this question in the Coronary Interventional Equipment and Techniques
chapter/Chronic Total Occlusions module.
Procedural Techniques
Question 84 of 130
#908
You have just completed a single-vessel right coronary artery stent placement in a previously healthy 70-year-old
woman who presented with Canadian Cardiovascular Society class III angina pectoris. Thirty minutes after you
perform femoral artery closure with a vascular closure device, the catheterization laboratory nurse calls to inform
you that the patient’s blood pressure is now 70/40 mm Hg, with a heart rate of 110 bpm.
Which of the following is the most likely cause of postpercutaneous coronary intervention (PCI) hypotension in this
patient?
• Cardiac tamponade.
• Retroperitoneal hemorrhage.
Submit Answer
97
That is incorrect
• Reasoning
• References
Following femoral access, the most likely complications are associated with bleeding. Randomized controlled trials
have demonstrated that vascular closure devices do not reduce the risk of bleeding complications.
Coronary artery perforation can complicate almost any PCI, but it is unlikely in this uncomplicated case. Similarly,
delayed reactions to contrast or sedatives are rare. Contrast allergy should be considered more likely if cutaneous
manifestations or evidence of airway obstruction is also found. A midazolam reaction would likely also include
severe respiratory depression, and would be unlikely in this time frame. In any case, rapid administration of
intravenous fluids, possibly vasopressors including epinephrine, and diagnostic studies to identify the cause should
be considered. Vasovagal reactions are common and should be considered in the differential diagnosis, but they are
generally characterized by hypotension and bradycardia.
Key Point
ACD use is optimal when used to close common femoral artery access sites, avoiding both high (external iliac artery
access) and low (superficial femoral or profunda femoral artery access) arteriotomies.
Related Text
Click the title below to read text related to this question in the Post-Procedural Care chapter/Arteriotomy Closure
Devices and Complications module.
Procedural Techniques
Question 85 of 130
#706
A noninterventional cardiologist just performed coronary angiography on a 57-year-old man because of severe
exertional angina despite medical therapy. The patient was anxious and received 10 mg of diazepam orally before
the procedure and additional fentanyl and midazolam during the procedure. His angiogram shows a severe proximal
left anterior descending stenosis amenable to stent placement, which was the patient’s expressed treatment
preference if feasible.
He is sent to the holding area with the femoral artery sheath in place until the interventional cardiologist arrives to
perform the procedure. He is easily arousable, and while in the holding area, a physician assistant asks if he has any
questions and has him sign the consent for percutaneous coronary intervention (PCI). The interventional cardiologist
confirms the consent issues with the patient.
During the procedure, the vessel dissected and abruptly closed, causing severe chest pain and ST-segment elevation.
Although flow was restored and the stenosis was successfully treated with a stent, there was enzyme evidence of a
new infarction; thus, he is hospitalized for 2 days after the PCI. The patient sues, claiming he was not informed of
98
this potential complication.
• The consent is valid because the consent form clearly lists myocardial infarction as a possible complication.
• Because a consent form was signed, the patient cannot file a lawsuit.
• The consent is invalid, as the patient had received sedation shortly before the consent was obtained.
• The consent is valid because the patient had two opportunities to discuss the risks of PCI.
Submit Answer
That is incorrect
• Reasoning
• References
Full disclosure should occur before beginning the case, and before the patient receives sedative medications that can
affect decision-making capabilities and recall. In cases where PCI may be performed at the same sitting, a full
discussion of goals, risks, and alternatives of angiography as well as PCI should be undertaken before angiography. A
mutually agreed upon plan by the operator, patient, and/or family to proceed with PCI under these conditions, or
potentially defer PCI if the risk appears higher than expected/discussed, is acceptable. If a discussion regarding PCI
has not occurred prior to angiography, it is most appropriate to allow the effects of sedative agents to dissipate to
have a full discussion with the patient. In the nonurgent setting, that may mean deferral of PCI until the following
day.
Key Point
Informed consent includes the patient’s willingness to undergo the procedure, as well as acknowledgment that the
performing physician has discussed and confirmed the goals, risks, benefits, and alternatives.
Related Text
Click the title below to read text related to this question in the Fundamentals of Cardiac Catheterization
chapter/Patient Selection, Preparation, Risks, and Informed Consent module.
Informed Consent
Procedural Techniques
Question 86 of 130
#2802
99
Which of the following is the best way to prevent this complication?
Submit Answer
That is incorrect
• Reasoning
• References
Long arterial sheaths can be used to facilitate guide catheter manipulation and prevent guide catheter kinking.
Catheter rotation in either direction (especially when forceful) may worsen the problem, and use of a guide catheter
extension is unlikely to help. Guide catheter kinking should be suspected when the pressure waveform is lost during
catheter manipulations or when the tip of the catheter does not move despite proximal torquing.
Key Point
Guiding catheter and guidewire performance is closely tied to their structures.
100
Related Text
Click the title below to read text related to this question in the Coronary Interventional Equipment and Techniques
chapter/Coronary Guiding Catheters, Guidewires, and Snares module.
Procedural Techniques
Question 87 of 130
#739
A 60-year-old woman with long-standing diabetes and hypertension is hospitalized with progressively worsening
dyspnea. Her chest X-ray demonstrates mild cardiomegaly and pulmonary vascular congestion. Her
electrocardiogram (ECG) is shown (Figure 1).
Left- and right-heart catheterization is performed. Her mean right atrial pressure is 8 mm Hg, and her right
ventricular pressure is 50/10 mm Hg. On advancement of the balloon-tipped right-heart catheter into the pulmonary
artery, she complains of chest discomfort and dizziness. The ECG monitor strip at this time is shown (Figure 2), and
her blood pressure (BP) is 85/50 mm Hg.
Which of the following is the first priority in caring for this patient?
101
• Perform right coronary angiography and angioplasty.
102
• Administer intravenous atropine (1.0 mg).
Submit Answer
That is incorrect
• Reasoning
• References
Right bundle branch block (RBBB) may occur during right-heart catheterization as a result of mechanical pressure
against the right bundle. In patients with pre-existing left bundle branch block, this may result in complete heart
block. In this case, the patient has a reasonable BP and heart rate. The appropriate initial action is to withdraw the
catheter from the right heart. This will often resolve the mechanically induced RBBB. Atropine is unlikely to be
helpful because the heart block is infranodal. If heart block persists after the catheter is withdrawn, a temporary
pacemaker should be inserted immediately. Complete heart block may also occur during retrograde left-sided
cardiac catheterization in patients with pre-existing RBBB and left posterior fascicular block.
Key Point
Patient preparation for invasive procedures requires knowledge of the particular patient’s medical condition (i.e.,
history, physical, laboratory data, noninvasive testing) and how it relates to and/or dictates the procedures that
need to be performed.
Related Text
Click the title below to read text related to this question in the Fundamentals of Cardiac Catheterization
chapter/Patient Selection, Preparation, Risks, and Informed Consent module.
Procedural Techniques
Question 88 of 130
#935
A 63-year-old man is undergoing diagnostic cardiac catheterization for the evaluation of continuing stable angina
despite medical management. He was pretreated several days earlier with clopidogrel and takes aspirin 81 mg daily.
On coronary angiography, there is a severe lesion in the right coronary artery (RCA) and an intermediate lesion in the
left anterior descending (LAD) coronary artery. The decision is made to perform a physiological assessment of the
LAD lesion with fractional flow reserve (FFR) first, possibly followed by percutaneous coronary intervention (PCI) on
the RCA. A bolus of bivalirudin is administered, followed by an intravenous (IV) infusion. Ten minutes later, blood is
drawn from the arterial sheath, and a point-of-care activated clotting time (ACT) (Hemotec) is 158 seconds.
103
• Administer a weight-based dose of intravenous unfractionated heparin (70 U/kg) to achieve a target
activated clotting time of 250-300 sec.
• Confirm with the nurse that the bivalirudin was administered and that the intravenous line is working
adequately.
Submit Answer
That is incorrect
• Reasoning
• References
Whereas there is no quantitative relationship between ACT and ischemic or bleeding events with bivalirudin, there is
a qualitative relationship, as the presence of bivalirudin will prolong ACT. An ACT can therefore serve to confirm that
an anticoagulant effect has been achieved after the drug is administered. In this case, the ACT is consistent with a
nonanticoagulated state because the bivalirudin was not actually administered and/or the IV line was faulty.
Although administering another bivalirudin bolus or unfractionated heparin would be a reasonable eventual course
of action, neither of these strategies will be successful if the IV is not functioning, so this should be assessed first. The
clinical efficacy and safety of prasugrel would not solve the problem of lack of anticoagulation. In this patient
undergoing nonurgent PCI pretreated with clopidogrel, IV eptifibatide represents a PCI guideline Class IIb
recommendation for IV antiplatelet therapy, but again does not address the key problem of a lack of foundation
anticoagulation for the FFR and PCI.
Procedural Techniques
Question 89 of 130
#2631
A 74-year-old diabetic, female, cigarette smoker with two-vessel coronary artery disease, weighs 110 lbs and is 59
inches tall. She presents with prior history of angina pain and a positive treadmill stress test for ischemia. A
diagnostic catheterization reveals small caliber coronary vessels, and quantitative coronary angiography analysis
reveals that the vessel with the 90% stenosis (proximal left anterior descending artery) has a reference diameter of
2.5 mm. The patient's interventional attending opts for percutaneous coronary intervention (PCI) with a 2.5 x 12 mm
bare-metal stent (BMS).
Based on the results of the STRESS and BENESTENT I trials, which of the following is most accurate regarding PCI
options in this patient?
104
• This patient's risk of subsequent myocardial infarction (MI) will be reduced compared with plain old balloon
angioplasty (POBA).
• This patient's risk of subsequent re-hospitalization will be reduced compared with plain old balloon
angioplasty (POBA) for bleeding.
• This patient's risk of subsequent target lesion revascularization will be reduced compared with plain old
balloon angioplasty (POBA).
• BMS is superior to plain old balloon angioplasty (POBA), as it results in reduced late-loss.
• This patient's risk of subsequent target lesion revascularization will be reduced compared with plain old
balloon angioplasty (POBA) if the attending used a drug-eluting stent.
Submit Answer
That is incorrect
• Reasoning
• References
The STRESS and BENESTENT I trials were the initial pivotal trials to demonstrate the superiority of stenting over
balloon PCI in native coronary lesions, with a significantly lower rate of angiographic restenosis and target lesion
revascularization. This study only involved bare-metal stents, as drug-eluting stents were not yet available. The only
positive but never-the-less dramatic finding was a reduction in target lesion revascularization. No reduction in
mortality or MI was observed between BMS vs. POBA in these trials.
Key Point
The STRESS and BENESTENT I trials were the initial pivotal trials to demonstrate the superiority of stenting over
balloon PCI in native coronary lesions, with a significantly lower rate of angiographic restenosis and TLR.
Related Text
Click the title below to read text related to this question in the Coronary Interventional Equipment and Techniques
chapter/Stents—Bare Metal, Self-Expanding, and Covered—and Coronary Stenting module.
Pivotal Clinical Trials for Specific Lesion Subsets: Native De Novo Vessels
Procedural Techniques
Question 90 of 130
#734
A 65-year-old patient is seen in your office for evaluation of exertional angina that has progressed over the past 2
months. He undergoes a stress test that shows an inferior reversible defect after walking for 5 minutes on the
treadmill. His coronary angiogram reveals a heavily calcified mid left anterior descending (LAD) stenosis of 90%. In a
large proximal diagonal branch, there is a discrete concentric 70% stenosis. The circumflex artery comes off the left
main at 100 degrees and has an 80% proximal stenosis. The right coronary artery (RCA) is a chronic total occlusion
with bridging collaterals.
105
Which of the following lesions has the highest likelihood of technical success?
• Mid-LAD stenosis.
• RCA.
Submit Answer
That is incorrect
• Reasoning
• References
All lesions except the proximal diagonal stenosis have features that increase the likelihood of failure for
percutaneous coronary intervention (PCI); heavy calcification in the LAD, tortuosity proximal to the circumflex
stenosis, and the presence of bridging collaterals are each predictors of failure for PCI.
Key Point
Several coronary lesion characteristics correlate with a higher percutaneous coronary intervention risk and lower
success rate, and have been included in various lesion classification schemes.
Related Text
Click the title below to read text related to this question in the Fundamentals of Cardiac Catheterization
chapter/Qualitative and Quantitative Angiography module.
Procedural Techniques
Question 91 of 130
#764
A 68-year-old man is referred with a 5-year history of stable angina. His exercise capacity has decreased recently
despite the addition of antianginal therapy. He suffers with diabetes, hypertension, hyperlipidemia, mild chronic
obstructive pulmonary disease, and he is a smoker. Current therapy includes aspirin 81 mg/day, metoprolol 50 mg
twice a day, long-acting isosorbide mononitrate 120 mg/day, and simvastatin 40 mg/day. His physical examination is
normal, with a heart rate of 60 bpm and blood pressure (BP) of 124/84 mm Hg. Body mass index (BMI) is 28.7 kg/m2.
Low-density lipoprotein (LDL) cholesterol level is 102 mg/dl.
A coronary angiogram is performed, which demonstrates a right dominant system with a normal left main artery.
There is an 80% discrete proximal left anterior descending (LAD) artery stenosis, 90% discrete stenosis in a large first
106
obtuse marginal, and a 70% tubular stenosis in the mid segment of the right coronary artery (RCA). The left
ventriculogram demonstrates an ejection fraction (EF) of 35%.
• Addition of ranolazine.
Submit Answer
That is incorrect
• Reasoning
• References
In clinical trials comparing medical therapy with CABG, a survival benefit was noted in patients with three-vessel
disease associated with a reduced EF and/or significant proximal LAD stenosis. PCI is being increasingly used for
revascularization in multivessel disease, but has not been shown to improve survival compared with medical therapy
alone or CABG in patients with stable angina.
Angiotensin-converting enzyme (ACE) inhibitors improve survival in patients with high-risk coronary artery disease
(CAD), particularly among those with reduced EF, but there are no data to support the use of calcium channel
blockers. Ranolazine has not been associated with improved survival in patients with multivessel coronary disease.
Key Point
Calculation of the anatomic SYNTAX (Synergy between Percutaneous Coronary Intervention With Taxus and Cardiac
Surgery) score, Society of Thoracic Surgeons (STS) score, European System for Cardiac Operative Risk Evaluation
(EuroSCORE), and SYNTAX score II is reasonable to aid the heart team decision-making process.
Related Text
Click the title below to read text related to this question in the Coronary Interventional Equipment and Techniques
chapter/Multivessel Coronary Artery Disease module.
Procedural Techniques
Question 92 of 130
#2634
A 74-year-old woman who has diabetes and smokes cigarettes has two-vessel coronary artery disease (CAD), weighs
110 lbs, and is 59 inches tall. She presents with recurring angina pain and a positive troponin. In hospital on
107
medications, the pain recurs and she undergoes diagnostic catheterization, which reveals small caliber coronary
vessels with diffuse CAD. Quantitative coronary angiography analysis reveals that the vessel with the most
threatening lesion that needs to be treated has a reference diameter of 2.25 mm.
Plain old balloon angioplasty (POBA) leaves a 45% residual luminal diameter. At this point, which of the following
statements is most accurate regarding percutaneous coronary intervention (PCI) in this patient?
• The vessel was too small for PCI and the patient should have been managed with optimal medical therapy.
• Choosing a bare-metal stent will give her the best long-term outcome with regards to myocardial infarction
and death.
• PCI with a drug-eluting stent will prevent her from experiencing restenosis in this vessel.
• PCI with drug-eluting stent will yield the lowest rate of target lesion revascularization.
Submit Answer
That is incorrect
• Reasoning
• References
Since she had recurrent angina in spite of in-hospital medical therapy, she should proceed with more than just
optimal medical therapy. Stenting in small arteries is associated with a higher restenosis rate compared with larger
vessels, but is still the most likely to reduce target lesion revascularization in this patient with a suboptimal POBA
result.
Key Point
Stenting in small arteries is associated with a higher restenosis rate compared with larger vessels. Stenting may
reduce restenosis, especially if balloon angioplasty leads to suboptimal results.
Related Text
Click the title below to read text related to this question in the Coronary Interventional Equipment and Techniques
chapter/Stents—Bare Metal, Self-Expanding, and Covered—and Coronary Stenting module.
Procedural Techniques
Question 93 of 130
#2973
A 66-year-old man undergoes drug-eluting stent implantation to his mid left anterior descending artery in the setting
of progressively worsening angina despite optimal medical therapy. He is initiated on dual antiplatelet therapy
(DAPT) with aspirin and clopidogrel. Two weeks later, he presents to the emergency room complaining of right upper
108
quadrant pain consistent with biliary colic. Ultrasound demonstrates gall bladder inflammation and multiple
gallstones. The surgical team is preparing for cholecystectomy and would like to interrupt his DAPT in anticipation of
surgery within the next 5-7 days.
Which of the following would be the most appropriate response to the surgical team?
• Stop both clopidogrel and aspirin and proceed with planned cholecystectomy in 5-7 days.
• Delay surgery by 1 month and continue DAPT until that time. Continue aspirin throughout the perioperative
period.
• Delay surgery by at least 3-6 months and continue DAPT until that time. Continue aspirin throughout the
perioperative period.
• Stop clopidogrel, continue aspirin, and proceed with planned cholecystectomy in 5-7 days.
Submit Answer
That is incorrect
• Reasoning
• References
Premature discontinuation of DAPT in the immediate period after stent implantation is one of the strongest risk
factors for stent thrombosis. The magnitude of risk and impact on mortality rate is inversely proportional to the
timing of occurrence after the procedure.
The timing of noncardiac surgery in patients post-percutaneous coronary intervention should be assessed based on a
risk/benefit analysis that involves consideration of: 1) the risk of stent thrombosis (particularly if DAPT needs to be
interrupted); 2) the consequences of delaying the desired surgical procedure; and 3) the increased intra- and
periprocedural bleeding risk and the consequences of such bleeding if DAPT is continued.
Elective noncardiac surgery should be delayed 30 days after bare-metal stent implantation and optimally 6 months
after drug-eluting stent implantation (Class I, Level of Evidence B).
Key Point
Premature discontinuation of dual antiplatelet therapy (DAPT) is a strong risk factor for DES thrombosis.
Related Text
Click the title below to read text related to this question in the Coronary Interventional Equipment and Techniques
chapter/Permanent Polymer Drug-Eluting Stents module.
Procedural Techniques
109
Submit and Exit
Question 94 of 130
#2960
A 66-year-old man has a recent history of chest pain and a positive stress echocardiogram suggesting a large area of
inferior ischemia. He has been taking his aspirin, metoprolol, and nitrates as directed and experiences exertional
angina at a low workload. He undergoes coronary angiography and you note severe calcification at the ostium of his
right coronary artery (RCA). You intend to perform a percutaneous coronary intervention (PCI) of the RCA.
Which of the following is most closely associated with clinical outcomes in this patient?
• Coronary vasodilators.
• Scoring balloons.
• Coronary atherectomy.
• Stent underexpansion.
Submit Answer
That is incorrect
• Reasoning
• References
Calcified lesions are often complex and challenging. Stent underexpansion is associated with adverse clinical
outcomes. Strategies such as scoring balloons, IVUS, and atherectomy may facillitate PCI, but they have not been
definitely associated with clinical outcomes. Coronary vasodilation has no role for improving outcomes in this case.
Key Point
Coronary calcification is associated with worse PCI outcomes due to vessel dissection, perforation, balloon expansion
failure, device delivery failure, and stent damage and loss.
Related Text
Click the title below to read text related to this question in the Coronary Interventional Equipment and Techniques
chapter/Coronary Atherectomy: Concepts and Practice module.
Introduction
Procedural Techniques
Question 95 of 130
#2809
110
An 85-year-old man with a history of severe chronic obstructive pulmonary disease presents to the cardiac
catheterization laboratory with unstable angina. Coronary angiogram demonstrates severe lesion in the distal left
main with no other disease. After multidisciplinary discussion, the decision is made to proceed with percutaneous
coronary intervention (PCI).
Which of the following is most consistent regarding the use of PCI in this setting?
• Optical coherence tomography (OCT) is associated with superior clinical outcomes compared with
intravascular ultrasound (IVUS) for the evaluation of left main lesions.
• Intravascular ultrasound (IVUS)-guided PCI of the left main coronary artery is associated with reduced
mortality.
• Left main PCI is inferior to coronary artery bypass grafting (CABG) with regard to mortality at 5 years.
Submit Answer
That is incorrect
• Reasoning
• References
IVUS in the left main has been shown to reduce mortality when used to guide PCI. The other options are incorrect:
Bifurcation lesions in the left main have a higher rate of in-stent restenosis compared with ostial and mid-shaft
lesions. The EXCEL, NOBLE, and SYNTAX trials showed that PCI was equivalent to CABG with regard to mortality
endpoints at 3 years. A single stent approach is often appropriate for addressing left main lesions, based on
angiographic parameters such as the length and severity of branch disease.
OCT can be challenging to perform for ostial left main lesions and has not been compared with IVUS. After stenting,
the minimum stent areas that should be achieved at various locations are shown in the Figure 1. Particular attention
should be paid to the circumflex ostium that is the most common site of in-stent restenosis during left main stenting.
111
(Figure 1)
Citation:
Reproduced with permission from Kang, SJ, Ahn JM, Song H, et al. Comprehensive intravascular ultrasound
assessment of stent area and its impact on restenosis and adverse cardiac events in 403 patients with unprotected
left main disease. Circ Cardiovasc Interv 2011; 4(6):562-9.
Key Point
Studies comparing preintervention IVUS or OCT with physiology have produced a wide range of minimum lumen
area (MLA) cutoff values. In general, these studies have a high negative predictive value but a low positive predictive
value, indicating that IVUS or OCT may be useful to defer intervention but should not be used as definitive proof that
a lesion is flow limiting. For the left main coronary artery (LMCA), there is better agreement between IVUS and
physiology; and the best cutoff MLA that determines hemodynamic significance is probably <6.0 mm2. For technical
reasons, OCT is limited in assessing LMCA severity.
Related Text
Click the title below to read text related to this question in the Coronary Interventional Equipment and Techniques
chapter/ntravascular Ultrasound and Optical Coherence Tomography: Principles and Clinical Applications module.
112
Procedural Techniques
Question 96 of 130
#2844
A 65-year-old woman, nondiabetic, comes to you for percutaneous coronary intervention (PCI) after diagnostic
catheterization revealed a blunt, total left anterior descending artery (LAD) occlusion, 10 mm long just beyond the
first septal perforator. There is a large collateral arising from the right ventricular branch of the right coronary artery
going around the apex and reconstituting the distal LAD back to the total occlusion. You try a retrograde approach,
but you cannot cross the cap.
Which of the following is most accurate regarding the current state of the art of chronic total occlusion (CTO)
recanalization?
• If an antegrade approach is initiated, long-term outcome studies support an intraluminal versus a subintimal
strategy.
• Use of adjunctive imaging, such as intravascular ultrasound (IVUS) or optical coherence tomography (OCT), is
imperative in CTO recanalization.
• Current research regarding improved patient selection and lesion characterization allows this operator to
identify patients with a >50% procedural success rate.
• Switching to an antegrade strategy after retrograde failed should not be attempted because of the blunt
geometry of the total occlusion.
Submit Answer
That is incorrect
• Reasoning
• References
The prediction rule based on the J-CTO score was found to be closely associated with the probability of successful
guidewire crossing within 30 minutes, and could be applied for difficulty grading and procedural time prediction for
the interventional treatment of CTO lesions.
Current active areas of research in CTO lesions focus on improved patient selection, use of adjunctive imaging such
as IVUS or OCT, long-term outcomes with current-generation drug-eluting stents, and potential differences in major
adverse cardiac events and the long-term outcomes between the intraluminal or subintimal strategies for CTO
recanalization. However, none of the specific technical options have been conclusively identified as superior.
Key Point
Current active areas of research focus on improved patient selection, use of adjunctive imaging such as intravascular
ultrasound or computed tomography, long-term outcomes with current generation drug-eluting stents, potential
113
differences in MACE, and the long-term outcomes of intraluminal or subintimal strategies for CTO recanalization.
Related Text
Click the title below to read text related to this question in the Coronary Interventional Equipment and Techniques
chapter/Chronic Total Occlusions module.
Future Directions/Conclusion
Procedural Techniques
Question 97 of 130
#2980
A 57-year-old man with diabetes and long-standing systemic hypertension is discharged after a routine,
uncomplicated percutaneous coronary intervention with drug-eluting stent of a calcified 90% mid left anterior
descending (LAD) stenosis. Angiograms from multiple angles show good apposition and a 0% residual stenosis after
high pressure, post-stent deployment balloon dilation with a noncompliant balloon. Because of severe tortuosity in
the proximal LAD, the operator did not want to pass an intravascular ultrasound or do an optical coherence
tomography.
Three days later, the patient appears in the emergency room pale, diaphoretic, short of breath at rest, and with
acute ST-segment changes.
Which of the following is most likely to explain the sudden clinical deterioration in this patient?
• Stent fracture.
Submit Answer
That is correct!
• Reasoning
• References
This most likely mechanism of deterioration is stent thrombosis secondary to stent malapposition, which the
operator did not check for after deployment.
Key Point
Stent thrombosis is almost always related to incomplete stent expansion or vessel dissection and complicates <1% of
stent procedures.
114
Related Text
Click the title below to read text related to this question in the Coronary Interventional Equipment and Techniques
chapter/Stents—Bare Metal, Self-Expanding, and Covered—and Coronary Stenting module.
Complications of Stenting
Procedural Techniques
Question 98 of 130
#2801
Which of the following is the most likely cause of the complication shown in Figure 1?
Submit Answer
115
That is incorrect
• Reasoning
• References
The image demonstrates catheter kinking. Kinking is usually caused by aggressive catheter manipulation, especially
in the setting of severe iliac tortuosity. A long arterial sheath can actually help prevent this complication instead of
cause it. Given intensive quality control, defective catheter material is unlikely to be contributing to catheter kinking.
Key Point
Guiding catheter and guidewire performance is closely tied to their structures.
Related Text
Click the titles below to read text related to this question in the Coronary Interventional Equipment and Techniques
chapter/Coronary Guiding Catheters, Guidewires, and Snares module.
Procedural Techniques
Question 99 of 130
#837
A 57-year-old man is referred for catheterization in the setting of exertional chest discomfort despite optimal
medical therapy and a nuclear myocardial perfusion imaging (MPI) with suggestive, but not definite, ischemia in the
inferoapical segment. Catheterization demonstrates a normal left main, 50-60% proximal left anterior descending
(LAD) stenosis, minimal circumflex disease, and a normal dominant right coronary artery.
In order to correctly evaluate and treat this patient, you should do which of the following?
• Medical therapy.
Submit Answer
That is incorrect
• Reasoning
116
• Related Text / Key Point
• References
To assess the physiologic significance of an intermediate-grade angiographic lesion in an epicardial coronary artery,
FFR is the appropriate modality. IVUS is not recommended for evaluation of intermediate lesions outside the left
main. For an intermediate-grade left main lesion, IVUS has a Class IIa recommendation. Directly stenting an
intermediate lesion of this severity in the presence of an ambiguous MPI would be a Class III indication.
Key Point
There is outcome-based evidence supporting the use of FFR for directing revascularization in patients with single-
vessel disease, multivessel disease, and left main artery disease, both in the stable and unstable setting.
Related Text
Click the title below to read text related to this question in the Coronary Interventional Equipment and Techniques
chapter/Application of Intracoronary Physiology: Use of Pressure and Flow Measurements module.
Procedural Techniques
#816
A 48-year-old woman presents to a community hospital with new-onset substernal chest discomfort accompanied by
nonspecific ST-T wave abnormalities. Her symptoms are relieved by aspirin, intravenous (IV) heparin, and
trinitroglycerin (TNG). She was given 75 mg clopidogrel.
She is transferred to your referral hospital and experiences two more episodes of chest discomfort, each relieved by
increasing her IV TNG infusion rate. Cardiac catheterization is performed and demonstrates normal left ventricular
function and a hazy 75% stenosis in the first obtuse marginal branch of the circumflex artery. The left anterior
descending and right coronary arteries have nonsignificant stenosis. Balloon angioplasty and stent placement are
performed to the obtuse marginal branch using only unfractionated heparin, and no residual stenosis is seen. Her IV
TNG is discontinued.
Four hours after returning to the inpatient unit, she develops chest discomfort similar to her previous pain. An
electrocardiogram (ECG) shows no changes compared with the pre-percutaneous coronary intervention (PCI) tracing.
Sublingual TNG 0.4 mg provides partial relief.
Which of the following is the best strategy for the management of this situation?
• Restart an IV nitroglycerin drip, and obtain serial electrocardiograms (ECGs) and cardiac enzymes.
• Schedule an exercise sestamibi single-photon emission computed tomography (SPECT) study for the first
thing in the morning.
117
Submit Answer
That is correct!
• Reasoning
• References
The major concern for this patient is acute stent thrombosis. Most patients will have typical angina with ECG
changes. In this case, however, the circumflex artery was the treated vessel, and the circumflex coronary artery is
notorious for having electrocardiographically silent myocardial ischemia. In one study, <50% of patients showed ST-
segment changes on the surface ECG during balloon inflation in the circumflex artery. A nuclear study the next day
would be too late to avoid a possible post-intervention myocardial infarction.
Referral for immediate angiography is the best answer for the following reasons: 1) her symptoms are similar to
those she presented with, 2) she received inadequate dual antiplatelet therapy (she did not get clopidogrel load),
and 3) the timing of her recurrent symptoms is consistent with acute stent thrombosis. This may be related to the
suboptimal antiplatelet therapy or a technical factor, such as inadequate stent expansion or apposition, or edge
dissection not detected initially by angiography.
Key Point
Stent thrombosis is almost always related to incomplete stent expansion or vessel dissection and complicates <1% of
stent procedures.
Related Text
Click the title below to read text related to this question in the Coronary Interventional Equipment and Techniques
chapter/Stents—Bare Metal, Self-Expanding, and Covered—and Coronary Stenting module.
Complications of Stenting
Procedural Techniques
#2966
A 65-year-old man developed progressive angina 1 year ago. He underwent nuclear perfusion stress testing that
demonstrated a large area of myocardial ischemia. His history was also notable for anemia due to gastrointestinal
bleeding. He underwent percutaneous revascularization with a single coronary artery stent.
He now presents with recurrent chest pain identical in character to the symptoms he had 1 year ago. A stress test
showed recurrence of myocardial ischemia. Coronary angiography was performed (Figure 1).
Which of the following most accurately describes the current clinical situation?
118
• The angiogram shows the predominant pattern of restenosis with a bare-metal stent (BMS).
• This patient has most likely suffered a rare occurrence after implantation of a drug-eluting stent (DES).
• Prolonged dual antiplatelet therapy (DAPT) would have reduced the frequency of this occurrence.
• Local catheter-based radiation therapy is the optimal treatment for this problem.
Submit Answer
That is correct!
• Reasoning
• References
This patient has diffuse in-stent restenosis. It is the most common pattern of restenosis after BMS. The circulation is
right dominant (large distal vessel). The optimal therapy for treating this problem is implantation of a DES. Prolonged
DAPT has not been shown to reduce restenosis. Vascular brachytherapy delivered by a catheter was used before DES
to treat BMS restenosis and was effective. The current recommendation to treat BMS restenosis is DES.
Key Point
DES restenosis is generally characterized by a focal ISR pattern. Conversely, the predominant pattern of BMS
restenosis is diffuse ISR.
Related Text
119
Click the title below to read text related to this question in the Coronary Interventional Equipment and Techniques
chapter/In-Stent Restenosis module.
Introduction
Procedural Techniques
#2850
You are considering starting a chronic total occlusion (CTO) program at your hospital, but are unsure if there will be
sufficient volume to make it worthwhile.
• Patients with prior coronary artery bypass grafting (CABG) have about as many CTOs as those who have not
undergone CABG.
• Low- and intermediate-volume operators are as likely as high-volume operators to perform CTO
percutaneous coronary intervention (PCI).
• Approximately 20% of patients with coronary artery disease diagnosed by coronary angiography have a CTO.
Submit Answer
That is incorrect
• Reasoning
• References
CTO of one or multiple coronary arteries is demonstrated in approximately 18-26% of patients with coronary disease
having diagnostic coronary angiography. In patients with prior CABG, the incidence may reach 50%.
Key Point
Chronic total occlusion (CTO) of one or multiple coronary arteries is demonstrated in approximately 18-26% of
patients with coronary disease having diagnostic coronary angiography. In patients with prior coronary artery bypass
grafting, the incidence may reach 50%.
Related Text
Click the title below to read text related to this question in the Coronary Interventional Equipment and Techniques
chapter/Chronic Total Occlusions module.
120
Procedural Techniques
#2977
A 55-year old man with a history of diabetes, end-stage renal disease (on hemodialysis), and coronary artery disease
is undergoing coronary angiography for stable exertional chest pain despite optimal medical therapy with multiple
antianginal medications. Noninvasive imaging shows inferior wall ischemia. His coronary angiogram reveals an 80%
lesion in the mid-right coronary artery. The vessel is a small vessel with a minimal lumen diameter of 2.3 mm.
Percutaneous coronary intervention is planned.
Which of the following pharmacologic agents has been consistently shown to reduce in-stent restenosis (ISR)?
• Cilazapril.
• Ticlopidine.
• Dipyridamole.
• None.
• Abciximab.
Submit Answer
That is incorrect
• Reasoning
• References
The CREST trial demonstrated the efficacy of cilostazol in prevention of ISR. Patients were randomized to cilostazol
or placebo. ISR reduction was significantly greater in patients with diabetes (17.7%) compared with patients on
placebo (37.7%; p = 0.01).
Cilazapril, an angiotensin-converting enzyme inhibitor, did not show reduction in ISR in two trials (MARCATOR and
MERCATOR trials). Anticoagulants (glycoprotein IIb/IIIa) and antiplatelets such as dipyridamole (phosphodiesterase
inhibitor) and ticlopidine (ADP receptor inhibitor) have not been shown to reduce ISR.
Key Point
The most important factors associated with increased risk for ISR include clinical factors (diabetes mellitus or chronic
kidney disease), anatomical factors (plaque burden, bifurcation lesions, vessel size, or calcifications), and procedural
factors (stent underexpansion, stent fracture, or residual uncovered plaque), among others.
Related Text
Click the title below to read text related to this question in the Coronary Interventional Equipment and Techniques
121
chapter/In-Stent Restenosis module.
Procedural Techniques
#2637
A 75-year-old diabetic male is admitted to the hospital with acute on chronic systolic dysfunction heart failure (left
ventricular ejection fraction 30%) and elevated troponins suggesting a non–ST-segment elevation myocardial
infarction. The patient underwent three-vessel coronary bypass surgery 4 months earlier involving a left internal
mammary artery (LIMA) to mid left anterior descending (LAD), and saphenous vein graft (SVG) to the left circumflex
(LCx) and an SVG to the right coronary artery (RCA).
He is status/post left below-the-knee amputation for peripheral vascular disease, a palpable right popliteal pulse, but
barely palpable pulses below the right popliteal. His comorbidities include moderate chronic obstructive pulmonary
disease, and an estimated glomerular filtration rate of 35 cc/min.
Diagnostic catheterization reveals total occlusion of his two saphenous vein grafts (SVGs), and a 90% stenosis just
proximal to the distal LIMA to LAD anastomotic site. His ostial LAD and his ostial LCx are both 90% narrowed, and his
native RCA is 100% occluded. He has very severe diffuse distal disease in both the LCx and the RCA.
Which of the following would be the next best step in this patient's management?
• The risk of intervention is too high, and Ranexa should be added to his medical regimen, which currently
includes beta-blocker, angiotensin-converting enzyme inhibitor, and nitrates.
• A quick plain old balloon angioplasty (POBA) of the LIMA to LAD anastomotic stenosis without any
mechanical circulatory support.
• Convene a heart team, including an interventional cardiologist with complex percutaneous coronary
intervention (PCI) experience and a cardiac surgeon, to consider high-risk PCI with Impella support.
• A quick percutaneous coronary intervention (PCI) with a stent of the LIMA to LAD anastomotic stenosis
without any mechanical circulatory support.
Submit Answer
That is incorrect
• Reasoning
• References
122
A heart team, including an interventional cardiologist with complex PCI experience and a cardiac surgeon and
possibly even a vascular surgeon, should be convened in a patient such as this with unprotected left main disease
and complex coronary artery disease (CAD). Unilateral decision making that the patient already failed coronary
artery bypass grafting (CABG) once before, and is likely to fail again secondary to severe diffuse distal disease, redo
CABG is not a viable option. Further, given his significant left ventricular systolic dysfunction and last remaining
vessel circumstance, high-risk PCI with mechanical circulatory support would give the safest result. The team would
need to pay close attention to his peripheral vascular disease in deciding which vascular access to use.
The 2011 PCI guidelines suggest that a heart team approach is recommended in patients with unprotected left main
or complex CAD as a Class II recommendation (Level of Evidence C).
Key Point
In patients with unprotected left main disease and/or complex multivessel disease, a heart team approach should be
used to decide on the coronary revascularization strategy.
Related Text
Click the title below to read text related to this question in the Coronary Interventional Equipment and Techniques
chapter/Multivessel Coronary Artery Disease module.
Procedural Techniques
#2978
A 70-year-old diabetic patient with hyperlipidemia presents with increasing angina on maximal medical therapy.
Coronary angiography reveals diffuse coronary disease with a long 80% lesion in the mid to distal left anterior
descending artery amenable to percutaneous coronary intervention (PCI). The vessel is a 2.5 mm diameter calcified
vessel crossing a large diagonal branch.
Which of the following has been shown to reduce the risk for in-stent restenosis (ISR)?
Submit Answer
That is correct!
• Reasoning
The most important factors associated with increased risk for ISR include clinical factors (diabetes mellitus or chronic
kidney disease), anatomical factors (plaque burden, bifurcation lesions, vessel size, or calcifications), and procedural
factors (stent underexpansion, stent fracture, or residual uncovered plaque), among others.
Diabetes management has not been shown to reduce ISR. Rotablation has been associated with increased rates of
ISR. Single-stent techniques should be used when possible to reduce ISR of bifurcation lesions. IVUS-directed PCI has
been associated with a lower risk of ISR.
Key Point
The most important factors associated with increased risk for ISR include clinical factors (diabetes mellitus or chronic
kidney disease), anatomical factors (plaque burden, bifurcation lesions, vessel size, or calcifications), and procedural
factors (stent underexpansion, stent fracture, or residual uncovered plaque), among others.
Related Text
Click the title below to read text related to this question in the Coronary Interventional Equipment and Techniques
chapter/In-Stent Restenosis module.
Procedural Techniques
#753
A 62-year-old woman with hypertension, paroxysmal atrial fibrillation (AF), nonischemic cardiomyopathy, congestive
heart failure, New York Heart Association class III, and left ventricular ejection fraction of 36% is referred for right-
and left-heart catheterization. She has stopped warfarin for the past 3 days. Her electrocardiogram reveals sinus
rhythm with first-degree atrioventricular block and left bundle branch block (LBBB). Her creatinine is 1.6 mg/dl,
hematocrit is 38%, and potassium is 4.0 mg/dl. She had a prior Holter monitor that revealed frequent premature
ventricular contractions with occasional ventricular couplets.
In reviewing potential complications that may occur during the cardiac catheterization procedure, which of the
following is more likely to occur in this patient than would normally be expected?
Submit Answer
That is incorrect
124
• Reasoning
• References
Cardiac catheterization has a low incidence of arrhythmic complications. However, the occurrence of procedurally
related arrhythmic events is not usually predictable and may have significant risk, particularly for those with
coronary heart disease and structural heart disease.
Although most vagal episodes and catheter-induced arrhythmias self-terminate without consequence, availability of
an external defibrillator, advanced cardiac life support medications, and equipment for temporary pacing
(transvenous wires vs. transcutaneous) are critical when performing catheter-based procedures. Low osmolar and
nonionic radiologic contrast agents generally result in a lower incidence of arrhythmic events related to coronary
angiography.
In this patient with baseline LBBB, there is a risk of trauma to the moderator band that carries the RBBB in the right
ventricle during the right-heart catheterization. This could result in complete heart block, and this contingency
should be anticipated by having a temporary pacemaker readily available during the catheterization procedure.
Key Point
Patient preparation for invasive procedures requires knowledge of the particular patient’s medical condition (i.e.,
history, physical, laboratory data, noninvasive testing) and how it relates to and/or dictates the procedures that
need to be performed.
Related Text
Click the title below to read text related to this question in the Fundamentals of Cardiac Catheterization
chapter/Patient Selection, Preparation, Risks, and Informed Consent module.
Procedural Techniques
#856
A 67-year-old man with type 2 diabetes is referred for percutaneous coronary intervention (PCI) because of
ischemic-type chest pain, a strongly positive nuclear stress test in the inferolateral region, and an 80-90% proximal
stenosis in a large, but not dominant, circumflex artery. In evaluating the patient, you learn that his glycated
hemoglobin (HgbA1c) has fairly routinely been 7-8. The patient’s spouse notes that his blood sugars have always
“been high,” even on hypoglycemic therapy, and asks if this is important relative to his planned PCI.
You explain to them that his glycemic control relative to the planned procedure is which of the following?
That is incorrect
• Reasoning
• References
Numerous studies have indicated that poor glycemic control is related to long-term diabetic complications, including
microvascular problems. The relationship to macrovascular problems has been the subject of some debate. Glycemic
control is not related to short-term procedure success. Also, it does not, per se, appear to be related to contrast
nephropathy, although diabetes is a risk factor for contrast-induced nephropathy (CIN) in the Mehran CIN Risk
Score.
At least one small study has shown that maintaining the HgbA1c <7 has been associated with reduced target vessel
revascularization and improved outcomes post-PCI. All of these data are consistent with the view that improved
diabetic control improves intermediate- and long-term outcomes.
Key Point
Patient preparation for invasive procedures requires knowledge of the particular patient’s medical condition (i.e.,
history, physical, laboratory data, noninvasive testing) and how it relates to and/or dictates the procedures that
need to be performed.
Related Text
Click the title below to read text related to this question in the Fundamentals of Cardiac Catheterization
chapter/Patient Selection, Preparation, Risks, and Informed Consent module.
Procedural Techniques
#1389
A 67-year-old woman with diabetes mellitus, hypertension, peripheral vascular disease, and prior myocardial
infarction (MI) with percutaneous coronary intervention (PCI) to the right coronary artery (RCA) 7 years ago presents
with fatigue and dyspnea. Her index treatment at the time of her MI was a bare-metal stent (BMS) to the mid-RCA.
This subsequently restenosed, and she underwent PCI with a drug-eluting stent within the BMS 2 years ago. She
underwent an exercise stress echocardiogram, and was found to have inferior ischemia. She was taken to the cardiac
catheterization laboratory, and was found to have severe in-stent restenosis within the previously placed stents.
Intravascular ultrasound showed severe neointimal hyperplasia within underexpanded stents.
• Brachytherapy.
Submit Answer
That is incorrect
• Reasoning
• References
In restenosis caused by stent underexpansion, a POBA that expands the stent is often sufficient to prevent further
restenosis. Placement of another stent in an underexpanded lesion may increase the risk of restenosis and possibly
stent thrombosis. Brachytherapy would not fix the underlying problem of underexpansion. Medical therapy may be
reasonable, but the patient is symptomatic with ischemia on a stress test and a high-grade coronary lesion. CABG is
not generally recommended for a single coronary lesion that can be treated with percutaneous revascularization.
Key Point
Certain lesion-specific considerations such as luminal diameter or treatment of in-stent restenosis may still rely on
balloon angioplasty as stand-alone therapy.
Related Text
Click the title below to read text related to this question in the Coronary Interventional Equipment and Techniques
chapter/Balloons and Balloon Angioplasty module.
Procedural Techniques
#798
A 64-year-old otherwise healthy man is scheduled for cardiac catheterization to evaluate typical exertional chest
discomfort relieved with rest. He first noticed symptoms about 3 months ago, and they have been largely unchanged
since they started. His cardiac risk factors include a positive family history for premature coronary disease,
hypertension, and hyperlipidemia. He is currently taking aspirin 81 mg daily, metoprolol 50 mg twice daily, and
simvastatin 40 mg daily. Cardiac catheterization documents two-vessel coronary disease, including 90% proximal left
anterior descending (LAD) and 90% right coronary artery lesions. The territory of myocardium at risk is large, and the
measured left ventricular (LV) ejection fraction is 34%. The anatomy appears suitable for percutaneous coronary
intervention (PCI).
127
Which of the following is the preferred management strategy?
• PCI and coronary artery bypass graft are equally appropriate, so discuss with the patient first.
• Consultation with cardiothoracic surgeon for consideration of elective coronary artery bypass surgery.
Submit Answer
That is incorrect
• Reasoning
• References
The patient should undergo revascularization, as there is a large territory at risk. The guidelines categorize both
coronary artery bypass graft (CABG) surgery and PCI as acceptable revascularization strategies in the patient with
two-vessel coronary artery disease (CAD) and suitable anatomy, normal LV function, and no diabetes. With two- to
three-vessel CAD involving the proximal LAD and either diabetes or abnormal LV function, PCI is categorized as Class
IIb, with the preferred approach being CABG. CABG additionally has been demonstrated to have a mortality benefit
over optimal medical therapy in the 10-year follow-up of the STICH (Surgical Treatment for Ischemic Heart Failure)
trial.
Key Point
Calculation of the anatomic SYNTAX (Synergy between Percutaneous Coronary Intervention With Taxus and Cardiac
Surgery) score, Society of Thoracic Surgeons (STS) score, European System for Cardiac Operative Risk Evaluation
(EuroSCORE), and SYNTAX score II is reasonable to aid the heart team decision-making process.
Related Text
Click the title below to read text related to this question in the Coronary Interventional Equipment and Techniques
chapter/Multivessel Coronary Artery Disease module.
Procedural Techniques
#2842
A 65-year-old woman with a chronic total occlusion (CTO) of her proximal left anterior descending artery (LAD) is
complaining of progressive exertional dyspnea. Diagnostic catheterization reveals good collaterals from her
dominant right coronary artery to the LAD. The distal and mid LAD reconstitute, but there is a long segment of
128
occlusion.
If you decide to offer her CTO recanalization, which of the following statements would be most accurate regarding
CTO techniques?
• Dual injection is not necessary in CTO, especially when the antegrade approach is used.
• In the setting of an "ambiguous" proximal cap and good collaterals, an initial antegrade approach is still
preferred.
• New and improved wires allow for a "wire escalation" strategy based on lesion characteristics.
• If the total occlusion is <20 mm long, new and improved wires and microcatheters will allow antegrade
wiring without the need for dissection and reentry.
• In the setting of favorable lesion characteristics, the operator does not have to prepare for a "hybrid"
approach.
Submit Answer
That is incorrect
• Reasoning
• References
New and improved wires and microcatheters and contemporary advanced percutaneous coronary intervention (PCI)
techniques such as coronary reentry, retrograde techniques, and the hybrid approach have enhanced the procedural
success rate for CTO PCI. A variety of CTO wires are available, designed for various lesion characteristics. Following
the hybrid algorithm approach has been shown to reduce procedure time and improve success. This includes dual
injections and being prepared to go retrograde, particularly if there is an ambiguous proximal cap, poor distal target,
and good collaterals. Despite advances in wire technology, staying intraluminal can still be a challenge and dissection
reentry is an effective alternative strategy.
Key Point
New and improved wires, microcatheters, and contemporary advance PCI techniques such as coronary re-entry,
retrograde techniques, and hybrid approaches for CTO PCI have created an environment where clinical success can
be obtained for appropriate clinical indications despite increased lesion anatomical complexity.
Related Text
Click the title below to read text related to this question in the Coronary Interventional Equipment and Techniques
chapter/Chronic Total Occlusions module.
Contemporary Paradigms for Chronic Total Occlusion Percutaneous Coronary Intervention: Hybrid Approach to
Chronic Total Occlusion Interventions
Procedural Techniques
A 56-year-old man, prior smoker and hypertensive, is referred to you for percutaneous coronary intervention (PCI)
after a diagnostic catheterization reveals a 90% stenosis in the mid left anterior descending artery (3.0 mm
diameter), 1 mm beyond the take-off of a 2.0 mm diameter diagonal branch. The branch appears to have a 60%
ostial stenosis as well.
Which of the following statements is most appropriate regarding the different approaches to bifurcation PCI?
• Provisional stenting of bifurcation lesions reduces the risk of stent thrombosis and major adverse cardiac
events (MACE).
• The Culotte technique is the preferred approach to minimize the risk of stent thrombosis.
• Use of bivalirudin in complex bifurcation PCI will minimize the risk of stent thrombosis.
• The T-stent approach is the preferred approach to minimize the risk of stent thrombosis.
• Provisional stenting is the preferred approach if glycoprotein IIb/IIIa is used to minimize the risk of stent
thrombosis.
Submit Answer
That is correct!
• Reasoning
• References
Bifurcation lesions are associated with a higher risk of stent thrombosis. While there are many different approaches
used to get a good angiographic result, the MACE rates for all are substantial. Currently, the "provisional" approach,
which tries to minimize the number of stents from two (one for each branch) to one stent in the main branch, is
viewed as the approach associated with the lowest MACE rate.
Key Point
Bifurcation lesions are associated with a higher risk of stent thrombosis.
Related Text
Click the title below to read text related to this question in the Coronary Interventional Equipment and Techniques
chapter/Ostial and Bifurcation Lesions module.
Stent Thrombosis
Procedural Techniques
#913
130
A 59-year-old man with obesity, diabetes, and hypertension presented with exertional chest pain for 3 months.
Medications include aspirin, simvastatin 40 mg qhs, metoprolol 50 mg bid, hydrochlorothiazide 25 mg qd, and
amlodipine 2.5 mg qd. His electrocardiogram showed nonspecific ST-T wave abnormalities. The patient was able to
exercise for 6 minutes 20 seconds on the Bruce protocol before the test was terminated for shortness of breath and
fatigue. Single-photon emission computed tomography images revealed a moderate, partially reversible inferior
defect, normal gated wall motion, and an ejection fraction of 65%. Isosorbide dinitrate 60 mg daily was added to his
regimen, but he continued to have symptoms.
Subsequent diagnostic coronary angiography showed no significant disease in the right coronary artery. The left
coronary artery showed no significant disease in the left main artery, an intermediate (70%) lesion in the proximal
left anterior descending (LAD) artery, and a 30% stenosis of the mid left circumflex (LCX) artery.
Submit Answer
That is correct!
• Reasoning
• References
This patient has multiple cardiac risk factors and presents with chronic stable angina. His baseline electrocardiogram
and stress test are abnormal, but he does not have any high-risk features. He is appropriately treated with escalation
of medical therapy. Given ongoing symptoms despite optimal medical therapy, he undergoes an appropriate
diagnostic angiogram, which shows no significant disease in the right coronary artery or the LCX artery to explain an
inferior perfusion defect, but it shows a moderate lesion in the proximal LAD artery.
Given the discordance between noninvasive findings and the angiogram, it is appropriate to perform an FFR of the
LAD artery. Performing PCI of the LAD artery without documentation of hemodynamic significance is not supported
by the Appropriate Use Criteria. Although there is some correlation between IVUS and FFR, the latter is the preferred
strategy for assessing hemodynamic significance of lesions. FFR is indicated in intermediate lesions; therefore, the
LCX lesion does not warrant further evaluation.
Key Point
There is outcome-based evidence supporting the use of FFR for directing revascularization in patients with single-
vessel disease, multivessel disease, and left main artery disease, both in the stable and unstable setting.
Related Text
Click the title below to read text related to this question in the Coronary Interventional Equipment and Techniques
chapter/Application of Intracoronary Physiology: Use of Pressure and Flow Measurements module.
131
Procedural Techniques
#2635
A 60-year-old diabetic, 50-pack-year smoker, is 10 years post successful two-vessel coronary artery bypass grafting
(CABG) (left internal mammary artery [LIMA] to left anterior descending [LAD], and saphenous vein graft [SVG] to left
circumflex [LCx]). He is mostly sedentary on disability post-CABG, but recently has experienced 1 month of stable
angina that occurs reproducibly when he climbs up the one flight of stairs to his apartment and remits when he
reaches the landing and pauses for a minute. His doctor referred him for catheterization, which revealed a patent
LIMA to LAD, the SVG to the LCx was occluded, and his native LCx in the obtuse marginal branch had a smooth, but
eccentric, 70% stenosis. His doctor suggested "fixing" the LCx lesion. He now comes to you for a second opinion. His
high-density lipoprotein is 35, low-density lipoprotein 140, and his hemoglobin A1C is 8.4.
Which of the following best describes guideline-directed medical therapy and risk-factor modification?
• Proceed with percutaneous coronary intervention of the LCx, and then have risk-factor modification
ordered.
• Optimizing this patient's medical therapy and risk-factor modification should be provided to this post-CABG
patient before considering revascularization.
• Based on the FREEDOM (The Future Revascularization Evaluation in Patients With Diabetes Mellitus: Optimal
Management of Multivessel Disease) trial, this patient should first undergo redo CABG.
• Because he has been sedentary since his CABG and is a chronic smoker as well, attempts at risk-factor
modification would not be beneficial.
• In the current era, revascularization strategies are no more risky in post-CABG patients compared with first
time percutaneous coronary intervention or CABG patients.
Submit Answer
That is incorrect
• Reasoning
• References
Guideline-directed medical therapy and risk-factor modification should be provided to post-CABG patients with
stable ischemia before considering revascularization with either percutaneous coronary intervention (PCI) or CABG.
Both revascularization strategies are high risk in post-CABG patients compared with patients without prior CABG.
Post-CABG, the risk of the PCI remains elevated, as does the risk of redo CABG, and this patient would have been
excluded from the FREEDOM trial given his prior CABG.
Key Point
Guideline-directed medical therapy and risk factor modification should be provided to post-CABG patients with
stable ischemia before considering revascularization with either PCI or CABG. Both revascularization strategies are
132
high risk in post-CABG patients compared with patients without prior CABG.
Related Text
Click the title below to read text related to this question in the Coronary Interventional Equipment and Techniques
chapter/Saphenous Vein Graft and Arterial Graft Intervention module.
Procedural Techniques
#2970
Which of the following is the mechanism of restenosis in the stent shown in the image (Figure 1)?
• Overlapping stents.
• Strut fracture.
• Neointimal hyperplasia.
133
Submit Answer
That is incorrect
• Reasoning
• References
The image (Figure 1) shows neointimal hyperplasia causing in-stent restenosis. Determining the mechanism of
restenosis is important, as it can help plan further treatment. Stent underexpansion is important to exclude using
intravascular imaging, as this would require treatment for achieving vessel expansion before additional stent
implantation. Incomplete strut apposition or malapposition is not present in the imaged stent, which is completely
embedded in tissue. The image demonstrates only a single stent layer, and there is no evidence of stent thrombosis
or fracture.
Key Point
The pathologic substrate of ISR is the neointimal hyperplasia (NIH) triggered by endothelial injury after stent
implantation.
Related Text
Click the title below to read text related to this question in the Coronary Interventional Equipment and Techniques
chapter/In-Stent Restenosis module.
Procedural Techniques
#2636
A 70-year-old male with a 40-pack-year smoking history underwent coronary artery bypass grafting (CABG) 5 years
ago. Yesterday, he underwent successful percutaneous coronary intervention (PCI) to the native left circumflex (LCx)
after experiencing an acute coronary syndrome (ACS). In preparing his discharge, the medical student on the ward
asks you whether it would be worthwhile to refer this patient for cardiac rehabilitation (CR)?
Which of the following best describes the role of CR post-PCI in this patient subset?
• American College of Cardiology/American Heart Association guidelines do not recommend CR until 3 months
post-PCI, given higher risk of arrhythmia in these patients.
• CR should be utilized in all post-PCI patients to reduce the rate of recurrent myocardial infarction.
• Prior CABG patients who subsequently undergo PCI should be referred for risk-factor reduction and CR.
• CR should be utilized in all post-PCI patients to reduce the rate of systolic dysfunction/chronic heart failure.
134
Submit Answer
That is incorrect
• Reasoning
• References
Many retrospective analyses and a few prospective analyses demonstrate a beneficial effect of CR on reducing all-
cause mortality. However, the beneficial effect of CR is not consistently evident in reducing the incidence of post-PCI
myocardial infarction or heart failure. CR post-PCI is a Class I indication, but only with Level of Evidence B. Referral to
these programs has been designated a quality performance measure. Aerobic exercise training can generally begin 1-
2 weeks after discharge. There is no stipulation that patients with prior CABG be treated any differently than other
post-PCI or post-ACS patients in this regard.
Key Point
As with all PCI patients, post-CABG patients who undergo PCI should have risk factors optimized and should be
referred to cardiac rehabilitation programs.
Related Text
Click the title below to read text related to this question in the Coronary Interventional Equipment and Techniques
chapter/Saphenous Vein Graft and Arterial Graft Intervention module.
Procedural Techniques
#2981
A 67-year-old man presents to your office with 6 months of severely limiting class III-IV angina. He is seeing you for
the first time, and his prior cardiologist told him that, based on an angiogram performed 1 month ago, he had a
chronically occluded vessel that had no percutaneous options for coronary revascularization. The patient's medical
therapy includes atenolol 100 mg daily, amlodipine 5 mg daily, isosorbide dinitrate 120 mg daily, and ranolazine 500
mg bid. Despite this therapy, he continues to have daily angina with minimal exertion.
A stress test done 2 months earlier reveals severe inferior ischemia with no infarction and preserved left ventricular
function. You offer the patient an attempt at revascularization of the chronic total occlusion. You are able to cross
the lesion with a guidewire, but unable to pass a balloon.
• Microcatheters may be useful to cross the lesion, but not to visualize the distal lumen.
135
• Balloon-assisted modification, specialized microcatheters, laser or orbital atherectomy, guide catheter
extension, and anchor balloon technique may be helpful.
• Guide catheter extensions are not associated with an increased risk of dissection.
Submit Answer
That is incorrect
• Reasoning
• References
There are several observational studies showing an increased risk of long-term mortality among patients with failed
chronic total occlusion revascularization. Diabetes and multivessel coronary artery disease are not associated with
higher mortality risk. Left anterior descending artery intervention is not associated with a higher mortality risk.
Key Point
Solutions for inability to cross a lesion despite guidewire crossing include balloon-assisted modification, specialized
microcatheters (e.g., Corsair, Tornus), laser, rotational and orbital atherectomy, guide catheter extension, and
anchor balloon technique.
Related Text
Click the title below to read text related to this question in the Coronary Interventional Equipment and Techniques
chapter/Coronary Guiding Catheters, Guidewires, and Snares module.
Inability to Cross the Lesion With a Wire: Inability to Cross With Balloon or Stent
Procedural Techniques
#2849
You have been using intravascular ultrasound (IVUS) in your laboratory and are considering getting optical coherence
tomography (OCT) as well.
Submit Answer
136
That is incorrect
• Reasoning
• References
OCT is very sensitive in defining thrombus and differentiating red from white thrombus. While OCT can assess plaque
composition - calcium, lipid, and fibrous tissue - calcium can be confused with lipid. The resolution of OCT is 10x that
of IVUS, with OCT having a resolution of 10 microns and IVUS having a resolution of 100 microns. While OCT uses
contrast, the total volume required is low.
Procedural Techniques
#2843
A 65-year-old man presents with chronic stable angina, but wants to start more vigorous exercise because he wants
to do more hiking and camping with his children. He had a "small heart attack" 3 years ago and was told he was
"safe," because he had a mid left anterior descending (LAD) occlusion with excellent collaterals from the right
coronary artery through multiple septal perforators retrograde filling his distal and mid LAD. However, every time he
walks up a hill quickly, he has to stop from angina, and he wants to pursue an intervention to improve his exercise
capacity.
Which of the following is the most accurate statement that you can share with this patient, based on several
different lesion parameters in the J-CTO score, to give him a sense of the odds of success or failure of your
attempting to recanalize this chronic total occlusion (CTO)?
• In calculating the J-CTO score, the higher the score, the "easier" it will be to achieve procedural success in
guidewire crossing within 30 minutes.
• If he has lesion tortuosity and calcification, a retrograde approach would be associated with more procedural
success than an antegrade approach.
• If he has a blunt stump, lesion calcification, and an occlusion >20 mm, the advent of newer, stiffer wires
would still be associated with a >50% likelihood of guidewire crossing within 30 minutes.
• The J-CTO score is calculated to predict the likelihood of crossing a CTO in an antegrade fashion.
• If he has minimal calcification, a total occlusion length of <20 mm, and a beaked stump, procedural success
to recanalize this CTO would be >50% within 30 minutes.
Submit Answer
137
That is incorrect
• Reasoning
• References
Predicting the likelihood for success and efficiency of CTO percutaneous coronary intervention (PCI) can be
facilitated by various scores, such as the Multicenter CTO Registry in Japan (J-CTO) score and the Progress CTO score.
The J-CTO score is calculated by assigning 1 point for each of 5 parameters: calcification, tortuosity, blunt stump,
occlusion length of ≥20 mm, and previously failed lesion. The J-CTO score stratifies CTOs into 4 difficulty groups: easy
(score of 0), intermediate (score of 1), difficult (score of 2), and very difficult (score of ≥3). The J-CTO score can help
predict the likelihood of guidewire crossing within 30 minutes, but also the likelihood of procedural success in some,
but not all series (Figure 1). In some centers, overall procedural success for CTO PCI appeared to remain stagnant
over time, but on more detailed evaluation, the reason was an increasing complexity of CTO lesions treated, as
reflected by higher J-CTO scores.
(Figure 1)
Legend:
Probability of success of the procedure in each risk category by J-CTO score. Actual GW manipulation time required
for successful GW crossing superimposed in these bar graphs by stratifying into 4 time intervals.
Citation:
Reproduced with permission from Predicting Predicting successful guidewire crossing through chronic total occlusion
of native coronary lesions within 30 minutes: the J-CTO (Multicenter CTO Registry in Japan) score as a difficulty
grading and time assessment tool. J Am Coll Cardiol Intv. 2011;4(2):213-221. doi:10.1016/j.jcin.2010.09.024.
138
Key Point
Recently reported scores predict major adverse cardiovascular events (MACE) during CTO PCI and could help guide
patient selection.
Related Text
Click the title below to read text related to this question in the Coronary Interventional Equipment and Techniques
chapter/Chronic Total Occlusions module.
Procedural Techniques
#907
A 58-year-old woman is undergoing coronary angiography after a stress test showed a large reversible inferior lateral
wall defect. The left coronary arteries have mild atherosclerosis, but no flow-limiting disease. The right coronary
artery (RCA) appears to have a high anterior take off.
Which of the following would be the best catheter to engage the RCA?
Submit Answer
That is incorrect
• Reasoning
• References
This normal variant is best engaged using a catheter with a longer tip; specifically, the Amplatz left catheter often
allows successful engagement.
Key Point
There are specific anatomical differences between the right radial, left radial, and femoral approaches that affect the
choice of guiding catheter, and familiarity with a wide range of guiding catheter shapes is required to match the best
tool to the anatomy.
Related Text
Click the title below to read text related to this question in the Coronary Interventional Equipment and Techniques
139
chapter/Coronary Guiding Catheters, Guidewires, and Snares module.
Procedural Techniques
#865
A 60-year-old man with blood pressure 135/80 mm Hg, but no known vascular disease, has a 42-inch waist, fasting
glucose of 98, and lipid profile with total cholesterol 162 mg/dl, triglycerides 75 mg/dl, and high-density lipoprotein
(HDL) 38 mg/dl.
• Cardiovascular disease risk is low because his low-density lipoprotein (LDL) is in the ideal range.
• HDL cholesterol of 38 mg/dl would represent a criterion for metabolic syndrome for women, but not for
men.
Submit Answer
That is incorrect
• Reasoning
• References
Although his LDL is low, metabolic syndrome (he has waist circumference, blood pressure, and HDL criteria)
increases cardiovascular disease risk. HDL <40 is a criterion for metabolic syndrome for men, whereas <50 is a
criterion for women. Since he does not have diabetes mellitus and chronic kidney disease is not mentioned, he does
not require drug treatment for blood pressure <140/90 mm Hg.
Key Point
Patient selection for invasive diagnostic/therapeutic catheterization procedures requires the operator to have a clear
understanding of the clinical question to be addressed for a particular patient and how to apply the expertise of the
catheterization laboratory to determine a diagnosis and treatment strategy.
Related Text
Click the title below to read text related to this question in the Fundamentals of Cardiac Catheterization
chapter/Patient Selection, Preparation, Risks, and Informed Consent module.
140
Procedural Techniques
#805
A 51-year-old nondiabetic man presents with typical class III exertional angina despite optimal antiangina therapy,
including metoprolol succinate 200 mg daily, isosorbide mononitrate 120 mg daily, and ranolazine 1000 mg twice
daily. A stress test demonstrates severe reversible inferior ischemia and preserved left ventricular function.
Cardiac catheterization documents a 100% occlusion of the proximal right coronary artery (RCA) with bridging
collaterals and an ambiguous proximal cap. The left coronaries are without significant disease, and they fill the distal
RCA via left-to-right septal collaterals. The occlusion appears to be 30 mm long.
• Refer for retrograde chronic total occlusion (CTO) percutaneous coronary intervention (PCI) with a drug-
eluting stent.
• Anterograde attempt at chronic total occlusion (CTO) percutaneous coronary intervention (PCI) with a drug-
eluting stent.
Submit Answer
That is incorrect
• Reasoning
• References
This patient is best treated at an experienced center with a retrograde approach to CTO PCI. The occlusion is >20 mm
with an ambiguous cap and bridging collaterals. These features suggest lower success rates with anterograde CTO
PCI. Revascularization is indicated on the basis of symptoms, maximal medical therapy, and documented ischemia in
the corresponding anatomic territory. CABG for one-vessel non-left anterior descending disease is not an optimal
initial strategy.
Key Point
New and improved wires, microcatheters, and contemporary advance PCI techniques such as coronary re-entry,
retrograde techniques, and hybrid approaches for CTO PCI have created an environment where clinical success can
be obtained for appropriate clinical indications despite increased lesion anatomical complexity.
Related Text
Click the title below to read text related to this question in the Coronary Interventional Equipment and Techniques
chapter/Chronic Total Occlusions module.
141
Contemporary Paradigms for Chronic Total Occlusion Percutaneous Coronary Intervention: Hybrid Approach to
Chronic Total Occlusion Interventions
Procedural Techniques
#849
A 91-year-old patient is admitted with refractory unstable angina. Coronary angiography shows severe and diffuse
disease of the left main, ostial left anterior descending (LAD), and ostial left circumflex (LCX) arteries. You plan to
perform simultaneous kissing stents with two 3.0 x 18 mm stents in the left main into the LAD and the LCX arteries.
• An 8 Fr XB guide.
• A 5 Fr JL guide.
Submit Answer
That is correct!
• Reasoning
• References
In order to perform simultaneous kissing stents, in general, a large internal diameter is needed to accommodate two
3.0 stents. A 7 Fr guide will accommodate two stents, but the use of a smaller Guideliner would preclude kissing
stents. Of the options presented, the 8 Fr XB guide is the best choice, given its ideal shape for LAD percutaneous
coronary intervention.
Key Point
A 6 French (F) guiding catheter is appropriate most of the time. When in doubt, do not hesitate to use a 7F or 8F
guide.
Related Text
Click the title below to read text related to this question in the Coronary Interventional Equipment and Techniques
chapter/Ostial and Bifurcation Lesions module.
142
Procedural Techniques
#2615
A 50-year-old diabetic smoker with new-onset congestive heart failure presents to the hospital in pulmonary edema
and with an elevated troponin. Diagnostic workup reveals an ejection fraction of 25% with multiple wall motion
abnormalities and moderate mitral regurgitation. He is adequately diuresed and stabilized.
Which of the following is correct regarding procedural risk assessment for this patient?
• An intra-aortic balloon pump (IABP) has been demonstrated to lower his risk of PCI if PCI is indicated.
• At this point, his risk of diagnostic angiography is similar to a patient with a normal ejection fraction.
• Given his high-risk features, noninvasive assessment for ischemia is preferred at this point.
• Performing ad hoc percutaneous coronary intervention (PCI) will not increase the risk of the procedure.
Submit Answer
That is incorrect
• Reasoning
• References
The risk of PCI is greater in this patient than diagnostic angiography. Given his high-risk presentation, delineating the
etiology of his systolic dysfunction and degree of coronary artery disease is preferred. IABP has not been proven to
lower periprocedural risk (e.g., BCIS-1 trial, and in ST-segment elevation myocardial infarction patients, CRISP AMI
and IABP-SHOCK II trials).
Key Point
The overall risks of invasive catheterization-based procedures are low. The risks associated with interventional
procedures are higher than for diagnostic cases.
Related Text
Click the title below to read text related to this question in the Fundamentals of Cardiac Catheterization
chapter/Patient Selection, Preparation, Risks, and Informed Consent module.
Procedural Risks
Procedural Techniques
143
#1388
A patient returns to you with angina 6 months after stent placement as part of a clinical study. A coronary angiogram
is performed and you see a lucency within the stent. You decide to perform intravascular ultrasound (IVUS) and have
an available comparison from the time of the stent placement. Figure 1 is from the earlier study and Figure 2 is from
a matched location from the current study.
Compared with the prior IVUS (Figure 1), what is your interpretation of the current IVUS study (Figure 2)?
144
• The current study shows severe intimal hyperplasia, compatible with in-stent restenosis.
145
• The current study shows late stent malapposition.
• There is too much nonuniform rotational distortion in the current study to make any judgment regarding the
lesion.
Submit Answer
That is incorrect
• Reasoning
• References
Nonuniform rotational distortion is not seen in any of these images nor is any significant intimal tissue. The stent
struts are clearly seen in both images with a "peeling" of tissue from the stent at 6 o’clock in the bottom panel.
Thrombus is not visualized but can be difficult to image with IVUS. No calcification is noted in either image. This set
of images is from a patient who underwent brachytherapy, with the Figure 1 image performed at the time of the
original intervention and the Figure 2 image at 6-month follow-up. Late stent malapposition has been seen in
brachytherapy and drug-eluting stents.
Procedural Techniques
#927
A 68-year-old woman with insulin-dependent diabetes mellitus is scheduled for cardiac catheterization and coronary
angiography next week. As a result of her long-standing diabetes, she has a glomerular filtration rate of 45 ml/min
per 1.73 m2.
Which of the following is the most appropriate preparation for her procedure?
• No special preparation is necessary if the procedure is done as the first case, and you anticipate using only a
small amount of contrast.
• Give half of her morning dose of insulin, and give a 500 ml bolus of saline in the procedure room with 20 mg
of intravenous furosemide.
• Give half of her evening dose of insulin the night before her procedure, no insulin the morning of the
procedure, and start hydration with 1.5 ml/kg/hr of saline beginning a minimum of 3 hours before the
procedure.
• Give half of her evening dose of insulin the night before her procedure, no insulin the morning of the
procedure, and give a 500 ml bolus of saline in the procedure room with 20 mg of intravenous furosemide.
146
• Start hydration with 1.5 ml/kg/hr of saline, beginning a minimum of 3 hours before the procedure, and have
50% dextrose available if symptoms of hypoglycemia develop.
Submit Answer
That is incorrect
• Reasoning
• References
As recommended in the 2012 expert consensus document on cardiac catheterization laboratories standards, in
patients with diabetes mellitus receiving insulin, the dose of insulin the night before the procedure is usually reduced
to half the usual dose, and on the morning of the procedure, oral hypoglycemic drugs and insulin are held. If
possible, such patients should be scheduled early in the morning to avoid prolonged fasting. Blood glucose levels
should be checked upon arrival to the laboratory and treated accordingly. The only recommended therapy to reduce
the chance of contrast-induced nephropathy is hydration with isotonic saline 1.5 ml/kg/hr, beginning a minimum of
3 hours before the procedure.
Key Point
Patient preparation for invasive procedures requires knowledge of the particular patient’s medical condition (i.e.,
history, physical, laboratory data, noninvasive testing) and how it relates to and/or dictates the procedures that
need to be performed.
Related Text
Click the title below to read text related to this question in the Fundamentals of Cardiac Catheterization
chapter/Patient Selection, Preparation, Risks, and Informed Consent module.
Procedural Techniques
#833
A 65-year-old man with diabetes and stage III chronic kidney disease presents to the emergency department with a
1-week history of increasing exertional chest pain and transient ST-segment depression on his electrocardiogram
during an episode of pain. His baseline chemistry panel shows an estimated glomerular filtration rate of 40 ml/min.
He is scheduled for coronary angiography, and you wish to minimize his risk of contrast-induced nephropathy (CIN).
147
• N-acetylcysteine.
Submit Answer
That is correct!
• Reasoning
• References
The patient in this clinical scenario is at high risk for CIN due to the presence of chronic kidney disease and diabetes
mellitus. Hydration is the only current strategy (besides limitation of contrast) that can reduce the incidence of CIN.
N-acetylcysteine, renal dose dopamine, and forced diuresis have not been found to be beneficial for the prevention
of CIN.
Procedural Techniques
#2810
What is the Medina classification of the left anterior descending artery/diagonal bifurcation lesion imaged below
(Figure 1)?
148
• 0.1.1.
• 1.0.0.
• 1.1.1.
• 1.1.0.
• 1.0.1.
Submit Answer
That is incorrect
• Reasoning
• References
The Medina classification records any narrowing =50% in each of the 3 arterial segments of the bifurcation in the
following order: proximal main vessel, distal main vessel, and proximal side branch. The presence of significant
stenosis is marked as “1” and the absence as “0”. In this case all three locations have significant angiographic
stenosis; hence, this is a Medina 1.1.1. lesion.
149
Procedural Techniques
#2863
As director of the cardiac catheterization laboratory at your hospital, you participate on the percutaneous coronary
intervention (PCI) quality improvement committee. One of the members notes that your hospital is in the bottom
50th percentile for PCI complications according to your hospital's most recent data from a national registry. This
member inquires as to whether the catheterization laboratory incorporates transradial access for PCI procedures.
You are asked to define the impact of transradial access on PCI complications.
• Vascular access-site complications are equivalent between transfemoral and transradial access.
Submit Answer
That is correct!
• Reasoning
• References
Transradial access has been shown in randomized trials to have higher access site crossover. However, with respect
to PCI complications, transradial access is associated with lower rates of bleeding and vascular access complications.
Radial artery occlusion is exceedingly rarely associated with ischemic symptoms. Stroke is a rare complication of PCI;
its risk is similar between transfemoral and transradial access.
Key Point
Technical percutaneous coronary intervention failure may be higher with transradial compared with transfemoral
arterial access.
Related Text
Click the title below to read text related to this question in the Fundamentals of Cardiac Catheterization
chapter/Vascular Access: Arterial, Venous, and Transseptal Access module.
Procedural Techniques
#2848
A 69-year-old woman with prior three-vessel coronary artery bypass grafting (CABG) (4 years ago) and a 20-year
history of diabetes presents to the emergency department with a non–ST-segment elevation myocardial infarction,
manifest by new electrocardiogram changes in leads 2, 3, aVF, but only mild elevation in serial troponins. Diagnostic
cardiac catheterization reveals a patent left internal mammary artery (LIMA) to left anterior descending (LAD), a
complete occlusion of the saphenous vein graft (SVG) to the left circumflex, and an ulcerated 90% stenosis in the
SVG to the distal right coronary artery (RCA). You make a decision to proceed with ad-hoc percutaneous coronary
intervention (PCI) of the SVG to the RCA stenosis.
Which of the following would be the most accurate statement to share with the patient as your team prepares for
PCI?
• PCI will improve her angina symptoms, but has not been shown to improve survival versus optimal medical
therapy.
• In diabetic patients, redo CABG has been shown to be superior to PCI with regard to survival.
• Retrospective cohort studies show a benefit for less invasive PCI over redo CABG with regard to survival.
• In patients with prior CABG, randomized clinical trials (RCTs) have shown that medical therapy is superior to
the more invasive PCI or CABG approaches.
• PCI will prevent any further myocardial damage, and is indicated to improve survival compared with optimal
medical therapy.
Submit Answer
That is correct!
• Reasoning
• References
In patients with prior CABG, no RCTs have compared strategies of medical therapy versus PCI or CABG. Retrospective
cohort studies generally do not show any benefit of PCI or CABG over medical therapy, or of PCI over CABG, for
survival. Thus PCI in post-CABG patients is not indicated to improve survival. Patients with ischemia in other
locations and those with a patent LIMA to the LAD artery are unlikely to experience a survival benefit from repeat
revascularization.
Key Point
In patients with prior CABG, no randomized controlled trials have compared strategies of medical therapy versus PCI
or CABG. Retrospective cohort studies generally do not show any benefit of PCI or CABG over medical therapy or of
PCI over CABG for survival. Thus, PCI in post-CABG patients is not indicated to improve survival.
Related Text
Click the title below to read text related to this question in the Coronary Interventional Equipment and Techniques
chapter/Saphenous Vein Graft and Arterial Graft Intervention module.
Strategies for Saphenous Vein Graft Percutaneous Coronary Intervention: Stenting vs. Coronary Artery Bypass
Grafting / Stenting vs. Percutaneous Transluminal Coronary Angioplasty
151
Guidelines and Appropriate Use Criteria for Post Coronary Artery Bypass Grafting Patients
Procedural Techniques
#926
A 72-year-old man is undergoing coronary angiography for the evaluation of ischemic heart disease. He has a severe
ostial circumflex stenosis as well as a chronic occlusion of the proximal to mid left anterior descending (LAD) artery
with no evident left-to-left collaterals.
In preparing to image the right coronary artery, you make a mental note that it will be important to do which of the
following during angiography?
• Continue image acquisition with minimal panning until the LAD is visualized.
Submit Answer
That is incorrect
• Reasoning
• References
In patients with chronic total occlusions (CTOs), adequate angiographic images (ideally with minimal panning in a
large field of view) must be obtained in order to visualize distal vessel targets supplied by collateral filling. This can
facilitate decision making regarding the suitability of revascularization by either coronary artery bypass grafting or
CTO percutaneous coronary intervention.
Key Point
A systematic approach to each diagnostic coronary angiogram should be utilized, which includes standard initial
gantry angles, a thorough review of images, and subsequent tailored views.
Related Text
Click the titles below to read text related to this question in the Fundamentals of Cardiac Catheterization
chapter/Optimal Angiography module.
Approach to Angiography
152
153